0% found this document useful (0 votes)
59 views238 pages

��سنوات الجراحة للمقيمين �

The document contains a series of clinical questions and answers related to various medical conditions and treatments. It covers topics such as hypotonic hypovolemic hyponatremia, hypercalcemia management, diabetes insipidus, and surgical procedures, among others. Each question is followed by multiple-choice answers, with correct answers indicated by a '+' sign.
Copyright
© © All Rights Reserved
We take content rights seriously. If you suspect this is your content, claim it here.
Available Formats
Download as PDF, TXT or read online on Scribd
0% found this document useful (0 votes)
59 views238 pages

��سنوات الجراحة للمقيمين �

The document contains a series of clinical questions and answers related to various medical conditions and treatments. It covers topics such as hypotonic hypovolemic hyponatremia, hypercalcemia management, diabetes insipidus, and surgical procedures, among others. Each question is followed by multiple-choice answers, with correct answers indicated by a '+' sign.
Copyright
© © All Rights Reserved
We take content rights seriously. If you suspect this is your content, claim it here.
Available Formats
Download as PDF, TXT or read online on Scribd
You are on page 1/ 238

<Q>Which of the following clinical situations can be associated with hypotonic

hypovolemic hyponatremia?
<C>CHF
<C>SIADH
<C>Cirrhosis
<C>Hyperglycemia
<C+>Gastrointestinal losses

<Q>An asymptomatic patient is found to have a serum calcium level of 13.5 mg/dL
(hypercalcemia). Which of the following medications should be avoided?
<C>Bisphosphonates
<C+>Thiazide diuretics
<C>Pamidronate
<C>Calcitonin
<C>Loop diuretics

<Q>A 12-year-old boy had severe head injury; the syndrome of diabetes insipidus is
characterized by which of the following?
<C>Low serum sodium
<C>High urinary specific gravity or osmolality
<C+>High serum osmolality
<C>Low urine output
<C>Expanded extracellular fluid volume

<Q>What is the predominant metabolic abnormality in pyloric stenosis?


<C>Respiratory alkalosis
<C+>Hypochloremic metabolic alkalosis
<C>Respiratory acidosis
<C>Hyperkalemia
<C>Metabolic acidosis

<Q>The least important in early management of a patient with scald burn includes the
following except:
<C+>Systemic broad spectrum antibiotic administration.
<C>Estimating the percentage of the burned area of the body.
<C>Ensuring a patent airway.
<C>Ensuring a good venous access.
<C>Monitoring the urine output.

<Q>When managing a patient with flame burn; the following may suggest the possibility
of inhalation injury except:
<C>History of burn in a closed space.
<C>Lost eye lashes.
<C>Presence of suits on the nostrils.
<C>Burned and swollen lips.
<C+>Burn on the anterior aspect of the chest.
<Q>Regarding differentiated thyroid carcinoma; the following are true except:
<C>The commonest presentation is clinically solitary thyroid nodule.
<C>It has to be suspected when goiter affects the extremes of age.
<C>It causes voice changes by infiltrating the recurrent laryngeal nerve.
<C>The surgical treatment is total thyroidectomy.
<C+>The parathyroid glands are usually affected and the patient will show signs and
symptoms of hypocalcemia.

<Q>The main aim in conservative non operative treatment of splenic trauma in young
ages is:
<C>To keep its function in producing IgM antibodies.
<C>To keep its function as a reservoir for RBCs.
<C>To keep its hemopoetic function.
<C+>To keep its function against capsulated bacteria.
<C>To keep its function in filtering the blood from the deceased RBCs.

<Q>Regarding clostridium myonecrosis; the following are true except:


<C>It has a significant toxemic symptoms.
<C>The organism is strictly anaerobic.
<C+>Administration of toxoid could be helpful in its management.
<C>Wound debridement is the initial line of its treatment.
<C>It may occur as post-operative complication of above knee amputation.

<Q>The following surgical operations are considered clean and does not need
prophylactic antibiotics except:
<C>Excision of breast fibroadenoma in a healthy female.
<C>Herniotomy in a two years old boy.
<C+>Repair of para-umbilical hernia using a mesh.
<C>Stripping a varicose saphenous vein.
<C>Excision of subcutaneous lipoma.

<Q>The commonest tumor in the spleen is:


<C+>Hemangioma.
<C>Lymphomas.
<C>Sarcomas.
<C>Metastatic tumors.
<C>Myelofibrosis.

<Q>All the following are usually stopped before elective general surgical procedures
except:
<C>Aspirin
<C>Clopidogrel (Plavix)
<C>Monoamine Oxidase Inhibitors (MAOIs)
<C>Warfarin
<C+>Prednisolone
<Q>All the followings are well recognized post-operative complications except
<C+>Urinary incontinence after inguinal hernia repair
<C>Renal failure in jaundiced patients
<C>Deep vein thrombosis after major pelvic surgery
<C>Chvostek’s sign after thyroid surgery
<C>Winging of scapula after breast surgery

<Q>In the early assessment and resuscitation of a trauma patient. All are true except
<C>Shock and congested neck veins may indicate pericardial tamponade and
necessitate pericardio synthesis
<C>A urinary catheter should be inserted if the patient is unconscious
<C+>If the patient is shouting loudly means that he is having partial airway obstruction
<C>Paradoxical movement of chest is serious (flail chest)
<C>A high blood pressure and bradycardia may indicate intra cranial bleeding

<Q>All the followings are predisposing factors for arterial mesenteric vascular disease
except
<C>Low cardiac output states
<C>Recent myocardial infarction
<C>Cardiac arrhythmias
<C>Vasculitis
<C+>Recurrent DVT

<Q>All the followings diseases are transmitted through blood transfusion except
<C>Syphilis
<C+>Typhoid and paratyphoid
<C>Malaria
<C>Hepatitis B and C
<C>Epstien-Bar virus

<Q>All the followings are causes of true hypovolemic shock except


<C>Severe burn
<C>Intestinal obstruction
<C>Bleeding after trauma
<C+>Bee sting
<C>Dysentery and severe diarrhea

<Q>All the followings are true in regard to acquired diverticular disease of the colon
except
<C>Diverticula is the out pouching of the mucosal layer of the colon
<C+>Typically patient suffers of pain after eating
<C>Correspond to the entry of blood vessel to the colon wall
<C>Does not happen in the rectum
<C>Sigmoid colon is the commonest site
<Q>A middle-aged man presents with symptoms that are very typical of intermittent calf
claudication. He is a smoker. Clinical examination however reveals normal palpable
peripheral pulses in both lower limbs and a normal ABPI (ankle brachial pressure
index). He also has symptoms suggestive of lumbosacral spine disease. Which non-
invasive physiological test would you recommend to determine whether ischemia is the
cause of his symptoms?
<C>Resting ABPI
<C+>Exercise ABPI
<C>Toe brachial pressure indices
<C>Spectral waveforms
<C>Pulse volume recording

<Q>All of the followings are causes of lower limb digital neuropathic ulceration; except:
<C>Chronic alcohol excess
<C>Vitamin B12 deficiency
<C+>Systemic sclerosis
<C>Diabetes
<C>Toxins

<Q>Regarding chronic venous insufficiency; all of the followings are true except:
<C>Varicose veins are more prevalent in the developed world
<C+>Obesity is a strong, consistent risk factor for chronic venous insufficiency
<C>Occupation is not a consistent risk factor for varicose veins
<C>There is no clear definition for chronic venous insufficiency
<C>Quality of life can be improved with treatment of chronic venous insufficiency

<Q>All of the following statements about antiplatelet treatment in patients with chronic
limb ischemia are correct except:
<C>Is indicated to reduce the risk of death
<C>May be achieved through administration of 75mg aspirin
<C>May be achieved through administration of 75mg clopidogrel
<C+>May be achieved by administration of warfarin
<C>May be achieved through administration of 325mg aspirin

<Q>The following conditions may be a differential diagnosis of intermittent claudication


except:
<C>Spinal canal stenosis
<C>Nerve Root compression
<C+>Diabetic neuropathy
<C>Symptomatic baker’s cyst
<C>Hip arthritis

<Q>The most offending organism in a diabetic patient with necrotizing fasciitis in the
leg.is:
<C>Actinomyces
<C>Candida
<C>Enterobacter
<C>Histoplasma
<C+>Streptococcus

<Q>A 42 year-old man is admitted following a road traffic accident complaining of pain
throughout his chest. He was fit and well prior to the accident. He is tachypnoeic and in
considerable pain. His brachial blood pressure is 90/60 mmHg and his pulse rate is 115
beats/minute. Both femoral pulses are present. A chest X-ray shows multiple rib
fractures and a right sided effusion the most likely cause is
<C>Pulmonary embolism
<C>Cardiogenic pulmonary edema
<C>Cardiac tamponade
<C>Aortic dissection
<C+>Hemothorax

<Q>Which of the following is true about a pulsatile mass in the abdomen?


<C>It must be an aortic aneurysm
<C+>An ultrasound would be the best initial investigation
<C>No imaging is needed if the mass is not tender
<C>Immediate surgery is indicated
<C>Immediate angiography is indicated

<Q>Patients with varicose veins may present to their doctor because of all of the
followings except:
<C+>Calf pain after walking 200 m that is relieved by resting for 5 minutes
<C>Superficial ulcer on the ankle
<C>Aching discomfort in the calf after prolonged standing
<C>Superficial thrombophlebitis
<C>Spontaneous bleeding from a varix

<Q>One of the following is true about the diabetic foot complications?


<C>The neuropathy is only sensory
<C>There is no relation to ongoing smoking
<C>The metatarsal phalangeal joints do not dislocate
<C+>The prognosis is worse with arterial disease and ongoing smoking
<C>There is no place for surgical management

<Q>The following are critical determinants of patient outcome following injury, except:
<C>Time from injury to definitive care
<C>Presence of a well-organised regional system of trauma care
<C>Protocols and guidelines when clinical experience is limited
<C+>Early mobilization of teams led by doctors to the scenes of injury
<C>Thrombosis prophylaxis

<Q>All of the followings are considered immediate threats to life except:


<C+>Fracture of T6 with a complete spinal cord transaction
<C>Splenic injury with ongoing bleeding
<C>Open pneumothorax
<C>Rapidly rising intra-cranial pressures
<C>Aspiration

<Q>Regarding chronic leg ulcers One of the following is true:


<C>They are generally well managed
<C>80% are due to superficial venous insufficiency
<C+>A biopsy is best done from the ulcer edge
<C>Basal cell carcinoma is the commonest cutaneous malignancy in the leg
<C>Arterial and venous ulcers are easily distinguished

<Q>A 75-year-old female underwent a right-sided total hip replacement. On post-


operative day 10, she complained of discomfort and swelling over her right thigh and
calf. She has been ambulating satisfactorily. Which of the following statements is true:
<C>Bed rest and diuretic therapy should be prescribed
<C+>A CT scan with IV contrast of the lower pelvis and right hip should be done
<C>A Doppler study of the lower limb deep venous system should be performed
<C>Local complications of surgery is the most likely cause for the swelling
<C>A plain X-ray of the right hip is most informative

<Q>Regarding imaging in surgery, all are true except:


<C>In abdominal X-ray,the transverse colon can be seen in various locations in the
centre of the abdomen because it is a totally intra-peritoneal organ.
<C>Gallstones are not usually seen on X-ray.
<C>The presence of gas in the small bowel raises the suspicion of bowel obstruction.
<C+>CT scan is the investigation of choice when gallstones are suspected.
<C>Ultrasound is better for solid organs than gas-filled organs.

<Q>At laparoscopic surgery for gallstones, a trocar is inserted through the midline of the
anterior abdominal wall just below the umbilicus. Which structure(s) would be pierced?
<C>Conjoint tendon
<C>External oblique muscle
<C>Internal oblique muscle
<C+>Linea alba
<C>Rectus abdominus muscle

<Q>Cardiopulmonary bypass machine use is mandatory in all the following except:


<C>Tricuspid valve repair
<C>Ascending aorta replacement
<C+>Coronary artery bypass surgery
<C>Atrial septal defect
<C>Mitral valve replacement

<Q>Bicaval cannulation is mandatory in:


<C>Coronary artery bypass surgery
<C>Mitral valve surgery
<C+>Tricspid valve surgery
<C>Ascending aortic aneurysm
<C>Pulmonary end arterectomy

<Q>The most precise method of assessing the ascending aorta for atherosclerosis:
<C>Finger palpation
<C+>Epiaortic ultrasound
<C>CT scan
<C>Transesophageal echocardiogram
<C>MRI

<Q>Heparin is reversed by:


<C+>Protamine sulfate
<C>Enoxoparine
<C>Factor IIV
<C>Fresh frozen plasma
<C>Vitamin K

<Q>A 45-year-old healthy woman arrives for follow-up after her primary care physician
discovered gallstones incidentally while performing imaging studies for an unrelated
event. She has no complaints and has a healthy diet but is worried about the stones. An
abdominal ultrasound is repeated and demonstrates several stones in her gallbladder
without any wall thickening. What is the recommended management for this patient?
<C>Prophylactic cholecystectomy
<C>Ursodeoxycholic acid
<C>Endoscopic retrograde cholangiography (ERCP)
<C+>Observation
<C>Extracorporeal shock wave lithotripsy

<Q>All of the following are correct regarding liver abscesses except


<C>Most common source of pyogenic liver abscess is the biliary tree
<C>Less than 5% of pyogenic liver abscesses are secondary to trauma
<C+>Gram positive cocci are the most common involved pathogens
<C>Amebic liver abscesses are usually treated by metronidazole only
<C>Diagnosis is usually made by US or CT scan

<Q>All are true regarding fresh frozen plasma, except


<C>Have a shelf life of one year when kept in a deep freeze
<C>Is prepared from single units of fresh blood and rapidly frozen within 4-6 hours of
collection
<C>It contains all the constituents of fresh plasma except platelets
<C>Is given to patients to reverse the action of warfarin
<C+>Is deficient of human albumin

<Q>All of the following are correct regarding lower urinary tract trauma except:
<C> More common in children subjected to blunt abdominal trauma.
<C> Ureteric injury is almost always iatrogenic.
<C> Pelvic fracture is the commonest associated injury.
<C+> Prostatic urethra injury is rarely present.
<C> Extra-peritoneal bladder injury is more common than intraperitoneal injury

<Q>All of the following are true regarding renal ultrasound except:


<C>Uric acid stone are identified by ultrasound
<C>Has limited role in kidney trauma.
<C>Is good to differentiate masses from renal cysts.
<C>Doppler ultrasound is sensitive in evaluating renal vein thrombosis.
<C+>It is part of staging testicular tumors.

<Q>all of the following is true regarding initial management of genitourinary trauma


except:
<C>Bladder injury is present in 10-15% of pelvic fractures.
<C>Mechanism of injury per se may warrant further assessment.
<C>Definitive imaging modality should be sought.
<C+>Diffuse abdominal tenderness points towards retroperitoneal bleeding.
<C>Ultrasound has limited role in renal injury.

45<Q> All of the following medical conditions are associated with increased renal stone
formation except:
<C>Patient receiving chemotherapy.
<C+>High dairy products intake.
<C>Patients with exocrine pancreatic insufficiency.
<C>Patients on chronic mechanical ventilation.
<C>Hyperoxaluria

46<Q>All of the followings are true regarding hydronephrosis except:


<C>It appears first in the fornices.
<C>The effect on renal parenchyma is more in the context of Intrarenal pelvis.
<C>Spotty atrophy of the kidney is secondary to local ischemia.
<C+>The lower the obstruction in the ureter the more the effect on the renal function.
<C>Hydronephrosis can develop without obstruction.

<Q>The first line of treatment of third degree hemorrhoids is?


<C+>Bulk laxatives
<C>Sphincterotomy
<C>Hemorrhoidectomy
<C>Nitroglycerine ointment
<C>Xylocain ointment

<Q>Anal examination of a young patient revealed multiple fissures, sinuses and fistulas.
You should rule out?
<C>Rectal tumor
<C>Sever constipation
<C+>Crohn's disease
<C>Anal digitation
<C>Rectal prolapse

<Q>Regarding rectal prolapse, one is true:


<C>Usually associated with diarrhea
<C>Does not occur in females who never had children (nulliparas)
<C+>Most of the cases are idiopathic
<C>Incontinence is cured after surgery in all cases
<C>Strangulation does not occur

<Q>Regarding colonic polyps, one is true:


<C>All are premalignant
<C>The ideal treatment is colectomy
<C>The size is not related to malignant potential
<C>Villous adenomas are the least malignant
<C+>Malignant transformation is secondary to mutations

<Q>Regarding hereditary nonpolyposis colorectal cancer, one is true:


<C>Inheritance is limited to colorectal cancer
<C>Autosomal recessive
<C>Most of the tumors occur in the rectum
<C>The tumors do not start as polyps
<C+>Synchronous lesions are commoner than in sporadic ones

<Q>A 63 year old man complained of bloody bowel movements. The small amount of
blood coats the stool and has been present on & off for 2 months. Lately, he has been
constipated and stool have become of narrow caliber. What is the likely diagnosis?
<C>Anal fissure
<C>Hemorrhoids
<C+>Colonic tumor
<C>Diverticular disease
<C>Ulcerative colitis

<Q>Which of the following is the most important prognostic determinant of survival after
treatment for colorectal cancer?
<C+>Lymph node involvement
<C>Site of the tumor in the colon
<C>Tumor size
<C>Histologic differentiation
<C>DNA content

<Q>The most definitive diagnostic technique for tracheobronchial injuries is


<C> Chest X-Ray
<C> CT scan of the chest with IV contrast
<C+> Flexible laryngobronchoscopy
<C>MRI of the chest
<C> Thoracic ultrasound

<Q>All of the following may be responsible for respiratory failure in flail chest except:
<C>Chest wall pain
<C>Alteration in chest wall compliance
<C+>Decreased work of breathing
<C>Pulmonary contusion
<C>Hemopneumothorax

<Q>One of the following is the most important predictor of mortality and morbidity in
thoracic surgical procedures
<C> FEV1/FVC
<C+>Predicted post-operative FEV1 and DLCO
<C>FEV1 absolute value
<C>Resting PO2
<C>Resting PCO2

57<Q>All of the following are clinical factors in favor of benign solitary pulmonary
nodule except:
<C>Age less than 40
<C>Patient is nonsmoker
<C>No previous history of malignant disease
<C>Patient is immunosuppressed
<C+>Presence of systemic symptoms like weight loss and osteoarthropathy

<Q>Which of the following represents the most common cause of esophageal


perforation/rupture
<C>Blunt trauma (deceleration and steering wheel).
<C+>Esophageal instrumentation (iatrogenic).
<C>Foreign body aspiration/ impaction.
<C>Penetrating civilian trauma.
<C>Spontaneous post-emetic rupture (boerhaave's syndrome).

<Q>Increased levels of β-HCG and/or α-FP are specific for the following mediastinal
tumor:
<C>Neurogenic
<C>Thymic
<C>Lymphoma
<C+>Germ cell
<C>Endocrine

<Q>Inability to create a high-pitched sound following total thyroidectomy may be due to


damage of one of the following
<C+>External branch of the superior laryngeal nerve
<C>Internal branch of the superior laryngeal nerve
<C>Recurrent laryngeal nerve
<C>Ansa cervacalis nerve
<C>Deep cervical plexus

<Q>All of the following are correct regarding papillary carcinoma of thyroid, except
<C>Can be reliably diagnosed using fine needle aspiration cytology
<C+>Usually it is uni- focal
<C>Typically spreads to the cervical lymph nodes
<C>Requires a total thyroidectomy
<C>Is responsive for radioiodine ablation

<Q>Regarding femoral hernia, all of the following statements are true, except
<C>Are more common in women than in men
<C+>Should only be repaired if symptomatic
<C>It has a high incidence of strangulation
<C>It appears in the upper medial thigh just below the inguinal ligament
<C>Small-bowel obstruction may be the presenting feature in some patients.

<Q>Concerning strangulated hernias, all are true except:


<C>Usually present with local then general abdominal pain and vomiting
<C>Hernia can strangulate at any time
<C>Strangulation is more common in hernias with narrow necks such as femoral hernia
<C>Femoral hernias are more likely to strangulate
<C+>Bluish discoloration of skin over the hernia is mandatory for diagnosis.

<Q>All of the following statements are true regarding the natural history of inguinal
hernias, except
<C>Hernias in babies are a result of a persistent processus vaginalis
<C>Direct hernias become more common in elderly men
<C> Among females indirect inguinal hernias are more common than femoral hernias
<C>Direct inguinal hernias are acquired
<C+>Indirect inguinal hernias may disappear spontaneously later in life

<Q>All of the followings are recognized risk factors for breast cancer except:
<C>Positive family history
<C>Alcohol intake
<C+>Late menarche
<C>Delaying full term pregnancy till after the age of 33 years
<C>Obesity

<Q>A 33-year-old lady presented with a 2×3 cm ulcerative lesion in the medial aspect of
her right breast. Which statement about her staging is correct?
<C>This is a Tx tumor
<C>This a T1 tumor
<C>This is a T2 tumor
<C>This is a T3 tumor
<C+>This is a T4 tumor

<Q>A 30-year-old lady presented with a 2×2 cm breast mass. True –cut biopsy
revealed infiltrating ductal carcinoma with strong estrogen receptor positivity. Estrogen
receptor positivity in this patient denotes:
<C+>Better prognosis
<C>Ovarian metastasis
<C>Resistance to treatment by tamoxifen
<C>Liver disease
<C>The need to add avastin to her treatment protocol.

<Q>All of the followings suggest carcinoma in mammography except:


<C>Microcalcification
<C>Destruction of breast contour
<C>The presence of a spiculated lesion
<C>The presence of a hyperdense lesion
<C+>Macrocalcification

<Q>Neo-adjuvant chemotherapy is best described by which of the following


statements?
<C+>Chemotherapy given in order to down –stage a solid tumor
<C>Chemotherapy given in order to palliate metastataic symptoms
<C>Chemotherapy given to ablate the bone marrow preparing for bone marrow
transplantation
<C>Chemotherapy given after loco-regional resection of a solid tumor with negative
metastatic workup.
<C>Chemotherapy given prophylactically to family members at a high risk of developing
a certain solid tumor

<Q>Which of the followings is the most characteristic sign of a fibroadenoma?


<C>Reducibility
<C+>Mobility
<C>Associated axillary lymph node enlargement
<C>Skin retraction (puckering)
<C>Trans- illumination

<Q>Tumor grading is of great significance in predicting the prognosis and guiding the
treatment of all the following tumors except:
<C>Soft tissue sarc
<Q> An elderly diabetic patient presents with gangrene of the right 2nd toe. Clinically she has palpable femoral pulses
and popliteal pulses but no pulses palpable distal to this. The 2nd toe requires amputation. Which test would you use to
determine whether the perfusion is adequate for healing of the amputation site?
<S>Y
<C> Resting ABPI
<C> Exercise ABPI
<C+>Toe brachial pressure indices
<C> Spectral waveforms
<C> Pulse volume recording

<Q> Which of the followings is best in the assessment of vascular tree on a patient who wishes to donate a kidney as a
live-donor for his brother.
<C> Ultrasound
<C> Digital subtraction angiography
<C> CT angiography
<C+>MRA
<C> Abdominal x-ray

<Q> One of the following agents is not considered as an adjunct to control major haemorrhage induced by a vascular
disease process, trauma or surgery:
<C> Fresh frozen plasma
<C> Prothrombin complex concentrate
<C> Recombinant activated factor VII
<C+>Tissue plasminogen activator
<C> Tranexamic acid

<Q> A 76 year old patient is admitted for open aortic aneurysm repair. He has suffered a previous deep venous
thrombosis. Which of the following is an appropriate single method of perioperative thromboprophylaxis?
<C> Warfarin
<C+>Factor Xa inhibitors
<C> Early mobilisation
<C> Intraoperative pneumatic calf compression
<C> Dextran

<Q> A 24 year-old man is brought into the Emergency department having been stabbed with a screwdriver. He is
conscious. On examination he is tachypnic and has a tachycardia of 120 beats/minute. His blood pressure is 90/50
mmHg. He has a small puncture wound below his right costal margin. A central venous line is inserted with ease, and his
central venous pressure is 17 cm. A chest X-ray shows a small pleural effusion with a small pneumothorax. He has
received two units of plasma expander, which has failed to improve his blood pressure. The most likely diagnosis is:
<C> Acute coronary syndrome
<C> Pulmonary embolism
<C> Acute respiratory distress syndrome
<C+>Cardiac tamponade
<C> Aortic dissection

<Q> A 64-year-old non-smoker complains of severe foot pain during the night .It awakes him from sleep. He has recently
begun to dangle the leg off the end of the bed, as this seems to alleviate some of the pain. In this patient, which of the
following is true?
<C+>Rest pain is an indication for surgical revascularization
<C> Pedal ulcers are likely to heal quickly due to the robust granulation tissue that forms in those with ischemic rest pain
<C> These days, prior to surgical intervention, most patients are routinely sent for medical optimization despite no
observed improvement in perioperative morbidity and mortality
<C> A percutaneous intervention at the level of the common iliac artery is less likely to last 5 years than one at the
common femoral artery
<C> PTA is ideal for opening up long segments of stenosis
<Q> Which of the followings is true about arterial trauma?
<C> All arterial injuries are associated with pulsatile bleeding
<C> The commonest cause is penetrating injury
<C> The distal pulses will always be absent
<C> There is no relationship to major joint dislocations
<C+>Bleeding is more likely with partial than with complete arterial transection

<Q> A 32-year-old male involved in a head-on motor vehicle collision presents with chest pain and the following vital
signs on arrival in the emergency department: Heart rate – 120/minute; Blood pressure – 86/50; GCS score – 10; and O2
saturations of 92%. Neck veins are distended and breath sounds are absent on the right side. Which of the following takes
first priority?
<C> Urgent CT scan of the head to rule out an extradural hemorrhage with midline shift
<C> Rapid resuscitation with 2 large-bore intravenous cannula and warmed fluids
<C> ECG and an echocardiogram to eliminate cardiac contusion as the cause of his hypotension
<C> Obtaining an urgent crossmatch
<C+>Elimination of tension pneumothorax as a cause of his symptoms/signs

<Q> Hemorrhage initiates a series of compensatory responses. All of the followings statements are true concerning the
physiologic responses to hemorrhagic shock EXCEPT?
<C> An immediate response is an increased sympathetic discharge with resultant reflex tachycardia and vasoconstriction
<C> Transcapillary refill is a response serving to restore circulating volume
<C> Extracellular fluid becomes increasingly hyperosmolar
<C+>An immediate rise in the level of thyroxine hormone is the most recognized response
<C> An increased release of stress hormones coupled with relative insulin resistance

<Q> An otherwise fit 57-year-old man spikes a temperature of 39◦C five days after appendicectomy. There is a tender,
reddened and fluctuant swelling at the medial end of the wound. What is the most appropriate initial action to take?
<C> Arrange a CT scan of the abdomen
<C> Arrange an ultrasound scan of the wound and anterior abdominal wall
<C> Start the patient on oral antibiotics
<C+>Open the wound to allow free drainage
<C> Send off blood samples for a white cell count and culture

<Q> The following statements on acute lymphangitis of the lower limb are correct EXCEPT:
<C> Improperly managed, it may lead to lymphadenitis
<C+>Lymphangiography is the investigation of choice in the management
<C> Rest and elevation of the affected limb is appropriate
<C> Cellulitis may be the initiating cause
<C> Appropriate antibiotics should include cover for streptococcal infection

<Q> With regard to ischemic colitis, which of the following statements is true?
<C> Usually painless
<C> Usually associated with melena
<C> Occlusion of the major mesenteric vessels is responsible in most cases
<C+>The splenic flexure and descending colon are the most vulnerable
<C> Non-operative management is not justified

<Q> A 60 year old male patient underwent surgery for a left side colonic tumor. Metastatic work up was negative. The
pathology report revealed that the tumor is an adenocarcinoma invading into the peri-colic fat, with 2 involved lymph
nodes. After the patient recovers from surgery, which of the following is the most appropriate next step in his treatment?
<C> No further therapy
<C+>Chemotherapy
<C> Targeted therapy (monoclonal antibodies)
<C> A combination of chemotherapy and targeted therapy
<C> Radiotherapy
<Q> A 60-year-old asymptomatic man presented for a routine check. The examination was normal except for a palpable
mass in the rectum on digital rectal examination. The patient denies any change in bowel habits and feels well. Rectal
cancer is suspected. What is the next best step in the evaluation?
<C> Computed tomography scan of the abdomen and pelvis
<C> Double-contrast barium enema
<C> Flexible sigmoidoscopy with biopsy of the lesion
<C+>Full colonoscopy with biopsy of the lesion
<C> Magnetic resonance imaging scan of the abdomen and pelvis

<Q> A 55-year-old man is hospitalized with a first attack of acute diverticulitis. He has acute left lower abdominal pain
with a palpable tender mass just above the left groin area. Steps in his management during the first 24 h after admission
should include intravenous fluids and:
<C+>Broad-spectrum antibiotics
<C> Diagnostic colonoscopy
<C> Saline enema to evacuate any retained stool
<C> Nasogastric suction
<C> Sigmoid resection once he is well-hydrated

<Q> A 70-year-old man with severe atherosclerosis underwent cardiac catheterization because of chest pain. Later in the
day, he developed severe abdominal pain and passed a large amount of bloody diarrhea. Physical examination revealed
no peritoneal signs. Which of the following is the most likely cause of the patient’s bleeding?
<C> Colon cancer
<C> Diverticulitis
<C> Bleeding from anticoagulation
<C+>Mesenteric ischemia
<C> Nonsteroidal anti-inflammatory drug enteropathy

<Q> A 65 year old woman had an attack of sigmoid diverticulitis diagnosed by CT scan and successfully treated by IV
antibiotics 6 weeks ago. She is currently asymptomatic. Your next step is?
<C> Barium enema
<C> Water-soluble contrast enema
<C> Repeat CT scan of abdomen and pelvis
<C+>Colonoscopy
<C> Sigmoid resection

<Q> Which of the following may be appropriate initial therapy for a 4 cm cancer of the anal canal?
<C> Local excision
<C> Abdominoperineal resection
<C+>Combined chemotherapy and radiotherapy
<C> Laser therapy
<C> Cryotherapy

<Q> A 60 year old man known to have hemorrhoids reported bright red blood in the toilet paper after defecation.
Anoscopy revealed hemorrhoids. What is next step in the management?
<C> High fiber diet and bulk laxatives
<C> sigmoidoscopy
<C+>Colonoscopy
<C> Hemorrhoidectomy
<C> Nitroglycerine ointment

<Q> A 28-year-old woman presents with a 3-month history of chronic right lower abdominal pain. The patient
occasionally had fever. She had 3 to 4 loose stools per day that contain mucus but not blood. She has lost 5 kg. She also
reported occasional eye pain with light sensitivity. On examination, her abdomen is focally tender in the right lower
quadrant without peritoneal signs or palpable masses. What is the most likely underlying diagnosis in this patient?
<C> Chronic appendicitis
<C+>Crohn’s disease
<C> Irritable bowel syndrome
<C> Systemic lupus erythematosus (SLE)
<C> Meckel's diverticulitis

<Q> Which of the following statements about lung cancer is TRUE?


<C> Most common type of non-cutaneous cancer in women
<C+>Most common cause of cancer death in men and women
<C> Deaths reduced by 25% in smokers undergoing annual screening with chest x-ray and sputum cytology
<C> Metastatic disease to regional or distant sites already present in one fourth of patients at diagnosis
<C> Overall 5-year survival rate 40%

<Q> A 65-year-old man develops a 1-cm nodule in the right lung 2 years after sigmoid resection for colon cancer. He
has previously had wedge resection of his left lung for a penetrating injury. Which of the following is the BEST
predictor of the risk of pulmonary complications and mortality after lung resection for this patient?
<C> Age > 60 years
<C> History of lung resection
<C+>Predicted postoperative FEV1< 40%
<C> Preoperative PaCO2 > 45 mmHg
<C> Preoperative PaO2 < 60 mmHg

<Q> Which of the following represents the most common cause of esophageal perforation \ rupture
<C> Blunt trauma (deceleration and steering wheel).
<C+>Esophageal instrumentation (iatrogenic).
<C> Foreign body aspiration/ impaction.
<C> Penetrating civilian trauma.
<C> Spontaneous post-emetic rupture (Boerhaave's syndrome).

Which of the following statements about diaphragmatic anatomy is true


a. The azygos vein passes through the aortic hiatus.*
b. The foramen of Morgagni is a posterior diaphragmatic defect.
c. Innervation is by the phrenic nerves which arise from the lower trunk of the cervical plexus.
d. The thoracic duct passes through the esophageal hiatus.
e. The vena caval foramen is located between the diaphragmatifcrura that arise from the lumbar vertebrae.

f. The most common type of Barrett’s epithelium is


a. Gastric fundic type epithelium.
b. Junctional type epithelium.
c. Low-grade dysplasea.
d. Metaplastic changes associated with gatroesophageal reflux.
<C+>Specialized intestinal type.

g. All the following statements about hydatid disease are correct except:
<C+>Humans get the infection by ingesting the ova secreted in the stool of sheep
a. The liver is the commonest site of cyst formation in humans
b. Intrabiliary cyst rupture may present as cholangitis
c. Rupture of the cyst may be associated with anaphylactic reactions
d. Medical treatment by albendazol is occasionally added to surgical treatment.

h. All are true regarding appendicitis except:


a. The initial peri-umbilical pain is a mid-gut visceral pain caused by distension of the lumen of the appendix.
b. The right iliac fossa pain is a somatic pain due to irritation of the parietal peritoneum.
c. Pelvic ultrasound in appendicitis is a useful diagnostic tool especially in females
<C+>CT scan of the abdomen is routinely used to confirm the diagnosis of appendicitis because of its high sensitively and
specificity.
d. Laparoscopic approach for appendectomy has advantages over open approach especially in females and obese
patients.
i. Which of the followings may be an indication for operation for mitral stenosis?
a. Symptomatic patient with mitral valve area less than 1.5 m2
b. Mitral valve area less than 1 m2
c. Embolisation
d. Pulmonary hypertension
<C+>Decreased transvalvular gradient on exercise echocardiography

j. Which of the followings are relative indications for mechanical, as opposed to tissue, valve replacement?
<C+>Patient younger than 30 years.
a. Young female patient who desires children.
b. An elderly patient.
c. Tricuspid valve replacement.
d. Chronic renal failure

k. Pancreatic cancer was found to be associated with all the followings except:
a. Smoking tobacco
<C+>Achlorhydria
b. Diabetes mellitus
c. Multiple endocrine neoplasia type 1 MEN1
d. Alcohol consumption

l. All the followings are true about acute pancreatitis except:


a. Chronic alcohol ingestion is a common cause in the west
b. Vomiting and belching are recognized symptoms
<C+>Pain is increased by leaning forward
c. Amylase level is the most important prognostic sign
d. Hemorrhagic pncreatitis is associated with a high mortality.

m. A 50 year old man presented with early satiety and weight loss. Physical examination revealed an enlarged hard left
supra-clavicular lymph node. He is a heavy smoker. The most likely diagnosis is:
a. Sigmoid carcinoma
b. Lung carcinoma
c. Oesophageal carcinoma
<C+>Gastric carcinoma
d. Gall bladder carcinoma

n. A 60 year old man presented with gradually enlarging swelling over his left parotid area for the last 6 months. During
the last few weeks the mass became painful with skin reddening above it. The patient noticed drooping of the left side of
his lips. The most likely diagnosis is:
a. Suppurativeparotitis
<C+>Adenoid cystic carcinoma
b. Parotid stone
c. Pleomorphic adenoma
d. Tuberculosis

o. Regarding venous thromboembolism (VTE), all are true EXCEPT:


a. Risk of VTE is higher in pelvic surgery.
b. Patients with varicose veins and phlebitis are at increased risk of VTE.
c. Intermittent pneumatic compression devices are effective mechanical prophylaxis methods.
<C+>Antiplatelet agents are the most effective drugs for VTE prophylaxis.
d. Inferior vena caval filters are sound prophylactic measures against VTE when mechanical and pharmacological
prophylaxis are contraindicated.

p. All the followings are recognized risk factors for breast cancer except:
a. Positive family history
b. Alcohol intake
<C+>Late menarche
c. Delaying full term pregnancy till after the age of 33 years
d. Obesity

q. A 30 year old lady presented with a 2×2 cm breast mass. True –cut biopsy revealed infiltrating ductal carcinoma with
strong estrogen receptor positivity. Estrogen receptor positivity in this patient denotes:
<C+>Better prognosis
a. Ovarian metastasis
b. Resistance to treatment by tamoxifen
c. Liver disease
d. The need to add avastin to her treatment protocol.

r. All the followings suggest carcinoma on mammography except:


a. Microcalcification
b. Destruction of breast contour
c. The presence of a speculated lesion
d. The presence of a hyperdense lesion
<C+>Macrocalcification

s. Which of the followings is the most characteristic sign of a fibroadenoma?


a. Reducibility
<C+>Mobility
b. Associated axillary lymph node enlargement
c. Skin retraction (puckering)
d. Trans- illumination

The possibility of a false positive diagnosis of cancer is more likely in which of the followings?
a. Core needle biopsy
b. Incisional biopsy
c. Fine needle aspiration biopsy*
d. Frozen section evaluation
e. Exisional biopsy

Which of the followings is true regarding wound contracture?


a. Is seen mainly in wounds closed primarily
b. It is based on specialized monocytes responsible for phagocytosis of collagen
a. It contributes significantly to the reduction in the size of an open wound*
b. Bacterial colonization significantly slows the process of contracture
c. Diabetic patients experience an exaggerated wound contracture

c. One of the following statements about HER II- neu positive breast cancer is correct:
a. Avastin is used with Her II-new overexpression
b. Her II-new overexpression necessitates adjuvant radiotherapy
c. Her II- neu mutation is encountered in 60 % of breast cancer patients.
<C+>Her II-new overexpression indicates a worse prognosis
d. FISH technique is indicated if immune-histo-chemistry for HER II-neu shows a +3 result

d. The most common arrhythmia created by laparoscopy is :


<C+>Bradycardia
a. Atrial fibrillation
b. Supra-ventricular tachycardia
c. Ventricular fibrillation
d. Pulsusalteranc

e. Neo-adjuvant chemotherapy is best described by which of the following statements?


<C+>Chemotherapy given in order to down –stage a solid tumor
a. Chemotherapy given in order to palliate metastataic symptoms
b. Chemotherapy given to ablate the bone marrow preparing for bone marrow transplantation
c. Chemotherapy given after loco-regional resection of a solid tumor with negative metastatic workup.
d. Chemotherapy given prophylactically to family members at a high risk of developing certain solid tumor

f. All the followings are sound anti-estrogen treatments in estrogen positive breast cancer except:
a. Oophorectomy in pre-menopausal patients
<C+>Specific aromatase inhibitors in premenopausal patients
b. Tamoxifen in post menopausal patients
c. Goserline ( LHRH agonist) in premenopausal patients
d. Estrogen receptor downregulators in postmenppausal patients\

g. The commonest presentation ( symptom) of breast cancer is:


<C+>Painless breast lump
a. Bleeding from the nipple
b. Paget’s disease of the nipple
c. Painful ulceration of the skin of the breast
d. Formation of an abscess inside the breast

h. The commonest subtype of breast cancer is:


<C+>Infiltrating ductal carcinoma NOS (Not Otherwise Specified)
a. Infiltrating lobular carcinoma
b. Lymphoma of the breast
c. Phylloides tumor
d. Ductal carcinoma in situ

i. The severity of inhalational burn injury is estimated By:


a. Blood gas analysis.
b. Chest x-ray.
c. Oxymetry.
<C+>Carboxy hemoglobin level.
d. Indirect laryngoscopy.

j. The most important treatment strategy in chemical burn is:


<C+>Copious irrigation of the area with water.
a. Ruling out smoke inhalational injury.
b. Prompt intravenous antibiotics.
c. Urgent fasciotomy .
d. Urgent CT abdomen to rule out acute liver failure from possible cutaneous absorption of the involved chemical
material.

k. The commonest cause of a clinically solitary thyroid nodule is:


a. Physiological goiter.
b. Hashimoto’s thyroiditis.
<C+>Multinodular goiter.
c. Papillary adenoma.
d. Medullary carcinoma

l. Regarding necrotizing fasciitis the followings are true Except:


a. It usually has a short incubation period to show.
b. The patient usually shows toxemia symptoms.
c. The spread of infection and necrosis is faster along the subcutaneous tissue planes than in the skin.
<C+>The causative organism is usually single gram negative bacteria.
d. It affects mostly patients with decreased immunity
m. The best approach to diagnose splenic injury is:
a. Observing enlarging abdominal girth.
b. Repeating hematocrit evaluation.
c. Performing peritoneal lavage.
d. Demonstrating fractured left lower ribs by chest x-ray.
<C+>Performing abdominal CT scan.

n. In intestinal obstruction, plain abdominal X-ray is least accurate in showing:


a. Air-fluid level.
b. Level of the obstruction.
<C+>Strangulated intestinal loop.
c. Closed loop obstruction.
d. Pseudo obstruction.

o. When observing a patient with simple adhesive intestinal obstruction; the most important to decide on operative
intervention is:
a. Vomiting feculant material.
b. Increase hematocrite level.
c. Leucocytosis.
d. Persistence of the symptoms for more than 48 hours.
<C+>Localization of the abdominal signs.

p. Regarding branchial cyst; the followings are true Except:


a. It usually shows positive fluctuation.
<C+>It has positive trans- illumination to light.
b. It represents remnant of the second branchial cleft.
c. Its fluid contains cholesterol crystals.
d. Its tract reaches the lateral pharyngeal wall

q. Concerning Hirschsprungs disease, which of the following is TRUE?


a. More common in females.
<C+>Absent ganglion cells in the Meissner and Auerbach plexuses.
b. Failure to pass meconium in the first 48 hours of life.
c. Best diagnosed by lower gastrointestinal contrast-enhanced study.
d. Atrophy of submucosal nerve endings seen on rectal biopsy specimen.

r. The most common cause of duodenal obstruction at birth is:


a. Malrotation.
<C+>Duodenal atresia.
b. Annular pancreas.
c. Midgut volvulus.
d. Preduodenal portal vein.

s. In case of intussuception ,hydrostatic reduction is contra-indicated in one of the followings:


a. After 48 hours of symptoms.
b. Age older than 5 years.
c. Recurrence after first hydrostatic reduction.
d. Recurrent symptoms in the immediate postoperative period.
<C+>Pneumoperitoneum.

t. Abdominal X-Ray in a neonate on erect position show “double bubble sign” in all the following conditions except.
a. Duodenal stenosis.
<C+>Meconium ileus
b. Duodenal atresia
c. Normal newborn radiograph finding at delivery.
d. Mal-rotation of the mid gut.
u. A 10 months old infant was admitted to the hospital for lower gastrointestinal bleeding. The probable admitting
diagnosis was complicated Meckel’s diverticulum. Which of the following is FALSE?
a. Most of the lesions are detected at laparotomy.
<C+>It is typically found on the mesenteric side of the bowel.
b. It results from incomplete closure of the omphalomesenteric duct.
c. Gastric mucosa within the diverticulum is indirectly responsible for the gastrointestinal bleeding.
d. It occurs in 2% of the population.

v. Polyhydramnios is frequently observed in all of the following conditions except:


a. Esophageal atresia.
b. Duodenal atresia.
c. Pyloric atresia.
<C+>Hirschsprung's disease.
d. Congenital diaphragmatic hernia.

w. Branchial cleft remnants most often present with which of the following clinical problems?
<C+>Infection
a. Airway obstruction
b. Hemorrhage
c. Malignant degeneration
d. Lymphedema

x. A 50 y old man comes to the hospital with severe bleeding and bruising. He is on warfarin and his INR is 11. For
rapid and complete reversal of his INR which one of the following is the best?
a. Factor X
b. Fresh frozen plasma plus vitamin K
c. Intravenous vitamin K
d. Protamine sulfate
<C+>Prothrombin complex concentrate plus vitamin K

y. All the followings are possible complications after parenteral nutrition for severely malnourished patients except:
a. Hyperglycemia
b. Hypomagnesemia
c. Hypokalemia
<C+>Hyperphosphatemia
d. Line infection

z. Which of the following anatomic locations of adenocarcinoma has the best prognosis after successful surgical
resection?
a. Pancreatic head
b. Pancreatic tail
c. Gallbladder
d. A common biliary tract
<C+>Ampula of Vater

aa. The most common liver disease related to oral contraceptive pills and estrogens is:
a. Hepatic abscess
b. Suppurative cholangitis
<C+>Cholestatic jaundice
c. Cirrhosis
d. Hepatocellular carcinoma

bb. Themost important prognostic factor for carcinoma of the esophagus is:
a. Cellular differentiation
<C+>Depth of involvement
b. Length of involvement
c. Age of the patient
d. Site of esophagus affected

cc. Regarding blind loop syndrome, all the followingsare true except:
a. Usually it manifests as diarrhea, weight loss, and deficiency of fat soluble vitamins.
b. Megaloblastic anemia is commonly seen
<C+>Surgery is almost always required to correct the syndrome
c. Broad spectrum antibiotics are the treatment of choice
d. 14C-xylose or 14C-cholylglycine breath tests are helpful in the diagnosis

dd. All the following statements regarding strangulatedventral hernias are true except:
a. Strangulation is associated with localized tenderness.
<C+>Attempt at reduction of astrangulated inguinal hernia is generally considered safe
b. A strangulated epigastric hernia usually contains unhealthy pre-peritoneal fat
c. Strangulated hernias are notexpansile on coughing.
d. Femoral hernias carry a higher chance of strangulation.

ee. All the following statements concerning intraperitoneal fluid collections are correct except:
<C+>Blood is very irritant to the parietal peritoneum
a. Ascites occurs when either the peritoneal fluid secretion rate increases or the absorption rate decreases.
b. Accumulation of lymph within the peritoneal cavity usually results from trauma or tumor involving the intra-
abdominal lymphatic structures.
c. Choleperitoneum (intraperitoneal bile) generally occurs following biliary surgery, but spontaneous perforation of the
bile duct has been reported.
d. The most common cause of hemoperitoneum is trauma to the liver or spleen.

ff. All the following statements about bariatric surgery are true except:
<C+>Body Mass index (BMI) = weight (kg) / [height (cm)]2
a. A person is considered obese when BMI is 30 or more.
b. Polycystic Ovary Syndrome, Cushing’s syndrome and Hypothyroidism may cause obesity.
c. Adipocyte (fat cell) is considered an endocrine cell.
d. Complications of obesity include diabetes, high blood pressure and cancers.

gg. All the following statements about about Meckel's diverticulum are true except:
a. Meckel's diverticulum usually arises from the ileum within 90 cm of the ileocecal valve.
b. Meckel's diverticulum results from the failure of the vitelline duct to obliterate.
c. The incidence of Meckel's diverticulum in the general population is 2%.
<C+>Meckel's diverticulum is a false diverticulum lacking the muscular layer of the intestinal wall.
d. Gastric mucosa is the most common ectopic tissue found within a Meckel's diverticulum.

hh. All the following statements about the small intestine tumors are true except:
a. The most common benign tumor of the small intestine is leiomyoma.
<C+>The most common cancer of the small intestine is carcinoid tumor.
b. People with familial adenomatous polyposis (FAP) have an increased risk of duodenal cancer.
c. Pancreaticoduodenectomy is required for tumors involving the first and second portions of the duodenum.
d. Carcinoid tumors are treated by resection even in the presence of metastases

ii. All the following statements about obesity management are correct except:
a. Obesity treatment options include exercise plans and eating habit changes.
b. Obesity treatment options include drugs to prevent the absorption of fat or suppress appetite.
<C+>Bariatric surgery is usually indicated when BMI is 30 or more.
c. The most common type of Bariatric surgery in Jordan is laparoscopic gastric sleeve.
d. Diabetes and high blood pressure may improve after bariatric surgery.

jj. In general, shock can best be defined as:


a. Hypotension.
<C+>The syndrome of tissue hypoperfusion.
b. Hypoxemia.
c. Chronic drop in hemoglobin level.
d. Heart rate less than 60/minute.

kk. All the following statements regarding hypercalcemia are trueexcept?


a. The symptoms may include constipation and abdominal pain.
b. Metastatic breast cancer is a recognized cause in females.
<C+>Parathyroid adenomas cause secondary hypercalcemia.
c. May occur in patients with sarcoidosis.
d. Is initially treated by giving high intravenous fluid volume.

ll. All the following statements about the pathophysiology of Ebstein's anomaly are true except:
a. The tricuspid valve is usually insufficient.
b. Typically there is right-to-left shunt across the ASD.
c. The redundant anterior leaflet of the tricuspid valve may cause obstruction of the right ventricular outflow tract.
<C+>Pulmonary hypertension is a common late complication.
d. High pulmonary vascular resistance in neonates exacerbates tricuspid regurgitation and cyanosis.

mm. The evaluation of hemostasis and coagulation of a patient scheduled for elective surgery is best achieved by:
<C+>History and physical examination.
a. Complete blood count (CBC), including platelet count.
b. Prothrombin time (PT) and activated partial thromboplastin time (APTT).
c. Studies of platelet aggregation with adenosine diphosphate (ADP) and epinephrine.
d. Repeated INR testing.

nn. 78.The most commonly transmitted infectious agent via blood transfusion is:
a. Human immunodeficiency virus (HIV).
b. Candida albicans.
<C+>Cytomegalovirus (CMV).
c. Hepatitis B virus.
d. Staphylococcus aureas.

oo. Which of the following statements regarding splenic function in humans is true?
a. The spleen is the principal source for platelets.
b. The spleen is the major site of synthesis of complement pathway proteins.
c. Splenic contraction is a major mechanism to maintain blood pressure in humans suffering from acute bleeding.
<C+>The spleen serves as a principal source of nonspecific opsonins.
d. The spleen is the principal source of vasoactive intestinal peptides (VIPs).

pp. An otherwise healthy 50 year old lady presented with a large multinodular goiter causing tracheal compression. The
preferred management is:
a. Iodine treatment.
b. Thyroid hormone treatment.
<C+>Surgical resection of the abnormal thyroid.
c. Radioactive iodine treatment.
d. External beam radiotherapy.

qq. One of the following statements about follicular carcinomas is true:


a. The highest incidence is among teenagers.
<C+>It disseminates via hematogenous routes.
b. Follicular thyroid carcinoma is the most common type of well-differentiated thyroid carcinoma.
c. Extensive angioinvasionis associated with a better prognosis.
d. Follicular carcinomas are frequently multicentric.
rr. Hyperthyroidism can be caused by all of the followings except:
a. Graves' disease.
b. Plummer's disease (toxic adenoma).
c. Struma ovarii.
d. Hashimoto's disease.
<C+>Medullary carcinoma of the thyroid.

ss. All the following clinical circumstances have been identified as predisposing factors for the development of gastric
stress ulceration except:
a. Intraperitoneal sepsis
b. Hemorrhagic shock
<C+>Isolated tibial fracture
c. 50% total surface area second degree burn
d. Adult respiratory distress syndrome

tt. A 45-year-old male patient presents with symptoms of epigastric pain, worsened with ingestion of food. Physical
examination is normal. Upper abdominal ultrasonography is unremarkable. Contrast radiography reveals a 2 cm ulcer in
the gastric fundus along the lesser curvature. Therapy with omeprazole 20 mg per day is begun but symptoms persist.
The most appropriate management will be:
a. Increase in omeprazole dose to 40 mg per day.
b. Advice the patient to avoid alcohol ingestion.
c. Addition of cimetidine 200 mg b.i.d.
<C+>Esophagogastroduodenoscopy with biopsy of ulceration.
d. Anti H.pylori treatment.

uu. All the followings are bacterial infections except:


a. Actinomycosisinfection of appendectomy wound.
<C+>Mycetoma of the foot.
b. Erysipelas of the skin
c. Hidradenitis suppurativa of the pubic region
d. Fournier’ gangrene of the Scrotum.

vv. One liter of normal saline contains:


a. 20.0 mill–equivalent of CIˉ
<C+>154 mill - equivalent of CIˉ
b. 308 mill - equivalent of CIˉ
c. 412mill - equivalent of CIˉ
d. 550 mill - equivalent of CIˉ

ww. The most common type of bladder cancer is


<C+>Transitional cell carcinoma
a. Adenocarcinoma
b. Squamous cell carcinoma
c. Clear cell carcinoma
d. Chromophobe cell cancer

xx. The most common hernia in females is one of the followings:


a. Femoral hernia.
<C+>Inguinal hernia.
b. Obturator hernia
c. Epigastric hernia
d. Umbilical hernia

yy. Regarding acute intestinal obstruction all the following statements are correct Except:
<C+>Post-operative intestinal adhesion is a rare cause.
a. Incarcerated inguinal hernia is a possible cause.
b. Multiple air fluid levels are seen on an erect abdominal X-ray.
c. Electrolyte and fluid balance correction are of at most importance.
d. The lower the site of obstruction the more is the abdominal distension.

zz. Pancreatic cancer was found to be associated with all the followings except:
a. Smoking tobacco
<C+>Achlorhydria
b. Diabetes mellitus
c. Multiple endocrine neoplasia type 1 MEN1
d. Alcohol consumption

aaa. Regarding anal fissure, one is true:


a. It involves the whole length of the anal canal.
b. Usually associated with diarrhea.
c. The main complaint is anal discharge.
<C+>Usually located in the midline of the anal canal.
d. Medical treatment is usually not effective in the acute phase.

bbb. Regarding anal suppuration, one is true:


<C+>Starts by infection of the anal glands.
a. Abscesses are always clearly seen in the perianal region.
b. Treatment is usually by antibiotics.
c. Association with perianal fistula is rare (less than 5%).
d. Fever is unlikely

ccc. Worldwide, the most common cause of lymphedema is:


a. Cat-scratch disease
b. Preexisting CVI (chronic venous insufficiency )
<C+>Filariasis
c. Milroy disease
d. Atherosclerosis

ddd. A patient with head injury opens his eyes and withdraws his arm to pain. He is making incomprehensible sounds. His
Glasgow Coma Scale score is 8 (bonus question):
a. 12
b. 10
<C+>8
c. 6
d. 4

eee. The most common malignant tumor of the parotid gland is:
a. Adenocarcinoma
b. Adenoid cystic carcinoma
<C+>Mucoepidermoid carcinoma
c. Acinic cell carcinoma
d. Anaplastic carcinoma

fff. All the following statements regarding fat embolism are correct except:
a. It is relatively common but only rarely causes symptoms.
<C+>Neurological dysfunction is not a feature of the disease.
b. Petechial hemorrhages are usually seen in and around both axilla.
c. Fat droplets in sputum and urine are helpful in diagnosis.
d. For well-established cases, positive pressure ventilation is an essential line of treatment.

ggg. All the following statements regarding the vermiform appendix are true except:
<C+>Embryologically the appendix is derived from the hind gut.
a. A yellowish mass at the tip of the appendix during appendicectomy raises the possibility of carcinoid tumor of the
appendix.
b. The commonest complication after appendicectomy is wound infection.
c. Every removed appendix should be sent to the pathology department regardless of the gross diagnosis by the surgeon.
d. In a 75 year old patient presenting with signs and symptoms of acute appendicitis, the possibility of an underlying
cecal tumor should be kept in mind.

hhh. All the following statements about cellulitis are true except:
a. The commonest involved organism is Streptococcus.
b. Penicillin is the treatment of choice for community acquired cellulitis.
<C+>Surgery is the first line of treatment.
c. In erysipelas the patients are very toxic with more marked skin inflammation.
d. Effective pain control is an essential part of the management.

iii. All the following pathologies might mimic the presentation of acute appendicitis except:
a. Meckel’s diverticulitis.
b. Mesnteric adenitis.
c. Perforated duodenal ulcer.
<C+>Biliary colic.
d. Perforetdcecal carcinoma.

jjj. A 20 year old male patient presents with history of pain in his right submandibular area experienced during eating. He
recalls the occurrence of a transient swelling in the same area associated with the pain. Intra-oral examination of this
patient might reveal:
a. The presence of a ranula
b. The presence of ectopic thyroid tissue at the base of the tongue.
c. An ulcerating carcinoma on right edge of the tongue.
d. Kissing tonsils
<C+>The feeling of a small stone along the course of the right submandibular gland duct.
<Q> An elderly diabetic patient presents with gangrene of the right 2nd toe. Clinically she has palpable femoral pulses
and popliteal pulses but no pulses palpable distal to this. The 2nd toe requires amputation. Which test would you use to
determine whether the perfusion is adequate for healing of the amputation site?
<S>Y
<C> Resting ABPI
<C> B.Exercise ABPI
<C+>Toe brachial pressure indices
<C> Spectral waveforms
<C> E.Pulse volume recording

<Q> An alert patient with a suspected ruptured abdominal aortic aneurysm is taken to theatre. Which statement
concerning a surgical safety check (a pause) is NOT correct?
<C> A pause is feasible prior to an emergency operation
<C> A pause should involve anaesthetist, surgeon and scrub nurse
<C+>A pause should include a review of all regular medication
<C> A pause should include the availability of blood for transfusion
<C> A pause should include the availability of necessary vascular instruments

<Q> Which of the following agents is not considered an adjunct to control major haemorrhage induced by a vascular
disease process, trauma or surgery?
<C> Fresh frozen plasma
<C> Prothrombin complex concentrate
<C> Recombinant activated factor VII
<C+>Tissue plasminogen activator
<C> Tranexamic acid

<Q> A 76 year old patient is admitted for open aortic aneurysm repair. He has suffered a previous deep venous
thrombosis. Which of the following is an appropriate single method of perioperative thromboprophylaxis?
<C> Warfarin
<C+>Factor Xa inhibitors
<C> Early mobilisation
<C> Intraoperative pneumatic calf compression
<C> Dextran

<Q> Mycotic aneurysms are rarely seen in patients:


<C> Aged over 80 years
<C+>With normal inflammatory markers
<C> With an aneurysm greater than 5cm maximum diameter
<C> Among smokers
<C> Patients on steroids

<Q> Which of the following is the most effective oral treatment for improving the walking distance in patients with
intermittent claudication and heart failure?
<C> Gingko biloba
<C+>Pentoxyfylline
<C> Iloprost
<C> Cilostazol
<C> Propionyl L-Carnitine

<Q> In the context of critical limb ischaemia, primary amputation should be considered in all of the following situations
except:
<C+>Significant necrosis of all 5 toes
<C> Significant necrosis of the weight bearing portions of the foot
<C> Paresis of the extremity
<C> Significant sepsis secondary to osteomyelitis of the calcaneus
<C> Uncorrectable flexion contracture of the knee
<Q> A 24 year-old man is brought into the Emergency department having been stabbed with a screwdriver. He is
conscious. On examination he is tachypnic and has a tachycardia of 120 beats/minute. His blood pressure is 90/50 mmHg.
He has a small puncture wound below his right costal margin. A central venous line is inserted with ease, and his central
venous pressure is 17 cm. A chest X-ray shows a small pleural effusion with a small pneumothorax. He has received two
units of plasma expander, which has failed to improve his blood pressure. The most likely diagnosis is:
<C> Acute coronary syndrome
<C> Pulmonary embolism
<C> Acute respiratory distress syndrome
<C+>Cardiac tamponade
<C> Aortic dissection

<Q> A 42 year old diabetic lady presents with digital micro-infarcts of her toes .She gives a history of colour change
affecting her digits in cold weather. She has a full complement of arterial pulse and no cardiac murmurs. There are no
other positive findings on vascular examination. She has raised inflammatory markers and positive anti-neutrophil
cytoplasmic antibodies. The most likely diagnosis is:
<C> Burger’s disease
<C+>Systemic sclerosis
<C> Thromboembolism
<C> Diabetic foot disease
<C> E.Neuropathy

<Q> A 64-year-old non-smoker complains of severe foot pain during the night .It awakes him from sleep. He has recently
begun to dangle the leg off the end of the bed, as this seems to alleviate some of the pain. In this patient, which of the
following is true?
<C+>Rest pain is an indication for surgical revascularization
<C> Pedal ulcers are likely to heal quickly due to the robust granulation tissue that forms in those with ischemic rest pain
<C> These days, prior to surgical intervention, most patients are routinely sent for medical optimization despite no
observed improvement in perioperative morbidity and mortality
<C> A percutaneous intervention at the level of the common iliac artery is less likely to last 5 years than one at the
common femoral artery
<C> PTA is ideal for opening up long segments of stenosis

<Q> Which of the followings is true about arterial trauma?


<C> All arterial injuries are associated with pulsatile bleeding
<C> The commonest cause is penetrating injury
<C> The distal pulses will always be absent
<C> There is no relationship to major joint dislocations
<C+>Bleeding is more likely with partial than with complete arterial transection

<Q> A 32-year-old male involved in a head-on motor vehicle collision presents with chest pain and the following vital
signs on arrival in the emergency department: Heart rate – 120/minute; Blood pressure – 86/50; GCS score – 10; and O2
saturations of 92%. Neck veins are distended and breath sounds are absent on the right side. Which of the following takes
first priority?
<C> Urgent CT scan of the head to rule out an extradural hemorrhage with midline shift
<C> Rapid resuscitation with 2 large-bore intravenous cannula and warmed fluids
<C> ECG and an echocardiogram to eliminate cardiac contusion as the cause of his hypotension
<C> Obtaining an urgent crossmatch
<C+>Elimination of tension pneumothorax as a cause of his symptoms/signs

<Q> Hemorrhage initiates a series of compensatory responses. All of the followings statements are true concerning the
physiologic responses to hemorrhagic shock EXCEPT?
<C> An immediate response is an increased sympathetic discharge with resultant reflex tachycardia and vasoconstriction
<C> Transcapillary refill is a response serving to restore circulating volume
<C> Extracellular fluid becomes increasingly hyperosmolar
<C+>An immediate rise in the level of thyroxine hormone is the most recognized response
<C> An increased release of stress hormones coupled with relative insulin resistance

<Q> A gunshot wound transecting the superficial femoral artery:


<C> Should be repaired with a PTEE graft as the fist option
<C> Should be repaired with ipsilateral superficial femoral vein graft
<C> Should be ligated primarily to prevent disruption from infection
<C+>Should be repaired with a vein graft from the contralateral leg
<C> Should never be treated without preoperative angiography in the radiology suite

<Q> An otherwise fit 57-year-old man spikes a temperature of 39◦C five days after appendicectomy. There is a tender,
reddened and fluctuant swelling at the medial end of the wound. What is the most appropriate initial action to take?
<C> Arrange a CT scan of the abdomen
<C> Arrange an ultrasound scan of the wound and anterior abdominal wall
<C> Start the patient on oral antibiotics
<C+>Open the wound to allow free drainage
<C> Send off blood samples for a white cell count and culture

<Q> 17.The following statements on acute lymphangitis of the lower limb are correct EXCEPT:
<C> Improperly managed, it may lead to lymphadenitis
<C+>Lymphangiography is the investigation of choice in the management
<C> Rest and elevation of the affected limb is appropriate
<C> Cellulitis may be the initiating cause
<C> Appropriate antibiotics should include cover for streptococcal infection

<Q> An exploratory laparotomy is done for an un-localized massive hematochezia. At surgery a large right-sided
diverticulum is seen with more blood pooling in the right colon than the left colon. The procedure of choice is:
<C> Right hemicolectomy
<C> Right hemicolectomy with intra-op small bowel enteroscopy
<C> Inferior mesenteric artery ligation and observation
<C+>Total abdominal colectomy and ileo-rectal anastomosis
<C> Intraoperative angiogram to localize the bleed

<Q> A 68-year-old man presents for a routine physical examination. The patient is found to have guaiac-positive stools.
Colonoscopy reveals a “near-obstructing tumor in the descending colon, not admitting the scope. The biopsy is positive
for adenocarcinoma of the colon. Which of the following is the next step in the management of this patient?
<C> Full metastatic workup first, and if negative, plan for colon resection
<C> A course of radiation therapy prior to any resection
<C> Pre-operative chemotherapy
<C+>Metastatic work up, but plan for colon resection anyway
<C> Barium enema to evaluate the proximal colon

<Q> A 78-year-old woman with coronary artery disease and severe chronic obstructive pulmonary disease is admitted to
the hospital with large bowel obstruction. CT scan revealed the presence of multiple lesions in the liver, suggestive of
metastases, and a nearly obstructing upper rectal mass. Colonoscopy demonstrates a large, ulcerated obstructing tumor in
the proximal rectum. What is the best initial treatment modality for this patient?
<C> Radiation therapy of the rectal mass
<C> Placement of a colonic decompression tube
<C> Emergency surgery with resection of the mass
<C> Emergency surgery with creation of a diverting colostomy
<C+>Placement of a rectal self-expanding metal stent

<Q> A 57-year-old man is found to have a rectal adenocarcinoma 3 cm from the anal verge. EUS examination
demonstrated penetration of the tumor into, but not through, the muscularispropria, but shows significant perirectal lymph
nodes. CT scan of chest/abdomen/pelvis demonstrated no metastases. The patient is staged as T2N1M0. The treatment of
choice is:
<C> Endoscopic mucosal resection (EMR)
<C> Endoscopic argon plasma coagulation
<C> Surgical transanal excision
<C> Neo-adjuvant chemoradiation followed by transanal excision
<C+>Neo-adjuvant chemoradiation followed by abdominoperineal resection (APR)

<Q> 72 year old woman presents to the ER with left lower quadrant abdominal pain and tenderness, fever and
leukocytosis. A CT scan suggests a diagnosis of diverticulitis with a pelvic abscess 5 cm in diameter. The most
appropriate treatment at this time is:
<C> Broad spectrum IV antibiotics
<C+>Percutaneous drainage of the pelvic abscess
<C> Laparotomy with drainage of the abscess
<C> Laparotomy, drainage of the abscess, sigmoid resection and colorectal anastomosis
<C> Laparotomy, drainage of the abscess, sigmoid resection and end colostomy (Hartmann’s procedure)

<Q> An 80 year old lady developed sudden severe abdominal distention associated with colicky pain and nausea. She has
not had passed stool or gas for the past 24 hours. Abdomen is tympanic with hyperactive bowel sounds. Abdominal XR
showed a very large gas shadow tapering towards the pelvis and distended loops of small & large bowel. A diagnosis of
sigmoid volvulus was made. What is the best treatment option?
<C> IV fluids and observation
<C+>Colonoscopic deflation
<C> Sigmoid resection and primary anastomosis
<C> Hartman's procedure
<C> Colostomy

<Q> A 42 year old man has suffered from chronic ulcerative colitis for 15 years. He had multiple admissions for
exacerbations of the disease, requiring high dose steroids. He is now on a low dose of steroids, sulfasalazine and immuran.
He is admitted with a temperature of 39, abdominal pain and distention. He appears ill, his abdomen is markedly tender &
his WBC count is 18,000. What is the most likely diagnosis?
<C+>Toxic megacolon
<C> Exacerbations of the disease
<C> Pseudo-obstruction
<C> Obstruction by tumor
<C> Membrenous enterocolitis

<Q> A 65-year-old woman with no significant past medical history, resented to the ER with a 2-day history of left lower
quadrant abdominal pain. The patient denies nausea and vomiting. She describes a milder episode several years ago,
which resolved on its own. Physical examination revealed left lower quadrant tenderness with mild guarding. She is
hemodynamically stable. In the initial management, which of the following is the most sensitive diagnostic test?
<C> Complete blood count and electrolytes
<C> Plain abdominal X-Ray erect and supine
<C> A barium enema study
<C+>Abdominal/pelvic CT with oral contrast
<C> Abdominal ultrasound

<Q> The prognosis of a resected thymoma is affected by


<C> Identification of calcification.
<C> Percentage of lymphocytic infiltration.
<C> Presence of antibodies to cytokeratin.
<C> Presence of myasthenia gravis.
<C+>Size of the tumor.

<Q> Each of the following is associated with an increased risk for carcinoma of the esophagus EXCEPT
<C> Achalasia
<C> Barrett's esophagus.
<C> Exposure to N-nitrosmines.
<C+>Peptic stricture.
<C> Tylosis.

<Q> Each of the following posterior mediastinal tumors is found predominantly in adults EXCEPT
<C+>Ganglioneuroma
<C> Malignant schwannoma.
<C> Malignant paraganglionoma.
<C> Neurilemoma.
<C> Neurofibroma.

<Q> A 27-year-old patient involved in a motor vehicle accident is brought to the emergency room alert, cooperative, and
hemodynamically stable. The patient's respiratory rate is 24/minute and the arterial blood gas shows pH 7.49, PO2 95
mmHg, PCO2 30 mmHg. The chest radiograph obtained in the emergency room shows pneumomediastinum with no
peneumothorax. The initial management of this injury should include:
<C+>Bronchoscopy.
<C> Intubation.
<C> Observation.
<C> Thoracoscopy.
<C> Tube thoracostomy.

<Q> The single factor most predictive of survival in patients with a completely resected stage II non-small cell lung
cancer is
<C> Adjuvant therapy.
<C+>Number of metastatic N1 lymph nodes.
<C> Postoperative radiation therapy.
<C> T status ( T1 versus T2 )
<C> Visceral pleural invasion.

<Q> When considering a patient for resection of pulmonary metastases, all of these criteria should be met except:
<C> Control of the primary site
<C> No other distant extra-pulmonary metastatic disease
<C> Pulmonary metastases that are thought to be completely resectable, even if located in both the lungs
<C> Adequate cardiopulmonary reserve of the patient
<C+>Doubling time of the tumor more than 6 months

<Q> Which of the following statements regarding injury to the diaphragm is true
<C> Blunt injury to the diaphragm is more likely to involve the right side.
<C+>Delayed recognition of diaphragmatic injury is more common after penetrating trauma than blunt injury.
<C> The most common finding on the standard x-ray after penetrating injury is elevation of the affected diaphragm.
<C> Repair of acute diaphragm injuries is better handled using a transthoracic approach.
<C> Repair of chronic diaphragm injuries (greater than three months) is best-accomplished transabdominally.

<Q> 33. All the following statements about hydatid disease are correct except:
<C+>Humans get the infection by ingesting the ova secreted in the stool of sheep
<C> The liver is the commonest site of cyst formation in humans
<C> Intrabiliary cyst rupture may present as cholangitis
<C> Rupture of the cyst may be associated with anaphylactic reactions
<C> Medical treatment by albendazol is occasionally added to surgical treatment.

<Q> All are true regarding appendicitis except:


<C> The initial peri-umbilical pain is a mid-gut visceral pain caused by distension of the lumen of the appendix.
<C> The right iliac fossa pain is a somatic pain due to irritation of the parietal peritoneum.
<C> Pelvic ultrasound in appendicitis is a useful diagnostic tool especially in females
<C+>CT scan of the abdomen is routinely used to confirm the diagnosis of appendicitis because of its high sensitively and
specificity.
<C> Laparoscopic approach for appendectomy has advantages over open approach especially in females and obese
patients.
<Q> Which of the following may be an indication for operation for mitral stenosis?
<C> Symptomatic patient with mitral valve area less than 1.5 m2
<C> Mitral valve area less than 1 m2
<C> Embolisation
<C> Pulmonary hypertension
<C+>Decreased transvalvular gradient on exercise echocardiography

<Q> Which of the following are relative indications for mechanical, as opposed to tissue, valve replacement?
<C+>Patient younger than 30 years.
<C> Young female patient who desires children.
<C> An elderly patient.
<C> Tricuspid valve replacement.
<C> Chronic renal failure

<Q> Pancreatic cancer was found to be associated with all the followings except:
<C> Smoking tobacco
<C+>Achlorhydria
<C> Diabetes mellitus
<C> Multiple endocrine neoplasia type 1 MEN1
<C> Alcohol consumption

<Q> All the followings are true about acute pancreatitis except:
<C> Chronic alcohol ingestion is a common cause in the west
<C> Vomiting and belching are recognized symptoms
<C+>Pain is increased by leaning forward
<C> Amylase level is the most important prognostic sign
<C> Hemorrhagic pncreatitis is associated with a high mortality.

<Q> A 50 year old man presented with early satiety and weight loss. Physical examination revealed an enlarged hard left
supra-clavicular lymph node. He is a heavy smoker. The most likely diagnosis is:
<C> Sigmoid carcinoma
<C> Lung carcinoma
<C> Oesophageal carcinoma
<C+>Gastric carcinoma
<C> Gall bladder carcinoma

<Q> A 60 year old man presented with gradually enlarging swelling over his left parotid area for the last 6 months.
During the last few weeks the mass became painful with skin reddening above it. The patient noticed drooping of the left
side of his lips. The most likely diagnosis is:
<C> Suppurativeparotitis
<C+>Adenoid cystic carcinoma
<C> Parotid stone
<C> Pleomorphic adenoma
<C> Tuberculosis

<Q> When closing laparotomy wounds, all the followings are recommended to decrease the risk of developing incisional
hernia EXCEPT:
<C> Continuous suturing technique.
<C> Avoiding tension
<C> Use of slowly absorbable monofilament suture.
<C> For high risk patients (e.g. aortic aneurysm surgery and obese patients), prophylactic mesh augmentation is
suggested.
<C+>Choosing a vertical laparotomy wound rather than a transverse one.

<Q> Regarding venous thromboembolism (VTE), all are true EXCEPT:


<C> Risk of VTE is higher in pelvic surgery.
<C> Patients with varicose veins and phlebitis are at increased risk of VTE.
<C> Intermittent pneumatic compression devices are effective mechanical prophylaxis methods.
<C+>Antiplatelet agents are the most effective drugs for VTE prophylaxis.
<C> Inferior vena caval filters are sound prophylactic measures against VTEwhen mechanical and pharmacological
prophylaxis are contraindicated.

Tumor grading is of great significance in predicting the prognosis and guiding the treatment of all the following
tumors except:
a. Soft tissue sarcoma.
b. Astrocytoma
c. Transitional cell carcinoma.
d. Breast carcinoma*
e. Non-Hodgkin lymphoma.

All the followings are sound examples of prophylactic organ surgical excision except:
f. Ulcerative Colitis with premalignant features
b. APC mutation
c. (RET-mutation positive relatives )of patients with RET mutation in MEN 2a and MEN 2 b medullary thyroid
carcinomas
atives of a HER 2 nu positive breast carcinoma patients
a. Heriditary nonpolyposis colorectal cancer ( HNPCC) germ line mutations

d. All the following data can be accurately obtained by a pathologist evaluating a core needle biopsy of a malignant breast
mass except:
a. The exact histologic subtype of the tumor
<C+>The T Sage of the primary tumor
b. The HER 2 mutation status
c. The presence of vascular invasion
d. The estrogen receptor status

e. All the following statements about malignant phylloides tumors are correct except:
<C+>Axillary lymph node metastasis occurs early in the course of the disease
a. Chemo-radiotherapy has a limited role in the treatment
b. Mammographic findings are not specific
c. True cut biopsy is diagnostic
d. Huge tumors can cause pressure ulceration of the skin of the breast

f. All the following statements about HER 2 neu positive breast cancer are true except:
a. They constitute about 20-25 % of breast cancers
b. Herceptin can be useful in the treatment
c. HER 2 positive tumors have a worse prognosis compared to HER 2 negative tumors
d. FISH testing is indicated if the immunohistochemistry score is + 2
<C+>HER 2 positivity is a contraindication to breast conservation surgery

g. The most common arrhythmia created by laparoscopy is :


<C+>Bradycardia
a. Atrial fibrillation
b. Supra-ventricular tachycardia
c. Ventricular fibrillation
d. Pulsusalterance

h. During CO2 pneumo-peritoneum for a routine laparoscopic cholecystectomy potential respiratory acidosis is largely
buffered by:
a. Kidneys
b. Lungs
<C+>Bone
c. Increasing sweat production
d. Serum albumin

i. All the followings are sound anti-estrogen treatments in estrogen positive breast cancer except:
a. Oophorectomy in premenopausal patients
<C+>Specific aromatase inhibitors in premenopausal patients
b. Tamoxifen in post menopausal patients
c. Goserline ( LHRH agonist) in premenopausal patients
d. Estrogen receptor downregulators in postmenppausal patients

j. Adjuvant chemotherapy is given to some patients with breast cancer .Which of the followings represent these patients:
<C+>Patients who had surgery and no evidence of systemic metastasis
a. Patients who are above the age of 70 years at the time of diagnosis
b. Patients who are not willing to have mastectomy
c. Patients who are not willing to receive radiotherapy
d. Patients who are not willing to have breast reconstruction after mastectomy

k. A 50 year-old woman is having infiltrating ductal carcinoma of her left breast. The size of the primary tumor is 1.5 cm.
The sentinel lymph node was free of malignancy. She has no distant metastasis. The TNM sage will be:
a. Stage X (cannot be assessed)
<C+>Stage I
b. Stage II
c. Stage III
d. Stage IV

l. A 45 year old lady presented with a 3×3 right sided breast mass. True cut biopsy revealed infiltrating ductal carcinoma
with HER II over expression. HER II overexpression in this lady denotes:
a. a better prognosis
<C+>The need to add trustuzumab (Herceptin) to her treatment
b. Resistance to chemotherapy
c. The need to perform radical mastectomy
d. Autosomal dominant inheritance of her cancer

m. The severity of inhalational burn injury is estimated By:


a. Blood gas analysis.
b. Chest x-ray.
c. Oxymetry.
<C+>Carboxy hemoglobin level.
d. Indirect laryngoscopy.

n. The most important treatment strategy in chemical burn is:


<C+>Copious irrigation of the area with water.
a. Removal of dry irritant chemicals.
b. Identifying the irritant to use the suitable antidote.
c. Removal of contaminated clothes.
d. Recognition and treatment of systemic toxicity.

o. The commonest cause of a clinically solitary thyroid nodule is:


a. Physiological goiter.
b. Hashimoto’s thyroiditis.
<C+>Multinodular goiter.
c. Papillary adenoma.
d. Medullary carcinoma

p. Regarding necrotizing fasciitis the followings are true Except:


a. It usually has a short incubation period to show.
b. The patient usually shows toxemia symptoms.
c. The spread of infection and necrosis is faster along the subcutaneous tissue planes than the in the skin.
<C+>The causative organism is usually single gram negative bacteria.
d. It affects mostly patients with decreased immunity

q. The best approach to diagnose splenic injury is:


a. Observing enlarging abdominal girth.
b. Repeating hematocrit evaluation.
c. Performing peritoneal lavage.
d. Demonstrating fractured left lower ribs by chest x-ray.
<C+>Performing abdominal CT scan.

r. In intestinal obstruction, plain abdominal X-ray is least accurate in showing:


a. Air fluid levels.
b. Level of the obstruction.
<C+>Strangulated intestinal loop.
c. Closed loop obstruction.
d. Pseudo obstruction.

s. When observing a patient with simple adhesive intestinal obstruction; the most important to decide on operative
intervention is:
a. Vomiting feculant material.
b. Increase hematocrit level.
c. Leucocytosis.
d. Persistence of the symptoms for more than 48 hours.
<C+>Localization of the abdominal signs.

t. Regarding branchial cyst; the followings are true Except:


a. It usually shows positive fluctuation.
<C+>It has positive trans- illumination to light.
b. It represents remnant of the second branchial cleft.
c. Its fluid contains cholesterol crystals.
d. Its tract reaches the lateral pharyngeal wall

u. In a 2 months old infant, which of the following is the indicated management for a-non-cummunicating hydrocele?
<C+>Observation.
a. Needle aspiration.
b. Hydrocelectomy through a groin incision.
c. Hydrocelectomy through a scrotal incision.
d. Repair of the hernia and hydrocelectomy.

v. The most common malignancy found in children is.


a. Lymphoma.
<C+>Leukemia
b. Neuroblastoma.
c. Nephroblastoma.
d. Rhabdomyosarcoma.

w. The most common cause of duodenal obstruction at birth is:


a. Malrotation.
<C+>Duodenal atresia.
b. Annular pancreas.
c. Midgut volvulus.
d. Preduodenal portal vein.
x. In case of intussuception ,hydrostatic reduction is contra-indicated in one of the followings.
a. After 48 hours of symptoms.
b. Age older than 5 years.
c. Recurrence after first hydrostatic reduction.
d. Recurrent symptoms in the immediate postoperative period.
<C+>Pneumoperitoneum.

y. Abdominal X-Ray in a neonate on erect position show “double bubble sign” in all the following conditions except.
a. Duodenal stenosis.
<C+>Meconium ileus
b. Duodenal atresia
c. Normal newborn radiograph finding at delivery.
d. Mal-rotation of the mid gut.

z. Polyhydramnios is frequently observed in all of the following conditions except:


a. Esophageal atresia.
b. Duodenal atresia.
c. Pyloric atresia.
<C+>Hirschsprung's disease.
d. Congenital diaphragmatic hernia.

aa. The treatment of choice for neonates with uncomplicated meconium ileus is:
a. Observation.
b. Emergency laparotomy, bowel resection, and Bishop-Koop enterostomy.
<C+>Intravenous hydration and a gastrograffin enema.
c. Emergency laparotomy, bowel resection, and anastomosis.
d. Sweat chloride test and pancreatic enzyme therapy.

bb. Branchial cleft remnants most often present with which of the following clinical problems?
<C+>Infection
a. Airway obstruction
b. Hemorrhage
c. Malignant degeneration
d. Lymphedema

cc. A patient was admitted to the cardiac surgery department after being diagnosed with atrial septal defect. This condition
is most likely due to incomplete closure of:
a. Ductus arteriosus
b. Coronary sinus
c. Ductus venosus
d. Sinus venarum
<C+>Foramen ovale

In Laparoscopic Nissenfundoplication , prior to dividing the pars falccida , what aberrant structure may be encountered
a. An accessory left hepatic artery from left gastric artery *
b. A replaced right hepatic artery arising from the superior mesenteric artery
c. An accessory right hepatic artery originating from celiac trunk
d. A replaced left hepatic artery arising from superior mesenteric artery
e. An accessory right hepatic artery originating from left gastric artery

f. The most common cause of benign biliary stricture is :


a. Chronic pancreatitis
b. Primary sclerosing cholangitis
c. Portal hypertention
<C+>Previous cholecystectomy
d. Choledocholithiasis

g. Investigation of a 45 year old lady reveals the presence a large 3 cm stone in the neck of the gallbladder eroding into
the common bile duct effacing about 50 percent of its diameter. What would be the most appropriate surgery for this
patient?
a. Cholecystectomy , removal of the stone and primary repair of the CBD over a T tube
b. Radical cholecystectomy
c. Laparoscopic cholecystectomy with per operative choledochoscope
<C+>Cholecystectomy with Roux en Y hepaticojejunostomy
d. Cholecystectomy and use of a flap of gall bladder to repair the defect in CBD

h. A 50 year old male patient is having vague persistent abdominal pain. CT scan reveals duodenal diverticulum. Which
of the following statements regarding duodenal diverticulum is TRUE?
<C+>Surgery is reserved for complications
a. The majority of duodenal diverticulae are associated with lethal complications.
b. Most of the duodenal diverticulae are congenital
c. Perforation is the most common complication
d. Juxta vaterian diverticulum is the easiest to manage

i. Hemorrhage of over one liter will produce all of the following except:
<C+>Increase in renal blood flow
a. An increase in platelet count
b. A fall in arterial PH
c. A fall in body Temperature
d. Increased fibrinolysis

j. All the following statements regarding obstructive jaundice are true except:
a. Urinary conjugated bilirubin is incresed
b. Urinary urobilinogen is reduced
c. Serum conjugated bilirubin is increased
<C+>Oral Vitamin K supplements correct the bleeding tendency
d. Clay color stool is characteristic

k. Which of the following anatomic locations of adenocarcinoma has the best prognosis after successful surgical
resection?
a. Pancreatic head
b. Pancreatic tail
c. Gallbladder
d. A common biliary tract
<C+>Ampula of Vater

l. The most common liver disease related to oral contraceptive pills and estrogens is:
a. Hepatic abscess
b. Suppurative cholangitis
<C+>Cholestatic jaundice
c. Cirrhosis
d. Hepatocellular carcinoma

m. Themost important prognostic factor for carcinoma of the esophagus is:


a. Cellular differentiation
<C+>Depth of involvement
b. Length of involvement
c. Age of the patient
d. Site of esophagus affected

n. Regarding blind loop syndrome, all the following are true except:
a. Usually it manifests as diarrhea, weight loss, and deficiency of fat soluble vitamins.
b. Megaloblastic anemia is commonly seen
<C+>Surgery is almost always required to correct small bowel syndrome
c. Broad spectrum antibiotics are the treatment of choice
d. 14C-xylose or 14C-cholylglycine breath tests are helpful in the diagnosis

o. All the following statements about the repair of inguinal hernias are true except:
<C+>The conjoined tendon is sutured to Cooper's ligament in the Bassini hernia repair.
a. The McVay repair is a suitable option for the repair of femoral hernias.
b. The Shouldice repair involves a multilayer, imbricated repair of the floor of the inguinal canal.
c. The Lichtenstein repair is accomplished by prosthetic mesh repair of the inguinal canal floor in a tension-free manner.
d. The laparoscopic approaches to inguinal hernia repairs are now replacing the conventional open techniques

p. All the following statements concerning the abdominal wall layers are true except:
a. Scarpa's fascia affords little strength in wound closure, but its approximation contributes to the creation of an
aesthetically acceptable scar.
<C+>The internal inguinal ring is a congenital defect in the external oblique aponeurosis through which the testis
descends into the scrotum.
b. The transversalis fascia is the most important layer of the abdominal wall in preventing hernias.
c. The lymphatics of the abdominal wall drain into the ipsilateral axillary lymph nodes above the umbilicus and into the
ipsilateral superficial inguinal lymph nodes below the umbilicus.
d. The lineaalba extends all through between the xiphesternum and the symphysis pubis

q. All the following statements concerning intraperitoneal fluid collections are correct except:
<C+>Blood is very irritant to the parietal peritoneum
a. Ascites occurs when either the peritoneal fluid secretion rate increases or the absorption rate decreases.
b. Accumulation of lymph within the peritoneal cavity usually results from trauma or tumor involving the intra-
abdominal lymphatic structures.
c. Choleperitoneum (intraperitoneal bile) generally occurs following biliary surgery, but spontaneous perforation of the
bile duct has been reported.
d. The most common cause of hemoperitoneum is trauma to the liver or spleen.

r. All the following statements about bariatric surgery are true except:
<C+>Body Mass index (BMI) = weight (kg) / [height (cm)]2
a. A person is considered obese when BMI is 30 or more.
b. Polycystic Ovary Syndrome, Cushing’s syndrome and Hypothyroidism may cause obesity.
c. Adipocyte (fat cell) is considered an endocrine cell.
d. Complications of obesity include diabetes, high blood pressure and cancers.

s. All the following statements about about Meckel's diverticulum are true except:
a. Meckel's diverticulum usually arises from the ileum within 90 cm of the ileocecal valve.
b. Meckel's diverticulum results from the failure of the vitelline duct to obliterate.
<C+>The incidence of Meckel's diverticulum in the general population is 8%.
c. Meckel's diverticulum is a true diverticulum containing all the layers of the intestinal wall.
d. Gastric mucosa is the most common ectopic tissue found within a Meckel's diverticulum.

t. All the following statements about the small intestine tumors are true except:
a. The most common benign tumor of the small intestine is leiomyoma.
b. Adenocarcinoma is the most common cancer of the small intestine.
c. People with familial adenomatous polyposis (FAP) have an increased risk of duodenal cancer.
<C+>GIST tumors are thought to arise from the peyer’s patches.
d. Carcinoid tumors are treated by resection even in the presence of metastases.

u. All the following statements about management of obesity are correct except:
a. Obesity treatment options include exercise plans and eating habit changes.
b. Obesity treatment options include drugs to prevent the absorption of fat or suppress appetite.
<C+>Bariatric surgery is usually indicated when BMI is 30 or more.
c. The most common type of Bariatric surgery in Jordan is laparoscopic gastric sleeve.
d. Diabetes and high blood pressure may improve after bariatric surgery.

v. Which of the following statements are true of the multiple organ dysfunction syndrome (MODS)?
a. The “two-hit” model proposes that secondary MODS may be produced when a second insult reactivates the systemic
inflammatory response that was primed by an initial insult to the host.
b. The systemic inflammatory response syndrome (SIRS), shock due to sepsis or SIRS, and MODS may be regarded as a
continuum of illness severity.
<C+>Prolonged stimulation or activation of Kupffer cells in the lung is thought to be a critical factor in the sustained,
uncontrolled release of inflammatory mediators.
c. Despite advances in critical care, no significant improvement in outcome of MODS has been achieved.
d. E.Macrophages are known to play a critical role in the elaboration of numerous inflammatory mediators.

w. All the following statements regarding hypercalcemia are trueexcept?


a. The symptoms may include constipation and abdominal pain.
b. Metastatic breast cancer is a recognized cause in females.
c. Parathyroid adenomas cause primary hypercalcemia.
d. May occur in patients with sarcoidosis.
<C+>Is initially treated by limiting fluid intake.

x. All the following statements about the pathophysiology of Ebstein's anomaly are true except:
a. The tricuspid valve is usually insufficient.
b. Typically there is right-to-left shunt across the ASD.
c. The redundant anterior leaflet of the tricuspid valve may cause obstruction of the right ventricular outflow tract.
<C+>Pulmonary hypertension is a common late complication.
d. High pulmonary vascular resistance in neonates exacerbates tricuspid regurgitation and cyanosis.

y. The most commonly transmitted infectious agent via blood transfusion is:
a. Human immunodeficiency virus (HIV).
b. Candida albicans.
<C+>Cytomegalovirus (CMV).
c. Hepatitis B virus.
d. Staphylococcus aureas.

z. An otherwise healthy 50 year old lady presented with a large multinodular goiter causing tracheal compression. The
preferred management is:
a. Iodine treatment.
b. Thyroid hormone treatment.
<C+>Surgical resection of the abnormal thyroid.
c. Radioactive iodine treatment.
d. External beam radiotherapy.

aa. Advantages of total thyroidectomy(as compared to lobectomy and isthmictomy) for management of papillary
carcinomas of the thyroid larger than 1.5 cm. include all the followings except:
a. Possibility of using radioactive iodine postoperatively to identify and treat metastases.
b. The ability to use thyroglobulin levels as a marker for recurrence.
c. Lower overall recurrence rate.
<C+>Lower risk of hypoparathyroidism.
d. Better control of unrecognized occult multifocal andmulticentricpapillary cancers.

bb. Regarding Addisonian crisis, (or acute adrenocortical insufficiency) one of the following statements is correct:
a. Occurs only in patients with known adrenal insufficiency or in those receiving long-term supra-physiologic doses of
exogenous steroids.
<C+>Can mimic an acute abdomen with fever, nausea and vomiting, abdominal pain, and hypotension.
b. May cause electrolyte abnormalities, including hypernatremia, hypokalemia.
c. Should be diagnosed with the rapid ACTH stimulation test before steroid replacement is instituted.
d. Should be treated promptly with adrenal gland transplantation.

cc. The principal blood supply to the parathyroid glands is which of the following?
a. Superior thyroid arteries.
<C+>Inferior thyroid arteries.
b. Thyroideaima arteries.
c. Parathyroid arterial branches directly from the external carotid artery.
d. Highly variable.

dd. All the following statements about gastric cancer are trueexcept:
a. Pernicious anemia is associatedwith an increased rate of gastric cancer.
b. Intestinal-type gastric tumors have a better prognosis than the diffuse type.
c. Early gastric cancers are confined to the mucosa and submucosa of the stomach.
<C+>The number of involved lymph nodes is of limited prognostic value.
d. H. pylori infection has been found to be associated with gastric cancer.

ee. All the followings are bacterial infections except:


a. Actinomycosis of appendectomy wound.
b. Fournier’ gangrene of the Scrotum.
c. Erysipelas of the skin
d. Hidradenitis suppurativa of the pubic region
<C+>Madura foot

ff. One of the following statements about an amebic abscess is true:


a. Is mainly a disease of female patients.
<C+>The diagnosis of amebic abscess may be based on serologic testing.
b. The treatment of amebic abscess is primarily surgical.
c. Patients with amebic abscess tend to be older than those with pyogenic abscess.
d. Aspiration of the abscess contents yields amebic organisms in 90% of the cases.

gg. All the following statements regarding fat embolism are correct except:
a. It is relatively common but only rarely causes symptoms.
<C+>Neurological dysfunction is not a feature of the disease.
b. Petechial hemorrhages are usually seen in and around both axilla.
c. Fat droplets in sputum and urine are helpful in diagnosis.
d. For well-established cases, positive pressure ventilation is an essential line of treatment.

hh. All of the following statements about renal ectopia are true except:
a. Patients have increased incidence of urinary tract infections and urolithiasis.
b. Patients may have increased incidence of vsico-ureteric reflux or urinary obstruction.
c. Horseshoe kidneys are the most common anomaly.
<C+>Surgery is needed to correct the anomaly in most of the cases.
d. Patients may have increased incidence of urinary tract tumors.

ii. Bonus ouestion: The approximate number of newly diagnosed cases of breast cancer per year in Jordan is:(one
thousand)
a. 500
<C+>1000
b. 1500
c. 2000
d. 2500
1) regarding appendix what is true

-Paucity of lymphoid tissue

-normal up to 8 mm (x: 12.8 mm)

-appendicular artery from right colic (x: ileocolic from SMA)

-structure similar to terminal ilium

2) not indication for incidental appendectomy

-diagnostic laparoscopy for female

- ladds procedure

-surgery to right ovary

- child with cognitive disability undergoing small bowel resection

-old man undergoing sigmoidectomy for diverticulosis

3) most common primary small bowel malignant tumor

-carcinoid

-adenocarcinoma

4) least likely to leak anastomosis

-hand-sewn ilioileal

-stapled ilorectal with ILEOSTOMY

-stapled ilorectal with colostomy

-hand-sewn esophagojejunal

Ileum always least likely

5) type of calcification in chest Ct suggesting lung malignancy

-popcorn calcification (benign)

-stippled calcification (along with eccentric, amorphous and flecks)

-laminar calcification (benign)

6) prognosis of chest wall sarcoma depend on

-histoligical type
-size

-invasion to ribs

-involved margins

7) posterior medistinal tumor not found in adults

-ganglioneuroma yes

-neurofibroma yes

-neurolemoma yes

-malignant schwannoma yes

8) about gallbladder anatomy what is true

- capacity 100 to 300 ml (x: 50 ml)

- capacity increase to 500ml with cholecystitis

- length 7 to 10 cm (yes)

- most muscle fibers in body

9) during cholecystectomy gallbladder nodule frozen section showed T1 adenocarcinoma what


should be done

-resect port site

-convert to open

-cholecystectomy with extensive local lymph node excision

-cholecystectomy with segment 4 or 5 liver resection

-nothing more should be done

10) pancreatic lesion least likely to become malignant

-serous cystic neoplasm

-intraductal papillary mucinous neoplasm (potential)

-pseudo papillary neoplasm (cancer)

-mucinous cystic neoplasm (potential)

11) nerve most likely to injure with parotid surgery


-preauricular

-occipital

-facial

Submand: hypoglossal

12) AAA 6.6 cm possibility of rupture

-10-20 %

-40%

-50%

-60%

13) AAA does not affect EVAR

- length of inrarenal neck

-diameter at inrarenal neck

-atheroma and calcification at neck

-iliac artery diameter

-shape in size of aneurysm

14) initial management of zone 3 penetrating neck injury

-surgical explration

-CT angiogram

-bronchoscopy

-esophagoscopy

15) false about cholangiocarcinoma

- most are intrahepatic (false more than half and prehilar aka klatskin)
-10% of population have Lewis antigen A deficiency so they don't have elevated Ca 19.9

-CEA is elevated in 30% of patients

-CA 19.9 above 1000 is associated with metastasis

-tumor at hepatic duct confluence are referred to as klatskin tumor true

16) about primary hepatocellular carcinoma what percentage of patient have chronic liver disease

-40-60%

-75-80% (even 80 to 90%)

-20-40%

17) 2nd degree 70kg 30% burn, parkland for first 8 hours

-1.2 l

-2.4l

-4.2l

-8.4l

18) about chemical burn what is false

-initial step is identification of nature of chemical agent

-copious irrigation is important in management

-calcium gluconate cream for acidic burn can change it to salts

-acidic burn may cause hypocalcemia

-delayed use of split thickness grafts can play role

19) meshed split thickness graft are used in burn for

-don't retain hematoma

-fast healing of donor site

-better cosmotic results


20) hypercalcemia 13 first step

- thiazide diuritic

-furosemide

-normal saline

-calcitonin

21) crush injury most important

-iv fluid

-bicarb

-mannitol

-Ecg monitor

22) lynch syndrome patient which surgery to do

-proctocolectomy with ileoanal pouch

-proctocolectomy with end ILEOSTOMY

-total colectomy with ileorectal anastomosis

-hemicolectomy

23) missile trauma, False

-nail brush used to removed hard particles

-not all hard objects seen in imaging must be removed

-large fascial opening to explore injuries to muscular compartments

-large skin incision to have better view

-nerves should by dissected and sutured

24) tibial fracture with CT angiogram showing Popliteal artery injury best steps

-shunting, reduction and fixation, arterial repair, fasciotomy

25) between acute and chronic limb ischemia

-pulse in contralateral limb


-location of obstruction

-cardiac arrythmias

-all of the above

26) about submandibular gland anatomy

-lies on posterior aspect of mylohiouyd muscle

-lingual artery divide before entering it

-can safely do incision at margin of mandible

27) 50 YO female with supraclavicular LN fna showing normal thyroid tissue

-excisional biopsy

-lobectomy

-thyroudectomy with central LND

-thyroidectomy with MRND

28) management of medullary thyroid cancer in a patient with us negative for LN

-total thyroidectomy with central LND

-total thyroidectomy

-total thyroidectomy with MRND

29)central lymph node dissection is part of management of

-anaplastic thyroid ca

-papillary thyroid ca

-thyroid lymphoma

- Medullary

30) rectal sessile polyp showed moderately differentiated adenocarcinoma, can be excised locally if

-T1 or T2 (only 1)

-less than 5 cm in size (less than 3 cm) + less than 10 cm from anal verge (less than 15)

-all sessile should undergo segmental resection (no)

- moderetelly with not lymph or vascular invasion


31) TEF most common complication long term comlication

-GERD

-stricture

-recurrence

32)early mortality from pancreatitis is due to

-respiratory failure

-cardiac failure

-liver failure

-sepsis

33) with penetrating cardiac injury the worst prognosis is with

-multichamber injury

-coronary injury
34) which of the following patients with 3 vessels disease is not candidate for cabg

-3 vessels disease with poor ventricular function

-echo showing anterior wall akinesia

-young

-diabetic

-LAD and other two vessels involved

-post multiple stents restenosis

35) most common type of splenic cyst

-hydatid

-epithelial

-simple

(most common pseudocyst)

36) about HIT what is wrong

-defined as 50 % drop in platelet count initially

-platelet transfusion plays rule in management (avoided)

- serum should contain platelet factor 4 and antiplatelet antibodies

-characterized by thrombosis and thrombocytopenia

37) about anticoagulants what is wrong

-argatroban is factor x inhibitor

- dabigatran is reversed by idracizumab

-rivaroxaban don't need bridging

-rivaroxaban is reversed by andexxa


wrong not here

38) about wound retraction

-myofibroblast contract using actin filaments


39) not indication for aortic valve replacement

-pressure gradient more than 60

-asymptomatic patient with severe stenosis (< 0.8 cm)

-regurgitation with angina

-regurgitation with syncope

- valve more than 4 cm (no < 1cm)

40) about meckel diverticulum

-more than 50 % contain gastric mucosa

-if found incidentaly should be resected

-causes lower gi bleeding in children

-is true diverticulum

-found in 2% of population

41) post bariatric surgery has wernickes encephalopathy treatment is

-thiamine IV

42) post bariatric surgery has drop foot and parasthesia at dorsum of foot nerve affected is

-common peroneal nerve

43) medial border of triangle of Doom

-vas deferens
44) about femoral canal

-posterior border is lacunar ligament (post: pectineal ligament + super ramus+ pectineus muscle,
Med: lacunar, Lateral: vein, Anteror: I.L)

-anterior border is inguinal ligament

-is medial to femoral vein

-contains cloquet lymph nodes

45) not an effect of pneumoperitonium

-increase systemic vascular resistance

-bradycardia + tachy)

-increased venous resistance

-increased blood pressure

- Increase/decrease CO, Increase/Decrease CVP, decrease SV.

46) laparoscopic compared to open colectomy

-more lymph nodes harvested

-less complications

47) closely related to magnisium :

-Ca

-Na

-K

48) about GIST what is false

-most common mesenchymal tumor of GI tract

-can be present anywhere in GI tract but mostly stomach

-resection in mainstay of treatment

-age of presentation is below 30 (mostly over 50)

-origin is interstitial cells of cajal


49) about post cath pseudoaneurysm what is wrong

-more common in females

-more common with use of larger sheeth

-occurs when manual compression is used rather than femostop

-defined as bleeding with flow outside the lumen detected by US

50) about birad system what is wrong

-birad 4 needs core biopsy

-birad 6 known to have cancer

-birad 3 mostly benign that need close follow up

-not used in male breast CA

51) post colectomy T3N0 with known first degree family member who has colon Ca and number of
LN harvested is 8 should take chemotherapy because of

-number of lymph nodes (12 ‫)الزم يكون يكونوا‬

-stage

-family history
52) atypical ductal hyperplasia should undergo

-excisional biopsy

- wide excision

-mastectomy

53) about pathology what is wrong

-dysplasia is irreversible

-metaplasia is change in type and shape of cells yes

-hyperplasia is increase in number yes

-atypia is...

54) woman with nipple eczema that did not improve with corticosteroid cream what is not
acceptable as next step

-another course of steroid cream

-MR breast

-mammogram and ultrasound

-biopsy involving nipple ateolar complex

55) 80 YO with Barret esophagus showing high grade dysplasia in all biopsies

-repeat endoscopy and biopsy after 3 months

-argon beam ablation

-esophagectomy

- photodynamic

56) about rectal prolapse,wrong

-mannonetry gives idea about internal and external sphincter pressure

-surgery improves 50% of incontinence

-perinneal surgery for young

-abdominal surgery for old

-in abdominal surgery release anterior and lateral rectal attachments


57) class 2 hemorrhagic shock not correct

-lost 1000 cc blood

-anxiety

-orthostatic hypotension

-pulse 130

58) not part of SIRS

-RR above 20

-T <36

-P >90

-PaCO2 <32

-systolic blood pressure <90

59) about colloid what is wrong

-dextran can cause platelet dysfunction and anaphylaxis yes

-6% hetastarch is derived from polyglycan

-albumin has molecular weight of 67 kda yes

-albumin has half life of 15 days 21 days


60) about pancreatic neuroendocrine tumors

-50% of gastrinomas are malignant

-when malignant found in duodenum

61) surgical indication of flial chest include all except

-severe lung contusion (should be absent/or minimum)

-inability to wean from mechanical ventilator

-patient requiring thoracotomy for another reason

62) patient with diverticulitis has left lower quadrant mass additional to iv fluid patient shoul receive

-iv antibiotics

63) patient with Ogilvie syndrome post orthopedic surgery with immobility and narcotics CT showed
no obstruction but dilated colon 12 cm management is

-iv fluid

-iv neostigmine (can be used)

-nasogastric and rectal tube decompression


64) patient with diverticulitis who is febrile and has tachycardia what to do

-partial colectomy

65) regarding limb amputation what is wrong

-above knee is more suitable for bedridden patients

-above knee wound is more likely to heal

-most above knee patients don't ambulate on limb again

-below knee is associated with 30% increase in energy expenditure

66) surgeon making wound 2 inches above inguinal ligament is likely to meet inferior epigastric
artery

-preperitoneal fat and transversalis fascia

-transversus abdominis and internal oblique

-internal and external oblique

-external oblique and rectus sheeth

--- anterior to transversalis fascia

67) most common complication of brachial cleft cyst

-infection

-malignancy

-bleeding
68) about hirschsprung disease what is false

-pulthrough procedure has best outcome

-suction biopsy containing submucosa is adequate

69)about Dudenal atresia what is wrong

- associated with down syndrome

-best treated by gastrodudenostomy

-associated with Malrotation and annular pancrease

-associated with cardiac anomalies

-can be diagnosed antenataly

70) surgical intervention in NEC should be done when

-change in abdominal skin color

-septic shock

-pneumatosis intestinalis

71) premature babies what is wrong

-need more fluid than full-term

-surfactant is full at 34 weeks

-15 to 20% left to right shunting through foramen ovale and ductus arteriosis occurs

‫كلهم صح‬
72) about TPN what is wrong

-hypophosphatemia should be corrected before starting

-metabolic disturbances occur in 5%


-fat constitute 30 to 50 % of energy

-lead to LFT derangement

73) hard sign of vascular injury

-thrill or bruit

-decreased pulse

-associated nerve injury

-cold limb

-fracture in proximity

74) about thoracic outlet obstruction

-present with neck pain radiating to upper limb

-abscent radial pulse

-corrected surgicaly

- has claudication of upper limb

75) about lymphedema what is wrong

-diagnosed by lymphoscintigraphy (gold standard, but prefer clinical)

-milory syndrome for primary

-surgery for breast cancer most common cause of secondary

- can be disabling to patient

76) essential step in venous ulcer formation

-edema

-varicose vein

-hemosiderin deposition

-high protein exudate

77) day two post whipple drain output 30 cc with amylase 1500 what's next

-remove drain

-keep drain and send amylase at day 5


-CT abdomen

-surgical exploration

78) patient with obstructive jaundice, CT suspicious for CBD malignancy, Ca19. 9 high, Ercp not
diagnostic

-offer exploratory surgery

-ercp with brush cytology

-ercp and stent

-repeat CT after 3 months

79) breast CA histoligical type with best prognosis

-papillary

-ductal

-tubular

-mucinous

-medullary

80) regarding interrupted and continuous suture what is wrong

-interrupted is time consuming

-interrupted carries less risk of dehescence

-tensile strength is same

-interrupted cause more tissue edema

-interrupted allows proper alignment of tissue

81) regarding splenic abscess what is wrong

-caused by gram positive and negative

-can be drained percutaneously

-source is usually hematogenous spread

-usually causes splenomegaly

82) risk factor of liver failure post hepatectomy most common:


-DM

-obesity

-blood loss more than 500 ml

-remaining liver less than 25%

83) post thyroidectomy with stridor

-open bedside

84) hida scan is not helpful in

-hepatocellular dysfunction

85) about steal syndrome indication for surgical management:

-abscent distal pulse

-retrograde flow

-cold paralyzed with wrist drop

86)associated with wilms except

-hemolytic uremic syndrome

87) about necrotizing fascitis

-debride dead tissue (right)

88) in wound healing

-complement and pdgf play important rule in initial steps (true)

89) to decrease risk of biliary injury in lap chole

-critical view of safety

90) malignant biliary stricture

-radio-frequency ablation

-ERCP
91) about colorectal polyps what is true

-hyperplastic has low malignant potential (‫)اصال ما عنده‬

92) anal disease and hematologic problem

-correction of neutropenia

93) dumping syndrome with billroth 2

-high protein small meals + low carbs

94) female with symptomatic mesentric cyst treament

-biopsy

-resection with possible bowel resection

95) comparing gastric bypass to sleeve in weight reduction and p value 0.28

Gastric bypass is superior but this is not significant

96) not candidate for daycare hernia

-ASA 3

-DM

-age above 70

97) about conservative breast surgery

- axillary clearance needed for both (true)

-both need drain (don’t need)

-quadrantectomy is cosmoticaly better than wide local excision (true)

98) about lapchole for gallbladder stones, which is wrong:

-most common presentation biliary colic

-risk of stricture 5%
Residents 3+4:

1. The most common condition which needs liver transplantation in pediatric age group
is:

A. Metabolic liver disease.


B. Malignant liver tumors.
C. Biliary atresia.
D. Choledochal cyst.
E. Viral hepatitis.

Answer is C.

2. Regarding urethral injuries, all are true except:

A. Prostatic hematoma may be present.


B. Perineal hematoma may be seen.
C. Fracture pelvis may be the cause.
D. Descending urethrogram is the investigation of choice.
E. May be treated with suprapubic urinary bladder catheterization.

Answer is D.

3. Regarding overwhelming post splenectomy sepsis, which of the following is true.

A.Pneumococcal vaccination is not protective.

B. It is common after traumatic splenectomy.

C.Mortality rate is low.

D. It is common after splenectomy for blood diseases.

E. It is common after 10 years post surgery.

Answer is D
4- Preoperative bowel preparation helps reduce which of the following postoperative
complications by 40%?

a- Pneumonia

b- Urinary tract infection (UTI)

c-Dehiscence

d- Wound infection

e- Intraabdominal abscess

Answer is :D

7- The single most effective method of diagnosis for a 57-year-old man with a 6-month
history of hoarseness and a neck mass is

a- Fine-needle aspiration of the neck mass

b- Computed tomographic (CT) scan of the head neck and chest

c- Incisional biopsy of the neck mass

d- Excisional biopsy of the neck mass

e- Laryngoscopy with biopsy

Answer is : A

8- A 24-year-old man undergoing laparotomy for symptomatic Crohn's disese has a 2-cm
stricture of the mid-ileum without any evidence of disease elsewhere in the
gastrointestinal tract .

The best surgical option for this lesion would be

a- Resection with primary anastomosis

b- Heineke-Mikulicz strictureplasty

c- Heineke –Mikulicz strictureplasty with biopsy


d- Mechanical dilatation

e- Isoperistaltic side – to –side strictureplasty

Answer is : A

9- A 30 year –old-woman with recurrent peptic ulcer disease has a markedly elevated
serum gastrin level. The most likely site for her tumor is

a- Duodenum

b- Liver

c- Stomach

d- Lymph node

e- Pancreas

Answer is : A

10- Which of the following statements about aspiration of pancreatic cyst contents is true

a- It is not beneficial when the patient has a history of pancreatitis

b- Resection is not necessary if cyst fluid amylase level is elevated

c- It should be obtained if there is a central "starburst" calcification in the cyst

d- Cytology is useful in excluding malignancy

e- High cyst carcinoembryonic antigen (CEA)level excludes serous cystadenoma

Answer is : E

11- The most effective treatment for thromboangitis obliterans in the lower extremity is

a- Lumber sympathectomy

b- Cessation of smoking
c- Amputation

d- Inframalleolar bypass

e- Prostacycline analogue

Answer is : B

12. Eight hours after an infant exhibits excessive drooling and mild respiratory
distress. An abdominal radiograph shows complete lack of air in the gastrointestinal
tract. Which is the most likely diagnosis?

a. Hirschsprung”s disease.
b. Tracheoesophageal fistula H type.
c. Pyloric atresia.
d. Choanal atresia.
e. Esophageal atresia without Tracheoesophageal fistula

(E)

13. All of the following form radiolucent stones except:


A.Xanthine
B.Cysteine
C.Allopurinol
D. Orotic acid

E. purine

Ans:B

14. The feature most important in differentiating a malignant from benign tumor IS:

A. Lack of encapsulation.
B. High mitotic rate.
C. Presence of necrosis.
D. Presence of metastases.
E. Nuclear pleomorphism (anaplasia)

(D)

15- All the following pharmacologic agents can be used in the treatment of thyrotoxicosis
to block the production of thyroid hormone, EXCEPT?

a.Propylthiouracil
b.Propranolol
c.Methimazole
d.Carbimazole
e. Iodine

(B)

16- Which of the following statements about the diagnosis and treatment of esophageal
leiomyomas is/are correct :

. A.The majority are diagnosed after they cause dysphagia and chest pain.
B. Biopsy is indicated at the time of esophagoscopy, to rule out carcinoma.
C. Full-thickness elliptical excision of the esophageal wall is the preferred surgical
approach.
D. Endoscopic ultrasonography is a reliable means of following leiomyomas
conservatively.
E. Recurrence of resected leiomyomas is minimized by wide local excision.

(D)

17- Which of the following is contraindications to gastric bypass surgery?


A.Diabetes.
B.Hypertension.
C.Pickwickian syndrome

.D.Failure to agree to long term follow up

E,Sleep apnea.
(D)

18. Which of the following statements about the lower esophageal sphincter (LES)
mechanism, or high-pressure zone (HPZ), is true?

A. The LES is a circular smooth muscle ring that is 3 to 5 cm. long.

B. In assessing esophageal manometric data, mean HPZ pressure less than 6 mm. Hg or
overall length less than 2 cm. is more likely to be associated with incompetence of the
LES and gastroesophageal reflux.

C. Esophageal manometry and the acid perfusion (Bernstein) test reliably identify the
patient with an incompetent LES mechanism.

D. Distal HPZ relaxation occurs within 5 to 8 seconds of initiating a swallow.

E. Twenty-four–hour distal esophageal pH monitoring is achieved with an


intraesophageal pH electrode positioned at the esophagogastric junction.

Answer: B

19. Which of the following statements about achalasia is/are correct?

A. In most cases in North America the cause is a parasitic infestation by Trypanosoma


cruzi.

B. Chest pain and regurgitation are the usual symptoms.

C. Distal-third esophageal adenocarcinomas may occur in as many as 20% of patients


within 10 years of diagnosis.

D. Manometry demonstrates failure of LES relaxation on swallowing and absent or weak


simultaneous contractions in the esophageal body after swallowing.

E. Endoscopic botulinum toxin injection of the LES, pneumatic dilatation, and


esophagomyotomy provide highly effective curative therapy for achalasia.

Answer: D
20. Which of the following statements about epiphrenic diverticula of the esophagus
is/are correct?

A. They are traction diverticula that arise close to the tracheobronchial tree.

B. They characteristically arise proximal to an esophageal reflux stricture.

C. The degree of dysphagia correlates with the size of the pouch.

D. They are best approached surgically through a right thoracotomy.

E. The operation of choice is a stapled diverticulectomy, long esophagomyotomy, and


partial fundoplication.

Answer: E

21. Which of the following statements about the presence of gallstones in diabetes
patients is/are correct?

A. Gallstones occur with the same frequency in diabetes patients as in the healthy
population.

B. The presence of gallstones, regardless of the presence of symptoms, is an indication


for cholecystectomy in a diabetes patient.

C. Diabetes patients with gallstones and chronic biliary pain should be managed
nonoperatively with chemical dissolution and/or lithotripsy because of severe
complicating medical conditions and a high operative risk.

D. The presence of diabetes and gallstones places the patient at high risk for pancreatic
cancer.

E. Diabetes patients with symptomatic gallstones should have prompt elective


cholecystectomy, to avoid the complications of acute cholecystitis and gallbladder
necrosis.

Answer: E

. 22. Which of the following statements regarding unusual hernias is incorrect?

A. An obturator hernia may produce nerve compression diagnosed by a positive


Howship-Romberg sign.
B. Grynfeltt's hernia appears through the superior lumbar triangle, whereas Petit's hernia
occurs through the inferior lumbar triangle.

C. Sciatic hernias usually present with a painful groin mass below the inguinal ligament.

D. Littre's hernia is defined by a Meckel's diverticulum presenting as the sole component


of the hernia sac.

E. Richter's hernia involves the antimesenteric surface of the intestine within the hernia
sac and may present with partial intestinal obstruction.

Answer: C.

23. Staples may safely be placed during laparoscopic hernia repair in each of the
following structures except:

A. Cooper's ligament.

B. Tissues superior to the lateral iliopubic tract.

C. The transversus abdominis aponeurotic arch.

D. Tissues inferior to the lateral iliopubic tract.

E. The iliopubic tract at its insertion onto Cooper's ligament.

Answer: D

24. The following statements about the repair of inguinal hernias are true except:

A. The conjoined tendon is sutured to Cooper's ligament in the Bassini hernia repair.

B. The McVay repair is a suitable option for the repair of femoral hernias.

C. The Shouldice repair involves a multilayer, imbricated repair of the floor of the
inguinal canal.

D. The Lichtenstein repair is accomplished by prosthetic mesh repair of the inguinal


canal floor in a tension-free manner.

E. The laparoscopic transabdominal preperitoneal (TAPP) and totally extraperitoneal


approach (TEPA) repairs are based on the preperitoneal repairs of Cheattle, Henry,
Nyhus, and Stoppa.
Answer: A

25. The following statement(s) is/are true concerning umbilical hernias in adults.

a. Most umbilical hernias in adults are the result of a congenital defect carried into
adulthood
b. A paraumbilical hernia typically occurs in multiparous females
c. The presence of ascites is a contraindication to elective umbilical hernia repair.
d. Incarceration is uncommon with umbilical hernias
e. Repair always requires mesh

Answer: b

26 A number of special circumstances exist in the repair of inguinal hernias. The


following statement(s) is/are correct.

a.Simultaneous repair of bilateral direct inguinal hernias can be performed with no


significant increased risk of recurrence

b. The preperitoneal approach may be appropriate for repair of a multiple recurrent hernia

c. femoral hernia repair can best be accomplished using a Bassini or Shouldice repair

d, Management of an incarcerated inguinal hernia with obstruction is best approached via


laparotomy incision

Answer: b

27. An absolute contraindication to breast-conserving

surgery for breast cancer is:

(a) large tumor

(b) tumor of high grade

(c) early pregnancy

(d) retroareolar tumor

(e) clinical axillary nodes


Answer :( c)

28. Gastrointestinal stromal tumors (GIST):

(a) occur most commonly in the duodenum

(b) are almost always malignant

(c) can be treated adequately with enucleation

(d) are often radioresistant

(e) spread mainly via the lymphatics

Answer D

29.Biliary-enteric fistula most commonly connects:

(a) gallbladder and ileum

(b) gallbladder and duodenum

(c) common bile duct and jejunum

(d) gallbladder and jejunum

(e) common bile duct and ileum

Answer :B.

30. The hepatic caudate lobe:

(a) drains directly into the inferior vena cava

(b) represents segment IV

(c) is supplied by the left portal vein only

(d) is supplied by the right portal vein only

(e) lies to the right of the inferior vena cava

Answer a
31. Small bowel obstruction in an elderly female

without external hernia or previous surgery is most likely caused by:

(a) small bowel neoplasm

(b) volvulus

(c) gallstone ileus

(d) abdominal abscess

(e) obturator hernia

Answer: c

32. The development of thrombocytopenia and arterial

thrombosis with heparin requires:

(a) continuation of heparin and platelet trans-

fusion

(b) continuation of heparin and thrombolysis

(c) doubling the heparin dosage

(d) changing the route of heparin administration

e) discontinuation of heparin

answer e
33.The risk of overwhelming postsplenectomy sepsis is

highest for patients requiring splenectomy for:

(a) thalassemia

(b) trauma

(c) immune thrombocytopenic purpura

(d) hereditary spherocytosis

(e) acquired hemolytic anemia

Answer a

34 After mastectomy, winging of the scapula results


from injury to:
(a) the medial pectoral nerve

(b) the lateral pectoral nerve

(c) the long thoracic nerve

(d) the thoracodorsal nerve

(e) the intercostal-brachial nerve

Answer c

35 A sudden onset of glucose intolerance in patients


receiving total parenteral nutrition often indicates:

(a) diabetes mellitus

(b) sepsis

(c) hypophosphatemia

(d) adrenal insufficiency

(e) zinc insufficiency

Answer b

36 Intra-aortic balloon pump:


(a) increases pulmonary wedge pressure

(b) increases afterload

(c) increases diastolic pressure

(d) increases duration of systole

(e) decreases duration of diastole

Answer c

37: Which of the following distinguishes adrenal insuf-


ficiency from sepsis?

(a) hypotension

(b) fever

(c) tachycardia

(d) altered mental status

(e) hypoglycemia
Answer e

38. The most common etiologic factor for hepatocellu-


lar carcinoma worldwide is:

(a) hepatitis C virus

(b) hepatitis B virus

(c) alcoholic cirrhosis

(d) aflatoxin ingestion

(e) schistosomiasis

Answer b

39. The main complication of topical silver nitrate is:


(a) metabolic acidosis

(b) metabolic alkalosis

(c) hyperkalemia

(d) hypocalcemia

(e) hyponatremia

Answer :e

40. The adverse effects of steroids on wound healing


can be reversed with:

(a) vitamin C

(b) vitamin A

(c) copper

(d) vitamin D

(e) vitamin E

Answer b

41 Which of the following is least appropriate


when evaluating a 14-year-old girl with a breast lump?
(a) ultrasound

(b) clinical follow-up

(c) mammography

(d) fine-needle aspiration

(e) excisional biopsy

Answer c

42. Smoke inhalation is most reliably excluded by:


(a) absence of carbonaceous sputum

(b) normal carboxyhemoglobin level

(c) normal xenon-133 inhalation scan

(d) normal chest x-ray

(e) normal flexible bronchoscopy

Answer e

43. Hypotension develops after pneumoperitoneum


and trocar placement for laparoscopic cholecystec-

tomy. The next action is to:

(a) convert to open cholecystectomy

(b) deflate the abdomen

(c) give intravenous fluids

(d) place the patient in head down position

(e) check for bowel injury

Answer b

44. The most common cause of esophageal perforation


is:

(a) penetrating neck injury


(b) iatrogenic

(c) spontaneous

(d) foreign body

(e) malignancy

answer b

45 . The most common site for accessory splenic

tissue is:

(a) gastrosplenic ligament

(b) gastrocolic ligament

(c) splenic hilum

(d) splenocolic ligament

(e) the pelvis

Answer c

46 Lobular carcinoma in situ:

(a) is mostly found in premenopausal women

(b) usually presents as a breast lump

(c) has characteristic calcification pattern on mam-

mography

(d) is precancerous

(e) associated cancer is lobular in nature

Answer a

47 Optimum approach to inflammatory breast car-

cinoma is:

(a) total mastectomy

(b) modified radical mastectomy

(c) lumpectomy and radiotherapy


(d) chemotherapy

(e) chemotherapy, modified radical mastectomy,

and radiotherapy

answer e

48 Appropriate management of 3-cm squamous cell

carcinoma of the anal canal is:

(a) chemotherapy

(b) abdominoperineal resection

(c) local excision

(d) radiotherapy  local excision

(e) chemotherapy  radiation

Answer e

49. Pleomorphic parotid adenoma:

(a) requires core biopsy before resection

(b) is adequately treated with enucleation

(c) commonly undergoes malignant transformation

(d) commonly results in facial palsy

(e) is the most common parotid neoplasm

Answer e

50 The most common cause of nipple discharge is:

(a) duct ectasia

(b) breast cancer

(c) intraductal papilloma

(d) pituitary adenoma

(e) fibrocystic disease

Answer c
51 The optimal treatment for bleeding gastric varices

in chronic pancreatitis is:

(a) distal pancreatectomy

(b) splenorenal shunt

(c) portocaval shunt

(d) splenectomy

(e) transjugular intrahepatic portosystemic shunt

Procedure

Answer d
1. With regard to total body water, all of the following are true except:

A. 50 to 70% of total body weight is water.


B. In general the percentage of total body weight that is water is higher in males
than in females.
C. Lean individuals have a greater proportion of water (relative to body weight)
than do obese individuals.
D. The percentage of total body weight that is water increases with age.*
E. Body water is divided into extracellular (i.e. intravascular and interstitial) and
intracellular functional compartments.

2. With regards to the distribution and composition of the body fluid


compartments, which of the following statement is incorrect:

A. The majority of intracellular water is in skeletal muscle.


B. The major intracellular cation is sodium.*
C. The major intracellular anions are the proteins and phosphates.
D. The major extracellular cation is sodium.
E. The major extracellular anions are chloride and bicarbonates.

3. With regard to respiratory acidosis and alkalosis, all of the following


are true except:

A. Respiratory acidosis is associated with an increased denominator of the


Henderson- Hasselbalch ratio that is due to CO2 retention, resulting in a ratio
of less than 20:1.
B. Respiratory alkalosis is associated with a decreased denominator that is due to
a loss of CO2, resulting in a ratio of greater than 20:1.
C. Compensation for respiratory acidosis is primarily renal.
D. Compensation for respiratory alkalosis is primarily pulmonary. *

4. With regard to risk factors for breast cancer, one of the following is
true:

A. Incidence does not appear to be age related among those older than age 35
years.
B. Family history is not a major predictor of risk for developing breast cancer.
C. Late first pregnancy increases the risk.*
D. Diet and weight have no association with breast cancer risk.
5. Which of the following thyroid adenomas may in rare instances
behave in a malignant manner:

A. Colloid adenoma.
B. Embryonal adenoma.
C. Fetal adenoma.
D. Hurthle cell adenoma.*

6. With regard to the technique of needle aspiration biopsy of the


thyroid, one is true:
A. It is generally contraindicated because of the extremely vascular nature of
the thyroid gland.
B. False positive results are rare.*
C. False negative results occur less often than false positive results.
D. There is a 3% risk that cancer cell will implant.
E. Benign versus malignant follicular neoplasm are fairly easily differentiated .

7. With regard to vitamin D physiology, one is true:


A. The major non-dietary source of vitamin D is hepatic synthesis.
B. Hydroxylation of vitamin D3 results in a loss of metabolic activity.
C. Vitamin D decreases intestinal absorption of dietary calcium.
D. Vitamin D has a direct effect on bone resulting in ossification of the bone.*
E. Increased level of PTH inhibits the hydroxylation of 25-
hydroxycholiecalciferol in the kidneys.

8. Which of the following is the most important mechanism for


maintaining competence of the gastroeosophageal junction:

A. Intact vagus nerve.


B. Diaphragmatic crural pinch.
C. Angle of the esophageal entry into the stomach.
D. Orientation of muscle fibers of the esophageal sphincter.
E. Intraabdominal segment of the esophagus.*

9. With regard to Para esophageal hiatal hernia, all of the following are true except:

A. Symptomatic gastroeosophageal reflux is the most common complication.


B. Often there is an associated sliding hernia.
C. Large asymptomatic hernia necessitate repair.
D. Surgical treatment usually involves an antireflux procedure.
E. Pain is common.

10. One of the following is true regarding perforated duodenal ulcer:


A. Can be ruled out if pneumoperitoneum is demonstrated.
B. Can be treated non-operatively.
C. Requires a definitive anti ulcer procedure at the time of the operation.
D. Most commonly occur with posterior ulcers.
E. Is a contraindication to parietal cell vagotomy because of the risk of
mediastinal contamination.

General
1.D 2.B 3.D 4.C 5.D 6.B 7.D 8.E 9.A
10.B
1. All the following structures lie in the transpyloric plane except:

A. The neck of the pancreas.


B. The left kidney.
C. The fundus of the gallbladder.
D. Duodenojejunal junction.
E. The celiac trunk.*

2. All of the following muscles have attachment to the costal cartilage


except:

A. Pectoralis major.
B. Diaphragm.
C. External oblique.*
D. Transversus abdominis.

3. Regarding the sternal angle (angle of Louis) all true except:

A. Marks the demarcation between the superior and inferior mediastinum.


B. Marks the level of the top of the arch of the Aorta.*
C. Marks the level of the second costal cartilage.
D. Marks the level where the azygous vein drains into the superior vena cava.
E. Corresponds to the level of intervertebral disc between 4 th and 5th thoracic
vertebrae.

4. All of the following veins drain into the left brachiocephalic vein
except:
A. Vertebral.
B. Thymic.
C. Internal thoracic.
D. Left supreme intercostal vein.
E. Superior thyroid. *
5. During exercise, all true except:

A. Pulmonary ventilation increases up to a maximum of 10- fold.*


B. The depth of ventilation increases before the rate.
C. Oxygen debt can occur up to a maximum of 20 liters.
D. The total peripheral resistance falls.
E. Energy is derived from the local glycogen store rather than from extra
muscular carbohydrate.

6. On ascent to high altitude, all true except:

A. There is increased erythropoieten secretion.


B. Mountain sickness is mainly caused by cerebral edema.
C. The cardiac output is reduced.*
D. The oxygen dissociation curve is shifted to the right.
E. Pulmonary edema can occur in the unacclimatized individual.

7. Regarding thyroxin, all true except:

A. Acts via cyclic adenosine monophosphate.*


B. Can bind to albumin.
C. Has a half-life of about 7 days.
D. Is mainly metabolized by muscle and liver.

8. All of the following are true regarding absorption in the G.I. tract
except:
A. Active absorption of calcium occurs mainly in the duodenum.
B. Glucose and galactose compete for the same carrier system.
C. Protein may be absorbed as di and tri peptides.
D. Bile acids are predominantly absorbed in the terminal ileum.*

9. All of the following are true regarding bile, except:


A. Its electrolyte basis is an alkaline solution resembling that of the pancreatic
juice.
B. The main primary bile acids are colic acid and deoxycolic acid.*
C. It is concentrated in the gall bladder.
D. Bile salts entering the colon when the terminal ileum is resected prevent sodium
and water reabsorption.
10. Regarding calcium, all true except:
A. Total plasma levels are about 2.5mmol/L (10microgram/dl).
B. Is actively absorbed in the intestine.
C. Given intravenously as gluconate.
D. Is useful in the management of hypokalemia.*
E. Levels are raised in blood taken from a standing subject.

1.E 2.C 3.B 4.E 5.A 6.C 7.A 8.D 9.B


10.D

1. Alveolar ventilation(V) & perfusion(Q) are;


A. Evenly distributed through out the lung
B. Segmental & cannot be measured
C. Inversely related through out the lungs
D. Highest in the bases while V/Q is low*
E. Low in the apices & in the bases

2. If arterial pCO2 has markedly increased(pCO2 70 torr) one should


A.Rapidly decrease pCO2 to 40 torr.
B. Rapidly hyperventilate the patient
C.Decrease pCO2 slowly (1 torr.per minute)*
D.Infuse a pressure agent
E.Increase concentration of O2.

3. The main objective of therapy in ARDS is


A.Increase the number of functioning alveoli*
B. Increase PaO2 to more than 70 torr.
C. Increase pulmonary compliance
D. To maintain an open airway
E.To decrease the work of breathing

4.Which of the following is characteristic of Barrett’s ulcer?

A.Benign histology*

B. Location at the leading edge of the squamocolumnar junction

C. Occurrence in 30% 0f patients with Barrett’s esophagus

D. Presentation with cervical dysphagia

E.Responsiveness to H2 blockers
5. All the following are characteristics of the thoracic duct EXCEPT:
A. It takes its origin in the abdomen
B. It enters the chest through the esophageal hiatus*
C. In the neck, it is located behind the carotid sheet & jugular vein
D.It contains many valves to protect it against blood flow
E.Injury to the duct below T5 usually results into a right sided chylothorax
.
6. Regarding the right main bronchus, all the following are true EXCEPT:
A. It is longer & wider than the left*
B. It extends from the carina down to the origin of middle lobe bronchus
C. Its structure is identical to trachea
D. The right upper lobe bronchus leaves the main bronchus outside the hilum
E. It is more vertical than the left

7. The diagnosis of neurogenic thoracic outlet syndrome is usually established by:

A. Cervical spine Xray.


B. History & physical examination*
C. Nerve conduction studies
D. Digital plethysmography
E. Adson’s test

8. Causes of metabolic acidosis include all the following EXCEPT:

A. Diabetes millets
B. Vomiting*
C. Starvation
D. Renal failure
E. Small bowel fistula

9. All the following components or qualities of stored whole blood tend to

decrease over time EXCEPT:

A. Red blood cell viability


B. Potassium concentration*
C. pH
D. Platelet activity
E. O2 carrying capacity
10.Clinical features of Plummer-Vinson (Paterson-Killey) syndrome include all

the following EXEPT:

A. Atrophic oral mucosa


B. Subacute combined degeneration of the cord*
C. Koilonychia
D. Anemia
E.Dysphagia

11. Which of the following is most useful for determining the need for surgery in

a patient with a pharyngeal diverticulum (Zenker’s)?

A. Clinical history & esophagogram*


B. Flexible fiber-optic esophagoscopy
C. Manometry of the upper esophageal sphincter
D. Measured thickness of the cricopharyngeus sphincter on the esophagogram
E.24-hour distal esophageal pH monitoring

12. The procedure responsible for the largest number of esophageal perforations

is:

A. Esophageal bouginage
B. Esophageal sclerotherapy
C. Flexible esophagoscopy*
D. Insertion of naso-gastric tube
E.Pneumatic dilatation

13. Each of the following steps is appropriate in the initial management of patients

with acute corrosive injury to the esophagus, EXCEPT:

A. Administration of antibiotics
B. Administration of emetic agents*
C. Cricothyroidotomy or tracheostomy
D. Fiberoptic laryngoscopy
E. Placement of large bore intravenous cannula
14. The primary pathophysiologic effect of a pericardial effusion that produces

tamponade is:

A. Cardiac irritability
B. Impairment of left ventricular filling
C. Impairment of right ventricular filling*
D. Production of pulmonary edema
E. Shift of the superior & inferior vena cava

15. Each of the following statement regarding the anatomy and function of the

phrenic nerve is correct EXCEPT:

A. An accessory phrenic nerve commonly occurs in the neck but is rare in the thorax.
B. Diaphragmatic pacing requires intact lower motor neurons of the phrenic nerves
and viable cell bodies of C3, C4, and C5.
C. The left phrenic nerve passes anterior to the scalenus anterior muscle.
D. The phrenic nerve is the sole motor nerve supply to the diaphragm.
E. The phrenic nerve supplies sensory fibers only to the superior surface of the
diaphragm.*

16. The commonest tumor of trachea is:

A. Squamous cell carcinoma


B. Adenocarcinoma
C. Adenoid cystic carcinoma (cylendroma)*
D. Carcinoid tumor
E. Oat cell carcinoma

17. The bronchial mucosa is composed of all the following EXEPT:

A. Pseudostratified epithelium
B. Basal cells
C. Goblet cells
D. Adeno-cystic cells*
E. Cilia
18. Spinal cord injury at the level of C2, results IN:

A. Transient apnea
B. Permanent apnea
C. Loss of breathing automaticity(Ordine curse)*
D. Cheyne-Stokes respiration
E. Kussmaul respiration

19.Hoarseness secondary to bronchogenic carcinoma is usually due to extension of

the tumor in to which structure?

A. Vocal cord
B. Superior laryngeal nerve
C. Left recurrent laryngeal nerve*
D. Right vagus nerve
E. Larynx

1 --- D
2 --- C
3 --- A
4 --- A
5 --- B
6 --- A
7 --- B
8 --- B
9 --- B
10 – B
11 –A
12 ---C
13 ---B
14 --- C
15 --- E
16 --- C
17 --- D
18 --- C
19 --- C
1. Generalized edema results from all of the following disorders EXCEPT
A. Systemic hypertension.
B. Congestive heart failure.
C. Cirrhosis.
D. Nephrotic syndrome.
E. Hyperaldosteronism. (A)

2. Disorders that predispose to thrombosis include all of the following


EXCEPT:
A. Pancreatic carcinoma.
B. Pregnancy.
C. Vitamin K deficiency.
D. Sickle cell anemia.
E. Severe burns
(C)

3. The feature most important in differentiating a malignant from benign


tumor IS:
A. Lack of encapsulation.
B. High mitotic rate.
C. Presence of necrosis.
D. Presence of metastases.
E. Nuclear pleomorphism (anaplasia)

(D)

4. In inflammatory response, neutrophils release molecules that have all


of the following effects EXCEPT:
A. Chemotaxis of monocytes.
B. Chemotaxis of lymphocytes.
C. Degranulation of mast cells.
D. Increase vascular permeability independent of histamine
release.
E. Connective tissue digestive.

(B)
5. All of the following tumors are malignant EXCEPT:
A. Glomus tumor.
B. Ewing’s sarcoma.
C. Wilms’ tumor.
D. Seminoma.
E. Histocytosis X.
(A)

6. Malignancies that commonly occur in children younger than 5 years of


age include all of the following EXCEPT:
A. Leukemia.
B. Wilms’ tumor.
C. Retinoblastoma.
D. Rhabdomyosarcoma.
E.Osteosarcoma. (E)

7. All the following statements about vitamin A are true EXCEPT:


A. Deficiency causes blindness.
B. Liver disease causes deficiency.
C. Toxicity causes liver disease.
D. Deficiency causes squamous metaplasia in glandular
epithelium.
E. Toxicity is associated with increase infections.
(B)
8. Which of the following statements about vitamin K deficiency is
TRUE:
A. Deficiency causes defects in both the intrinsic and extrinsic
coagulation pathways.
B. Deficiency produces defects in platelets function.
C. Deficiency is associated with strict vegetarian diet.
D. Interstitial bacteria produce enough vitamin K to prevent
deficiency under normal conditions.
E. Deficiency develops less frequently in breast fed infants
than in bottle fed infants.
(A)
9. All of the following commonly contribute to postoperative atelectasis
after uncomplicated abdominal surgery EXCEPT:
A. Adult respiratory distress syndrome.
B. Diaphragmatic elevation.
C. Voluntary suppression of coughing.
D. Excessive bronchial secretion.
E. Limitation of respiratory movements.
(A)

10. diffuse alveolar damage (adult respiratory distress syndrome) is the


major pattern of pulmonary damage produced by all of the following
EXCEPT:
A. Oxygen toxicity.
B. Narcotic overdose.
C. Septic shock.
D. Cardiopulmonary bypass surgery.
E. Pneumothorax. (E)

11. All of the following statements about disseminated intravascular


coagulation are true EXCEPT:
A. the disorder is characterized by widespread thrombosis.
B. The disorder is characterized by widespread of
hemorrhage
C. It most often present as a primary (idiopathic condition)
D. The brain is the organ most often involved
E. The disorder is associated with mucin-secretion
adenocarcinoma
(C)
----------------------------------------------------------------------------------
----------------------------------------------------------------------------------
----------------------------------------------------------------------------------
----------------------------------------------------------------------------------
1. Which one of the following statements about duodenal atresia
and stenosis in the newborn is not true:

a. prenatal detection of duodenal atresia has remained constant


over the past three decades.
b. a mucosal web with a normal muscular wall is the most
common duodenal abnormality.
c. the double-bubble sign on plain films is the classic x-Ray
finding.
d. downs syndrome is identified in up to 25%of infants.
e. cardiac abnormalities are commonly associated with
duodenal atresia.

(a)

2. Jaundice in jejunal atresia:

a. Occur in 85% of cases.


b. More common in ileal atresia than in jejunal atresia.
c. Mainly due to unconjugated fraction of bilirubin.
d. Occur in less than 5%.
e. More common in males than females.
©

3. Which of the following statements about hypertrophic


pyloric stenosis in the newborn is not true?

a. There is familial predisposition.


B. Hypochloremia and hypokalemia may be present.
c. The pylorus can frequently be palpated on physical
examination.
d. Emesis on feeding is common in the early postoperative
period.
e. It is best diagnosed by an upper gastrointestinal study.

(e)

4. For pediatric patients with appendicitis, which of the


following statements is not true:

a. The rate of misdiagnosis is highest in children under 3


years old.
b. Computed tomographic (CT) scanning is less accurate in
children than adults.
c. CT scan is used to rule out peri-appendiceal abscess.
d. Urinary sepsis is the most common misdiagnosis.
e. WBC count does not effectively differentiate perforated
from nonperforated appendicitis.

(d)
5. Eight hours after an infant exhibits excessive drooling and
mild respiratory distress. An abdominal radiograph shows
complete lack of air in the gastrointestinal tract. Which is the
most likely diagnosis?

a. Hirschsprung”s disease.
b. Tracheoesophageal fistula H type.
c. Pyloric atresia.
d. Choanal atresia.
e. Esophageal atresia without Tracheoesophageal fistula

(E)
2017/5/2 ‫امتحان الجراحة العامه للمقيمين‬

: 1. Most mets. Of esophageal cancer

Liver Lung

:2. Retrosternal goiter , incision type

Low collar

:3. Papillary thyroid cancer ,wrong

Worst prognosis

4. Transplant between different species

Xenograft

5. All causes rise in CVP except

PE

Myocardial contusion

Cardiac taponade

Tension pneumothorax

6. Mostly causes pancreatitis in children

Trauma

7. Mosly causes liver abscess in children

TrUma

Perforated appendicitis

Immunocompromised patients

8. Ischemic colitis

May present with colonic stricture

Obstruction on IMA

9. Liver hemangioma wrong

It's the 2nd most common benign liver tumor

:10. All are causes of AAA except

dm><
Htn

Enzymes

11. Typical presentation of diverteculitis and the pt. Looks toxic, Tt

IV antibiotics and observation

Hartmans procedure

:12. Patient with sever bleeding , with inr 11 , what to give

Prothrombin concebtrates and vit k

Vit k

Ffp+vit k

:13. Cause lung fubrosis

Bleomycin

Adrimycin

Methotrexat

:14. M.c salivary gland cancer

Mucoepidermoid cancer

:15. All of the following cause compartment syndrome except

Massive fluid resuscitation-

penetrating injury-

combined arterial and venous injury -

Poplitial fossa trauma

:16. Q about salivary gland stones , whats wrong

Stones are radiolucent

Most common in submandibular gland

Cause distruction for the gland

: 17. Gas pressure in lap.chole

12

:18. All of the following are true except


Atypical ductal hyperplasia does not increase risk of breast Ca

19. Ngt coiling + gassles stomach :

esophageal atresia no fistula

?20. 70 kg patient , with na 120 , what is sodium deficit

840

: 21. Best approach in cardiac tambonade

Left lateral thoracotomy

Subexphoid

Median sternotomy

Rt lateral thoracotomy

:22. Most common complication of hepatic adenoma

Infection

Rupture and henorrahge

Malignancy transformation

23. PBS :

anti metacondrial

24. all are contraindication for pt to be a kidney doner except:

Gestational DM once

Bp 140/100

25. all are criteria to d/c patient post ambulatory sugery except :

Patient ambulate

Good pain control

solid food

26. Pt presented with jaundice and wt loss .. palpable nontender gallbladder

Courvoisers sign

27. Conservation breast surgery :

restricted to 1 quad only


central

28. 2 years old child with with proximal jeujenal atresia 20 cm , distal ilium atresia 8cm , the
best management :

Double barrel loop ilostomy then definitive surgery

29. Wrong about Liver trauma

Mostly needs laparatomy

30. Most common to have opss after splenectomy

Thalsemia

Itp

Trauma

31. all are true about laproscopic surgery true except :

Increase risk of wound infection

:32. You have near total left colonic obstruction what next step

metastatic work up and manage accordingly -

metastatic work up abd colectomy regardless of mets state -

33. about pancreatic tumers true except :

Less than 20% involve head of pancreas

34. Peutz jeghers all true except

Bleeding is the most common presentation

35. Not an indication of bariatric surgery

Psychiatric illness

:36. What false

Clostridium perfringes are gram negative soore forming bacetria that produce toxins

37. All are true about bariatric surgery except

Sleeve is malabsortive with a restrictive component

R EN Y restrictive with malabsorbtive component

38. synchronous colonic tumors in an obstructing

Palpation of the rest colon intra op


Ct colonograph

Barium enema

Colectomy with colonoscopy after 3 months

39. Most common gastric outlet obstruction :

PUD

Cancer

External compression

40. about maxillofacial face trauma:

Mandible most common fracture

41. during trocars insertion in laproscopic surgery , the most common organ to be injured :

Bowel

Aorta

Common iliac

42. Hx of intestinal obstruction with air in the biliary tree

Gallstone ileus

:43. All are increase surgical infection except

Wound tension

Depth

Shaving

Postive op room pressure

44. Pt did appendectomy, antibiotoc prophylaxis for infective endocarditis s needed in this
:pt if he have

hx of coronary artery surgery -

?defibrilator and endicardial leads -

:45. All are complication of TPN except

Hyperglycemia -

Hypoglycemia -

Hyperkalemia -
Hypercalcemia -

Hypoproteinemia -

:46. All are DDx for intermittent claudication except

symptomatic baker cyst -

diabetic neuropathy -

hip osteoarthritis -

spinal canal stenosis -

:47. Level I, II, III axillary lymph nodes are classified acording to their relation with

Pectoralis minor muscle

:48. True except

Hemangioma second most common benign liver tumor

:49. Hyponatremia pt can be caused by except

adrenocortical overactivity -

small bowel obstruction -

diarrhea -

:50. Pre-op thyroid surgery preperation except

potassium iodide can be given for 10 days -

usually used are anthyroid drugs -

antithyroid meds can take from 2 weeks - 3 months for symptons contral -

B- blockers should be used for tachycardia cobtrol -

:51. All are true regarding Medullaru thyroid CA except

origniate from parafollicular C cella -

produce calcitonin -

can be part of MEN 2 syndrome -

operation of choice is total thyroidectomy -

10% sporadic

:52. Most specific test for lung subcarinal lymph node mets
Bronchoscopy with endobronchial ultrasound -

Ct chest -

Vats mediastinonoscopy

:53. All of the follwing can be presentation of varicose vein except

bleeding from varix -

ulceration -

pain in calfs aggravated by ealking and releived by rest + -

heaviness -

:54. Most common presentation of PVD

Asymptomatic

Rest pain

Intermittent claufication

55. 40 years old man , hx of travil to Africa , 1 week later RUQ pain with fever , what is true :

Anchovy sauce

56. You have a patient that is already anticoagulated, and you want to
:investigate for thrombophilia, all can be done except

Protein C&S -

lupus anticoagulant -

prothromin mutation test -

factor V leiden -

57. All of the following drugs can lower cardioembolic events in atrial fibrillation patients
:expect

Dabigatran -

Apixaban -

Rivaroxban -

Warfarin -

Clopidogrel -

58. contraindication for mitral valve replacement except :


Normal echo stress

:59. Pt presnted with lower GI bleeding, after resuscitationnext step

Tagged rbc test

Colonoscopy

Proctoscopy

:60. All are are true except

Sepsis is SIRS in presence of infection -

Septic shock is sepsis with hypotension -

leucocytosis or leuckopenia is sign if sepsis -

Temp of >38 or <36 is sign of sepsis -

tachycardia and tachypnea can be signs of sepsis -

:61. All are indications of surgical intervention Except

Second ipsilateral pneumothorax -

First contralateral pbeumothorax -

Bilateral pneumothorax -

spontaneous Hemothorax -

62. one is an indication for surgery:

Gastric content in chest tube

200 cc serosanguenous over 3 hours

1000 cc serous after insertion of chest tube

63. Wound healing questions

Does not increase wound infx

Microorganism virulence

Does not affect wound healing

Depth of wound

:64. Q about premature


premature have higher risk of infection -

premature requirrment of fluid s is higher -

oxygen index of 20 is indication for ECMO -

lung hypoplasia is the cause of death -

65. q about bilirubin one is false :

Found in urine as unconjugated form

66. all are true about deep partial thikness burn except :

Painfull

Blisters

67. About bone scan one is false

Same dose of ct

…..

68. Q about epidural and peipheral analgesia...fentanyl lipid soluable comaoring to


morphine

69. Intrnsic factor which is false

Polysacharid

Acts on terminal ilium

Need vit b12 injection every 2-3 months

70. Post cardiac cath abdominal.pain and bloody diarrhea

Mesentric ischemia

71. Question about branchial cleft cyst excision is dangerous because it is close
to

Ext carotid

Int carotid

Hypoglossal nerve

None of the above

All of the above

72. Q about gallstone pancreatitis best mangment


ERCP then lap cholecytectomy

Observation then lap chole in same admission

: 73. Most common cause of burn in child < 5 years old

Scald burn

Flame burn

Chemical

Inhalatiinal

:74. Patient presented with multiple trauma, GCS 7 ... Most important

xray -

ct scan -

enditracheal intubation -

75. CEA: not used for screening for colonic CA

:76. All are indication for sleeve except

Motivated -

Psychatric illness + -

77. desmoids tumer:

:78. Flial chest true except

consist of two consecutive ribs fracture at 2 different sites -

surgery is rarely indicated -

have paradoxical chest movment withvrespiration -

:79. Most commin indication for surgery in goiter

Cosmotic -

pt anxiety about mass -

:80. Fast scan is used to asses

retroperitoneal organs injury -

intraperitoneal otgans injury -


intraabdominal vascular injuy -

:81. Best initial test to evaluate pneumothorax

upright erect CXR -

lateral decubitus -

Expiratory film

:82. All are normal ABG readings except

Base 0 +-2

PH 7.35-7.45

PCO2 35-45

HCO3 32-36 +

83. Question about rectus sheath hematoma

All cases need surgery

….

:84. Question about linitus plastica all are true except

Answer : mucosal ulceration is a marked feature

85.

PM, 5/3/2017] Dr.salah 3'araybeh ORTHO: Question about linitus plastica all are true 4:20[
:except

Answer : mucosal ulceration is a marked feature

: PM, 5/3/2017] Dr.salah 3'araybeh ORTHO: Question about staging 4:21[

Answer clinical assesment of invasion and spread


MCQ for Residents
1.Hyperglycemia in a surgical patient receiving TPN may best be managed by

a. Oral hypoglycemic drugs

b Decreasing the dextrose load and doubling the amount of fat

c. Adding regulalar insulin to the TPN

e. Increasing concentration of protein and carbohydrate calories and decreasing that of

lipids

2.which amino acids can be metabolized outside the liver and are a local source of energy

For muscle?

a. Leucine, isoleucine, valine


b. Alanine, argenine, lysine.
c. Ethionone, glutamine, lysine.
d. Phenyalanine,tyrosine,histidine.
e. None of the above.
3. The gastrointestinal tract can secrete and absorb how much water in the form of
gastric juices per day in 70-kg adult male?
a. 1-2 L/day
b. 4-5 L/day
c. 6-7 L/day
d. 8-10 L/day
e. 50 L/day

4. The protein –sparing effect of glucose administration begin to be manifested after


administration of how much glucose?
a. 1 L of 5% dextrose in water (DƽW)
b. 2 L of (DƽW)
c. 3 L of(DƽW)
d. 4 L of (DƽW)
e. 5 of (DƽW).

5. TNF-α release :
a. can be effectively blocked by anti-TNF-α antibodies to halt systemic
inflammatory syndrome ( SIRS).
b. Doesn't have any beneficial effects in the early phases of the inflammatory
response.
c. Is primarily from leukocytes
d. Promotes polymorphonuclear (PMN) cell adherence and further
cytokines release.
e. Is always deleterious.
6. you suspect that a patient has ARF secondary to hypovolemia.
All of the following are appropriate initial treatment except:
a. check the hemoglobin level
b. Give intravenous boluses.
c. Start vasopressor infusion to keep mean arterial pressure greater than
65 mm Hg
d. Calculate the fractional excretion of sodium
e. Rule out causes of outflow obstruction.

7. Which one of the following suggest an acute adrenal crisis:


a. Random cortisole level of 34 mcg/dl
b. Hypothermia
c. Hyperglycemia
d. Hypokalemia
e. Increase cortisole of 5 mcg/dl after stimulation with cosyntropin

8. The syndrome of multi-organ-failure (MOF):

a. Involve sequential insults that lead to systemic hyperinflammation


b. Require the documentation of active infection
c. Has decreased in incidence over the past decade
d. Require diagnosis within 3 days of the systemic insult
e. Demonstrate consistent improvement after blood transfusion.

9. The systemic parameters in the definition of SIRS include all of the following except:
a. Temperature lower than 36ºC
b. Respiratory rate greater than 20 breath/mint
c. Paco2 less than 32 mm Hg
d. Systolic blood pressure lower than 90 mm Hg
e. Heart rate greater than 90 beats/mint.

10. Strategies that have been suggested to decrease the risk for postoperative
Pulmonary complications include all of the following except:

a. Routine nasogastric tube decompression


b. Lung expansion maneuvers
c. Preoperative smoking cessation.
d. Postoperative epidural anesthesis
e. use of intraoperative short-acting neuromuscular blocking agents.

11. All of the following are true concerning the sympathetic nervous system except:
a. Circulating epinephrine is produced mainly in the adrenal glands and secreted as a
hormone.
b. Most circulating norepinephrine is derived from synaptic nerve clefts.
C. Activation of the sympathetic nervous system result in vasoconstriction,
Tachycardia, and tachypnea.
d. Norepinephrine acts as neurotransmitter.
e. Up to 5% of norepinephrine and 15% of dopamine are produced by the enteric
nervous system.

12. Acute repiratory syndrome (ARDS), histologic examination of an alveolar biopsy


In the first 24 hours would demonstrate.
a. influx of protein-rich leukocyte.
b. Preservation of type II pneumocytes.
c. Bacterial colonization.
d. Alveolar hemorrhage.
e. High level of collagen and fibronectin.

13. which of the following statements regarding the inflammatory phase of the wound
healing.
a. it last up to 24 hours after the injury is incurred.
b. Initial vasodilation is followed by subsequent vasoconstriction.
c. Bradykinin causes vasoconstriction, which inhibits migration of neutrophils
to the healing wound.
d. The complement component C5a and platelet factor attract neutrophil to
the wound.
e. the presence of neutrophil in the wound is essential for normal wound healing.

14. Which of the following statements is true about growth factors.


a. Epidermal growth factor (EGF) stimulates the production of collagen.
b. Vascular endothelial growth factor (VEGF) and PDGF both stimulate angiogesis
by binding to a common receptor.
c. Fibroblast growth factor (FGF) stimulate wound contracture.
d. Transforming growth factor-B (TGF-B) is stored in endothelial cells.
e. Tumor necrosis factorα (TNF-α) inhibits angiogenesis.

15. Which of the following statements regarding scar revision is true?


a. scar maturation refers to the change in size of the wound in the first 1-2 months
b. scar revision should have been performed in the first 3 months after injury
to minimize fibrosis.
c. revision should be performed earlier in children than in adult
d. It corrects undesirable pigmentation
e. Scar revision should be delayed approximately 1 year to allow maturation.

16. Which of the following statements regarding wound healing is true?


a. Vitamin A is needed for hydroxylation of lysine and proline.
b. High doses of vitamin C improve wound healing
c. Vitamin E is involved in the stimulation of fibroplasias, collagen cross-linking.
d. Zinc deficiency results in delayed early wound healing.
e. Iron deficiency had been linked to defects in long-term wound healing.
17 . Which portion of the cell cycle in actively dividing cells is most sensitive to
Ionizing radiation?
a. S phase
b. M phase
c. G1 phase
d. G2 phase
e. All phases are equally radiosensitive.

18. Which of the following statements regarding chromosome is not true?


a. The nucleus contains the entire cellular DNA
b. Histones compact and organize the DNA strands.
c. interactions between DNA and proteins expose specific genes and control
their expression.
d. During mitosis, the spindle apparatus attaches to the chromosome at centromere.
e. Telomeres maintain chromosomal length replication cycles .

19. In DNA replication, what type of mutation is specifically associated with generation
Of a stop codon?.
a. Point mutation
b. Missense mutation.
c. Nonsense mutation
d. Frameshift mutation
e. Neutral mutation.

20. Which of the following condition is associated with an isolated prothrombin time
(PT) prolongation.

a. Von willebrand disease.


b. Factor VIII deficiency ( hemophilia).
c. Common pathway deficiency ( factor II, V, X, and fibrinogen).
d. Therapeutic anticoagulation with warfain
e. Therapeutic anticoagulation with heparin.

21. Regarding complication of blood transfusion, which is true?


a. Febrile reaction is rare.
b. Gram-positive organisms are the most common contaminants of stored blood.
c. Screening for minor antigens should be repeated every week when multiple
transfusions are given.
d. A small amount (more than 0.1 ml air) is well tolerated.
e. Malaria, chagas disease,( HTL-V-1 virus,) ( AIDS) and hepatitis can be transmitted
by blood transfusion.

22. which of the following statements is true regarding the stored blood?
a. Packed RBC stored in additive solution (AS-3) and kept at 4ºC are suitable
for transfusion for 3 months.
b. Platelets in banked blood retain their functions for 3 days.
C. Factors II, VII, IX, and XI are stable at 4ºC.
d. A decrease in RBC oxygen affinity occurs during storage as a result of
a decrease in 2,3-DPG levels.
e. There a significant rate of hemolysis in stored blood.

23. Which of the following statements regarding daily fluid balance is incorrect?
a. Daily water intake is 2000-2500ml.
b. Average stool loss, 1000ml.
c. Average insensible loss, 600ml.
d. Average urine volume, 800ml-1500ml.
e. Average increase in insensible loss in febrile patient, 250 ml/day for each degree
of fever.

24. Which of the following statements concerning Na +concentration of various fluids


Is incorrect.
a. pancreatic secretion, 140 mEq/L
b. Sweat, 40 mEq/L
C. Gasric secretion, 50 mEq/L
d. Saliva, 100mEq/L
e. Ileostomy output, 125 mEq/L.

25. Which of the following condition is not associated with hypernatremia?


a. Diabetes insipidus
b. Tumor lysis syndrome.
c. Steven-Johnson syndrome.
d. Primary hypodepsia.
e. Enterocutaneous fistula.

26. Which of the following clinical signs or symptoms is not associated with serum Na+
Concentration below 125 mEq/L?
a. Headache.
b. Hallucination.
c. Bradycardia
d. Hypoventilation
e. Hyperthermia

27. Which of the following (ECG) findings is not associated with hyperkalemia?
a. Peaked T waves
b. Prolong PR interval
c. Loss of P wave
d. Narrowing of QRS complex.
e. T waves higher than R waves in more than one level.
28. Which of the following is least useful in the immediate treatment of hyperkalemia?
a. Calcium salts.
b. NaHco3
c. Potassium-binding resins.
d. Glucose and insulin
e. Hemodylesis

29. With regard to hypokalemia, which of the following statements is not true?.
a. K+ and H+ are exchanged for Na+ in renal tubule.
b. Respiratory acidosis is associated with increase renal K+ loss.
c. hypokalemia can cause decrease deep tendon reflexes.
d. Flattened T waves and a prolonged QT interval are associated with hypokalemia.
e. Intravenous K+ administration should not exceed 40 to 60 mEq/hour.

30. Which of the following clinical scenario is not associated with acute hypocalcemia?
a. Fluid resuscitation from shock.
b. Rapid infusion of blood products.
c. Improper administration of phosphates.
d. Vitamin D-deficient diets.
e. Acute pancreatic.

31. with regard to acid-base buffering. Which of the following is false?


a. The major extracellular buffer is Hco3
b. Intracellular PH and extracellular PH are usually the same.
c. The major intracellular buffer consists of protein and phosphate salts.
d. H+ cannot directly pass though cell membrane.
e. Treating acidosis with Hco3 infusion can cause cell death.

32. Which of the following statement is not correct with regard to cardiac output(CO)
a. CO alone is not an indicator of myocardial contractility.
b. Ventricular end-Diastolic volume (EDV),vascular resistance, and myocardial
contractility determine stroke volume (SV).
c. Arterial blood pressure alone is an accurate indicator of CO.
d. CO varies directly with pulse rate of up to 160 beats/min in sinus rhyhtm, after
which it decrease.
e. Atrial contraction contributes up to 30% of EDV.

33. With regards to ventilator mechanics, which of the following statements is false?.
a. The work of breathing at rest consume 2% of total oxygen consumption.
b. COPD is associated with increase in the work breathing as a result of increase
inspiratory work.
c. The work of breathing may increase to 50% of total oxygen consumption in
postoperative patients.
d. Airway pressure reflects the compliance of the chest wall and diaphragm
as well as that of lungs.
e. Comliance is measured as the change in volume divided by change in pressure.

34. With regard to Multi organ Failure(MOF). Which of the following statement is
false?.
a. Sepsis is the major risk
b. Injury to the microvascular endothelium is uniformly present.
c. Neutrophil-mediated injury is dependent on adherence to the microvascular
endothelium .
d. There is a bimodal pattern to the development of MOF.
e. An increase in the gastrointestinal barrier is often present.

35. Regarding pelvic , fracture which is true?


a. mortality rate is 30% if hypotension present.
b.A pubic symphysis diastasis of 3 cm double the volume of potential pelvic space
for hematoma.
c. It is imperative to make large surgical incision from xiphoid to symphysis to allow
better visualization of the pelvis.
d. The most common cause of death in pelvic fracture is overwhelming sepsis.
e. The external iliac artery is commonly involved in severe pelvic fracture.

36. Select the correct statement regarding flail chest.


a. It occur when three or more adjacent ribs are fractured.
b. work of breathing is increased secondary to paradoxical chest wall infection.
c. Patients with flail chest should be aggressively resuscitated because of probable
development of pulmonary contusion.
d. Patients with this condition should be prophylactically intubated secondary
to a high likelihood of respiratory failure.
e. If a patient doesn't require mechanical ventilation , it is important to avoid
use of positive end-expiratory pressure.

37. A 25 year old male, sustained flame burn, on his right arm circumferentially,
Bilateral on his legs, and on perineum. What the approximate % TBSA?
a. 28%.
b. 36%
c. 46%
d. 64%
e. 52%

38. Which of the following is a potential premalignant precursor of melanoma?


a. Keratoacanthoma
b. Actinic keratosis
c. Seborrheic keratosis
d. dysplastic nevus
e. Bowen disease.
39. The preferred diagnostic biopsy method for pigmented skin lesion is:
a. Punch biopsy
b. Incisional biopsy
c. Shave biopsy
d. Excisional biopsy
e. Excision with 0.5 –cm margin.

40. The most common histologic type of melanoma is :


a. Superficial spreading
b. Nodular
c. Lentigo
d. Acral lentiginous
e. Desmoplastic

41. Which of the following statements is false regarding direct inguinal hernias?
a. The most likely cause is destruction of connective tissue as a result of physical
stress.
b. Direct hernia should be repaired promptly because of the risk of incarceration.
c. A direct hernia may be a sliding involving apportion of the bladder wall.
d. A direct hernia may pass through the external ring.
e. An indirect hernia may present as well.

42. Inguinal hernia:


a. Are best treated with laparoscopic technique.
b. Should always be repaired
c. Benefit from the use of mesh in reducing recurrence.
d. Are associated with a high incidence of strangulation.
e. Are rare in infants.

43.Which of the following is not a characteristic of medullary breast cancer


a. Lymphocystic infiltrate.
b. Benign appearance on ultrasound.
c. High rate of lymph node metastasis
d. statistically better than average prognosis
e. Usually manifest as a palpable mass.

44. Which of the following is not an indication for postmastectomy radiotherapy?


a. T3 Tumors
b. Multicentric DCIS larger than 6 cm
c. inflammatory breast cancer.
d. four or more positive axillary lymph nodes.
e. Gross extranodal extension.

45. with regars to phyllodes tumor of the breast, which statement is not true?
a. It is histologically characterized by epithelial cystlike spaces.
b. Examination reveals a firm, moble, well circumscribed mass.
c. 10% to 15% are malignant
d. The benign version can grow aggressively and recur locally.
e. It commonly metastasizes to lymph nodes.

46. Regarding molecular marker in breast cancer, which of the following


Statements is correct?
a. Carrier of BRCA2 mutations are more likely to have triple –negative cancer.
b. HER2-positive cancers are unlikely to respond to treatment with trastuzumab.
c. ER-positive/HER2-negative patient should be treated by endocrine therapy
d. All breast cancer are sensitive to endocrine therapy.
e. Triple –negative patients have an excellent prognosis.

47. Which of the following is associated with appearance of invasive lobular carcinoma
On mammography?
a. Discrete bilateral mass
b. partially cystic appearance
c. Indistinct mass with poorly defined border.
d. mass with microcalcification.
e. Branching pleomrphic calcification.

48. what is percentage of false negative rate for sentinel lymph node biopsy(SLNB)?
a. 1%
b.8%
c. 15%
d. 20%
e. 25%

49. With regard to thyroid anatomy, which of the following statements is incorrect?
a. The inferior thyroid artery arise directly fro the external carotid artery.
b. the thyroid ima artery arises directly from the aorta in 3% of cases.
c. The ligament of berry is located near the entry point of RLN.
d. venous drainage of the thyroid gland is via superior, middle, and inferior branches.
e. The superior and middle thyroid vein drain into the jugular vein.

50.Which of the following statements regarding Hurthle cell carcinoma is incorrect?


a. It represent a subtype of follicular carcinoma
b. Hurthle cell carcinoma account for 3% of all thyroid cancer.
c. It is more likely to be multifocal
d. It demonstrate poor radioactive iodine.
e. Lymph node dissection is indicated for all patients.
 Crohn's disease:
A. Is caused by Mycobacterium paratuberculosis.
B. Is more common in Asians than in Jews.
C. Tends to occur in families.
D. Is less frequent in temperate climates than in tropical ones.
E. Is improved by smoking.
(c )

 A 15-year-old boy is admitted with a history and physical findings consistent with
appendicitis. Which finding is most likely to be positive?

 A Pelvic crepitus

 B Iliopsoas sign

 C Murphy sign

 D Flank ecchymosis

 E Periumbilical ecchymosis

 (b )

 A 50-year-old man is admitted with massive bright red rectal bleeding. He recently
had a barium enema that demonstrated no diverticular or space-occupying lesion.
Nasogastric suction reveals no blood but does produce yellow bile. The patient
continues to bleed. What is the next diagnostic step?

 A Repeat barium enema

 B Colonoscopy

 C Upper gastrointestinal series

 D Mesenteric angiography

 E Small bowel follow-through with barium

 (D )

 Recurrence after operation for Crohn's disease:


A. Occurs after operations for ileal Crohn's but not colonic Crohn's.
B. Is usually found just proximal to an enteric anastomosis.
C. Rarely requires reoperation.
D. Occurs in 1% of patients at risk per year during the first 10 years after the
operation.
E. Is prevented by maintenance therapy with corticosteroids.
(B )
 A 15-year-old boy awakens with sudden onset of right lower quadrant and scrotal
tenderness accompanied by nausea and vomiting. Which of the following is the most
appropriate diagnosis and represents a surgical emergency?

 A Acute prostatitis

 B Acute epididymitis

 C Torsion of the testicle

 D Acute appendicitis

 E Gastroenteritis

 (C )

 Factors that decrease collagen synthesis include all of the following except:
 A. Protein depletion.
 B. Infection.
 C. Anemia.
 D. Advanced age.
 E. Hypoxia.
 (C)

 A 47-year-old woman presents with dysphagia to both solids and liquids equally. She
has experienced a 10-kg weight loss over the last several months. A barium swallow
reveals a birdlike narrowing in the distal esophagus. What is the underlying cause of
her symptoms?

 A Disorganized, strong nonperistaltic contractions in the esophagus

 B Failure of the lower esophageal sphincter to relax

 C Hiatal hernia

 D Barrett's esophagus

 E Esophageal stricture secondary to untreated gastroesophageal reflux

 (B)

 A 45-year-old male executive is seen because he is vomiting bright red blood. There
are no previous symptoms. The man admits to one drink a week and has no other
significant history. In the hospital, he bleeds five units of blood before endoscopy.
What is the most likely diagnosis?

 A Gastritis

 B Duodenal ulcer

 C Esophagitis

 D Mallory-Weiss tear
 E Esophageal varices

 (B)

 45-year-old man is seen in the emergency department after vomiting bright red blood.
He has no previous symptoms. He drinks one alcoholic beverage a day.

 What is the most reliable method for locating the lesion responsible for the bleeding?

 A Upper gastrointestinal series

 B Exploratory laparotomy

 C Upper endoscopy

 D Arteriography

 E Radionuclide scanning

 (c)

 After several hours in the hospital, he begins to have recurrent bleeding. He is


transferred to a critical care bed and is persistently hypotensive despite trasnfusion of
nine units of packed red blood cells. Which is the most appropriate next step in
management of this patient?

 A Upper endoscopy with attempt at cauterization of bleeding

 B Transport to the interventional radiology unit to identify and embolize bleeding


source

 C Placement of a Blakemore tube to temporarily tamponade bleeding and to allow for


stabilization of blood pressure

 D Laparotomy to control bleeding

 E Infusion of vasopressin and additional units of blood

 (D)

 A 25-year-old man is admitted with a history of sudden onset of severe midepigastric


abdominal pain. Upright chest radiograph reveals free intraperitoneal air. What is the
therapy for this patient?

 A Upper endoscopy

 B Barium swallow

 C Gastrografin swallow

 D Observation

 E Laparotomy

 (E)
 Concerning severe pancreatitis:

 A. Hypocalcaemia is the most common metabolic problem

 B. Coagulopathy is usually the first organ system failure to manifest itself

 C. Failure of two organ systems is associated with 90% mortality

 D. Solid, infected pancreatic necrosis will often respond to intravenous


antibiotics

 E. Positive end expiratory pressure (PEEP) may be useful in managing


respiratory failure

 (E)

 A 55-year-old female patient is evaluated for new onset of diabetes mellitus. Her
medical history is largely unremarkable. Her physical examination is unrevealing
except for the presence of an erythematous skin rash. Her further evaluation should
include an investigation of the possibility of which of the following?

 A Insulinoma

 B Glucagonoma

 C Gastrinoma

 D Carcinoid tumor

 E Pancreatic cholera

 (B)

 A 60-year-old female patient has a workup for episodic symptoms of palpitations,


nervousness, and bizarre behavior, all of which tend to occur during fasting states.
Biochemically, she is diagnosed as having an insulinoma. What is the best choice for
localizing this tumor?

 A CT scan

 B MRI

 C Selective arteriography

 D Percutaneous catheterization of the portal vein with selective venous sampling

 E Surgical exploration and intraoperative ultrasound

 (E)

 3. A 55-year-old woman with progressive but episodic muscle weakness is diagnosed


as having myasthenia gravis. Her chest radiograph is normal and reveals no evidence
of mediastinal mass or tumor. What is the most definitive treatment that can be
offered this patient?

 A Prednisone

 B Neostigmine

 C Thymectomy

 D Plasmapheresis

 E Atropine

 (c)

 A 50-year-old hypertensive man has definitive biochemical evidence of a


pheochromocytoma. Computed tomography (CT) scan and magnetic resonance
imaging (MRI) do not reveal any abnormalities, and m-iodobenzylguanidine scanning
is not readily available. What should be the next step in the management of this
patient?

 A Abdominal exploration

 B Continued clinical observation

 C Mediastinoscopy

 D Selective venous sampling

 E Mediastinal exploration

 (D)

 The hemodynamic value characteristic of septic shock is


 A Cardiac index, 2.8 L/min/m2
 B Systemic vascular resistance index, 350 dynes cm-5 sec/m2
 C Oxygen consumption, 135 mL/min/m2
 D Oxygen delivery, 700 mL/min/m2
 E Pulmonary capillary wedge pressure, 6.0 mm Hg
 (c)
1. In the workup on a patient for possible appendicitis, CT scanning
should be performed:
a) Before consulting the surgeon, by the emergency physician
b) In patients with equivocal physical findings
c) Routinely, in all patients with right lower quadrant pain
d) With equal frequency in men and women
e) Never

b—

2. Acute appendicitis

a. Occurs most commonly in the second and third decades of life


b. Can be cured readily by antibiotics
c. Is most commonly caused by a fecalith
d. Carries an overall mortality rate of 7%
e. Induces leukocytosis in 90% of patients

3. The gastric mucosal cell that secretes intrinsic factor is the


a) G cell
b) Parietal cell
c) D cell
d) Enterochromaffin-like cell
d) Chief cell

b—

4. Hypertrophic pyloric stenosis is likeliest to occur in a(n)


a) Firstborn child
b) African-American infant
c) Child 6 to 9 months of age
d) Female infant
e) Infant born prematurely

a—

5. Overwhelming postsplenectomy infection (OPSI) a) Occurs more


frequently after resection for trauma than hematologic disease
b) Occurs with equal frequency in children and adults
c) Is most frequently caused by Streptococcus pneumoniae
d) Usually occurs within 2 years after splenectomy
e) Generally has an identifiable site of infection

c—

6. The characteristic feature of Crohn's colitis that best distinguishes


the clinical entity from ulcerative colitis is
a) Perianal disease
b) Rectal bleeding
c) Risk of malignancy
d) Obstructive symptoms
e) Pseudopolyps

a—

7. Type 1 gastric ulcers are


a) At the incisura
b) Along the greater curvature
c) Prepyloric
d) Associated with simultaneous duodenal ulcers
e) Close to the esophagogastric junction

a—

8. Radiation enteritis
a) Usually presents with perforation
b) Is caused by thrombosis of mucosal vessels
c) Occurs after 3,000 cGy of abdominal radiation
d) Routinely requires operative therapy
e) Is likely in patients who have undergone laparotomy

e–
9. The small bowel tumor with the greatest propensity for bleeding is
a) Carcinoid
b) Lymphoma
c) Adenocarcinoma
d) Hamartoma
e) Leiomyoma

e—

10. A grade III laceration of the spleen is characterized by

a. Intraparenchymal hematoma > 5 cm


b. Capsular tear 1-3 cm deep
c. Hilar vessel disruption
d. Subcapsular hematoma involving 10%-50% of surface area
e. Active hemorrhage

a—

11. The spleen filters all of the following particles/cells EXCEPT

a. Malformed erythrocytes

b. T lymphocytes
c. Malarial parasites

d. Streptococcus pneumoniae

e. Platelets

b—

12. which of the following is the strongest of all other risk factors in
the development of Gastric carcinoma :

A. Helicobacter pylori.
B. Atrophic gastritis.
C. Blood group A.
D. Pernicious anemia.
E. Low socioeconomic class.

13. whilst performing a small bowel resection for strictures following


crohn”s disease, you realize that on inspection, there are marked
differences between jejunal and ileal anatomy. Such differences
include all the following Except:

A. Wider lumen in the jejunum.


B. Less lymphatics in the jejunal mesentry compared to ileal.
C. More prominent and multiple arcades of vessels in the ileum.
D. Thicker wall of the jejunum.
E. Thicker and more fat-laden mesentry increasing towards the ileum.

D
14. A 54 years old woman is referred to your surgical team with a
diagnosis of small bowel obstruction. Which one of the following
clinical signs would you look for in trying to identify the commonest
cause of this condition :

A. surgery scar.
B. Lump in the groin above & medial to the pubic tubercle.
C. Lump in the groin below & lateral to the pubic tubercle.
D. Cachexia & nodule at the umbilicus.
E. Circumoral pigmentation & a family history of previous obstruction.

15. True statements regarding appendiceal neoplasms include which


of the following?

A. Carcinoid tumors of the tip of the appendix less than 1.5 cm


are adequately treated by simple appendectomy
B. Appendiceal carcinoma is associated with secondary tumors of the GI
tract in up to 60% of patients
C. Survival following right colectomy for a Dukes’ stage C appendiceal
carcinoma is markedly better than that for a similarly staged colon
cancer at 5 years
D. Mucinous cystadenocarcinoma of the appendix is adequately treated by
simple appendectomy, even in patients with rupture and mucinous
ascites
E. Up to 50% of patients with appendiceal carcinoma have metastatic
disease, with the liver as the most common site of spread

:A

16. All of the following statements about carcinoma of the


gallbladder are correct Except :

A. The neoplasm usually starts in the cystic duct and neck of the
gallbladder.
B. It is found more commonly in women than men.
C. It is associated with the presence of gallstones in > 85% of cases.
D. Prognosis is generally poor with < 1 year survival with local invasion.
E. Chemotherapy and radiotherapy do not alter disease progression.

17. All of he following statements are true regarding diverticular


disease Except :

A. It is found more commonly in the developed world.


B. Surgical treatment is usually unnecessary in acute uncomplicated cases.
C. Diverticulae are more commonly found in the descending colon.
D. Perforation and fistula formation can result from an attack of acute
diverticulitis.
E. Resolution of the diverticulae can occur with high fiber diet and
adequate hydration.

18. All of the following arteries are branches of the superior


mesenteric artery, except:

a. Ileocolic
b. Replaced left hepatic
c. Inferior pancreaticoduodenal
d. Jejunal
e. Replaced right hepatic

b—

19. Mesenteric cysts

a. Occur primarily in the mesocolon


b. Are discovered mainly in children
c. Can contain chyle or serous fluid
d. Require total enucleation for a cure
e. Are neoplastic
c—

20. Decompression for abdominal compartment syndrome should be


performed

a. If the urine output falls to 30 mL/hour


b. When the patient's respiratory rate increases to 24 breaths per
minute
c. Based purely on physical findings
d. When bladder pressure exceeds 35 mm Hg
e. If the patient becomes hypoxemic

d—

21. In laparoscopic surgery, the vessel most likely to be punctured


during trochar placement is the

a. Aorta
b. Right common iliac artery
c. Vena cava
d. Left common iliac artery
e. Right common iliac vein

b—

22. Following resolution of acute appendicitis, an interval


appendectomy

a. In 90% of interval appendectomies no pathologic


abnormality could be found.
b. Always reveals luminal occlusion of the appendix
c. Should be performed because of the high incidence of recurrent
appendicitis
d. Is not indicated in patients older than 40 years
e. Needs to be performed open because of the fibrosis induced by
appendicitis

a
23. True statements concerning the diagnosis and management of
retroperitoneal fibrosis include all of the following except:

A. Most patients present with dull, non-colicky back, flank, or abdominal pain
B. Evidence of impaired renal function with an elevated blood urea nitrogen
is common
C. The diagnosis is most commonly suggested by intravenous pyelography
although contrast studies with CT scan or MRI are useful in further
defining the disease
D. Most patients will need operative intervention.
E. The prognosis for nonmalignant retroperitoneal fibrosis is grim
with progression of disease until death occurring in most
patients

:E

24. Lynch Syndrome all are true except?

a) It is due to mutation in MMR gene

b) Associated with APC gene

c) It has Autosomal Dominant Inheritence

d) Adenomas in patients with Lynch syndrome display high grade dysplasia


than adenoma in patients with sporadic colorectal cancer?

e) It occurs predominantly on right side, has increased incidence of


synchronous and metachronous disease.

25. Which of the folowing about Pancreatic Ascites is not true

a) Conservative treatment effective in only 1/4th of patients

b) ERCP should be done before surgery

c) It is exudative

d) Metaplastic cells are present


e) Pancreatic fluid has high amylase and high albumin

26. Tetanus all are true except?


a. Is due to an infection with a gram-negative spore forming rod
b. The organism produces a powerful exotoxin
c. The toxin prevents the release of inhibitory neurotransmitter
d. Clostridium tetani is sensitive to penicillin
e. Risus sardonicus is the typical facial spasm

A
27. Which of the following statements about the surgical treatment
of esophageal carcinoma is/are correct?

A. The finding of severe dysphagia in association with Barrett's mucosa is an


indication for an antireflux operation to prevent subsequent development
of carcinoma.
B. Long-term survival is improved by radical en bloc resection of the
esophagus with its contained tumor, adjacent mediastinal tissues, and
regional lymph nodes.
C. The morbidity and mortality rates for cervical esophagogastric
anastomotic leak are substantially less than those associated
with intrathoracic esophagogastric anastomotic leak.
D. The leading complications of transthoracic esophagectomy and
intrathoracic esophagogastric anastomosis are bleeding and wound
infection.
E. Transhiatal esophagectomy without thoracotomy achieves better long-
term survival than transthoracic esophagectomy.

Answer: C

28. The most effective therapy for morbid obesity, in terms of weight
control, is:
A. Intensive dieting with behavior modification.
B. A multidrug protocol with fenfluramine, phenylpropanolamine, and
mazindol.
C. A gastric bypass with a 40-ml. pouch, a 10- to 20-cm. Roux-en-Y
gastroenterostomy.
D. A gastric bypass with a 15-ml. pouch, a 40- to 60-cm. Roux-en-Y
gastroenterostomy.
E. Daily exercise with strong emphasis on utilizing all four limbs.

Answer: D

29. All of the following statements about the embryology of Meckel's


diverticulum are true except:

A. Meckel's diverticulum usually arises from the ileum within 90 cm. of the
ileocecal valve.
B. Meckel's diverticulum results from the failure of the vitelline duct to
obliterate.
C. The incidence of Meckel's diverticulum in the general population
is 5%.
D. Meckel's diverticulum is a true diverticulum possessing all layers of the
intestinal wall.
E. Gastric mucosa is the most common ectopic tissue found within a Meckel's
diverticulum.

Answer: C
1-To maintain a normal hydrogen balance, total daily excretion of H+
should equal the daily: -
a) Fixed acid production plus fixed acid ingestion.
b) HCO3- excretion
c) Pseudomonas aeruginosa.
d) S.Fecales.
e) Staph. Aureus.

2- A 45-year-old obese man undergoes subtotal gastrectomy for an antral


carcinoma. On the fifth postoperative day, non-foul-smelling
serosanguineous drainage appears from the wound. All the following may
have been contributing factors EXCEPT
a) malnutrition
b) atelectasis
c) anemia
d) poor technique
e) ascites

3- A 32-year-old man is admitted to a hospital where he undergoes


exploratory surgery for a gunshot wound of the left colon. Subsequently,
he is maintained on intravenous hyperalimentation; his vital signs are
stable, and his urine output averages 250 mL/h. On the fourteenth hospital
day, he is reexplored under general anesthesia for drainage of a left
subphrenic abscess. Parenteral nutrition is continued during surgery. In
the recovery room 6 h later, the patient is stable and his urine output is
brisk, but he remains comatose with spontaneous respirations.
The most likely cause for the patient's delayed emergence from
anesthesia is
a) hypoglycemia
b) nonketonic, hyperosmolar coma
c) partial curarization
d) volume overload
e) hypoventilation and hypoxemia

4- A patient with a non obstructing carcinoma of the sigmoid colon is


being prepared for elective resection. To minimize the risk of
postoperative infectious complications, your planning should include
a) A single preoperative parenteral dose of antibiotic effective against
aerobes and anaerobes
b) Avoidance of oral antibiotics to prevent emergence of Clostridium
difficile
c) Postoperative administration for 2–4 days of parenteral antibiotics
effective against aerobes and anaerobes
d) Postoperative administration for 5–7 days of parenteral antibiotics
effective against aerobes and anaerobes
e) Operative time less than 5 h.

5- Regarding Parathyroid cancer, which one of the following statements


is true
a) Is a frequent cause of hyperparathyroidism
b) Most patients present with hypercalcemia and a palpable mass

c) The histopathological criteria to differentiate a parathyroid adenoma


and parathyroid carcinoma are well defined
d) It is associated with multiple endocrine neoplasia type 1
e) Localizing studies are mandatory before neck exploration.

6- The rational for Endoscopic management of cystic duct leak after


laparoscopic cholecystectomy is: -
a) Covering the cystic duct-common bile duct junction with plastic stent
to prevent further leak.
b) Increase in the transpapillary pressure gradient by papillotomy.
c) Promoting flow and reducing the amount of bile flowing through the
leak by placing a plastic stent and also performing a papillotomy.
d) The reduction in the transpapillary pressure gradient by insertion of a
temporary stent.
e) Promoting flow and reducing the amount of bile flowing through the
leak by placing a naso- biliary drain.

7- If tacks/staples are placed to affix the mesh during laparoscopic


inguinal hernia repair, they should NOT be placed:-
a) Into Cooper's ligament.
b) Inferior and lateral to the internal inguinal ring.
c) Superior to the inguinal ligament In to the posterior aspect of the
rectus muscle.
d) Superior and medial to the internal inguinal ring.
e) At the pubic bone.
8- What are the early signs and symptoms of gastrojejunostomy leak in
obese patients?
a) Fever.
b) Tachycardia
c) Peritonitis.
d) Oliguria.
e) Hypotension.

9- A 33-year-old man underwent laparoscopic Roux-en-Y gastric byoass


15 months prior to his presentation to an emergency room for sudden
onset of abdominal pain. He has no other health problems. The pain
extends "like a band" across his mid-abdomen and causes him to bend
over. Abdominal exam demonstrates minimal tenderness in the left upper
quadrant of the abdomen and no signs of peritonitis. Lab tests including
complete blood count and comprehensive chemistry panel are normal.
What is the most likely cause for his pain?
a) Anastomotic chronic stricture.
b) Internal herniation.
c) Marginal ulcer.
d) Late dumping syndrome.
e) Gastrointestinal leak.

10- The most common complication of PEG ( Percuatnious Endoscopic


Gastrostomy) placement is?
a) Bleeding
b) Infection
c) Peritonitis
d) Gastro-colic fistula
e) Tube dislodgment

11- About colorectal carcinoma associated with Crohn’s disease, one is


TRUE:
a) It is usually common in women.
b) The frequency of carcinoma is similar in patients with extensive, long-
standing ulcerative colitis.
c) The right colon is involved in over 70% of patients.
d) The mean age of patients with colorectal carcinoma is 35.
e) The occurrence of carcinoma is unrelated to the duration of Crohn’s
disease.

12- Prolonged absence of enteral nutrition during critical illness causes all
of the following changes in the small intestine EXCEPT:
a) Decrease villous height and cellular mass.
b) Reduction in the production of brush border enzymes.
c) Decrease nutrition absorption.
d) Increased translation in gut bacteria.
e) Irreversible changes in mucosal absorption.

13- Two yrs after resection of a T2N0M0 colon cancer, a chest X-ray
reveals a solitary peripheral pulmonary mass, chest and abdominal CT
confirms an isolated pulmonary lesion without other abnormalities.
Which of the following statements is true?
a) Staging with PET is not indicated
b) Percutaneous biopsy is not indicated because the risk of intrathoracic
spread.
c) A normal serum carcinoembryonic antigen (CEA) level excludes
metastatic disease.
d) Resection is indicated.
e) Primary treatment should be Cytotoxic chemotherapy rather than
resection.

14- A 35 year old female patient with BMI of 48 who complains of


obesity:-
What comorbidities might you expect NOT to improve with weight
loss?
a) Type II diabetes.
b) Hypertension.
c) Gastroesophageal reflux.
d) Cholesterol.
e) COPD.
15- A smooth, rubbery 3 cm lesion is removed from the breast of a 35-
year-old woman with a preoperative diagnosis of fibroadenoma.
Histologically, this lesion is found to be a phyllode tumor. Appropriate
management at this time is
a) observation only.
b) reexcision of the area with a 1 cm margin.
c) total (simple) mastectomy.
d) total mastectomy with axillary dissection.
e) tamoxifen therapy.

16- Which is true regarding operations for morbid obesity?

a) Bacterial overgrowth in the bypassed segment is a complication of


jejunoileal bypass b) Dumping symptoms are greater following a
vertical banded gastroplasty than after a Roux-en-Y gastric
bypass
c) Following Roux-en-Y gastric bypass, the patient may be permitted to
resume normal eating habits
d) Long-term weight loss is not sustained after a jejunoileal bypass
operation
e) Ulceration in the bypassed antrum has been a problem after Roux-en-
Y gastric bypass operations

17- Colorectal pseudo-obstruction has been associated with all of the


following EXCEPT
a) excess parasympathetic tone
b) malignant infiltration of the celiac plexus
c) neuroleptic medications
d) opiate usage
e) severe metabolic illness

18- During brief endoscopic abdominal procedures, the largest reserve of


body buffers to absorb CO2 is found in
a) bone
b) kidney
c) liver
d) lung
e) straited muscle

19- Abdominal insufflation to an intraabdominal pressure of 15 mmHg


produces all of the following effects EXCEPT
a) decreased cardiac output
b) decreased glomerular filtration rate
c) increased left heart filling pressure
d) increased peak inspiratory pressure
e) increased right heart filling pressure

20- Regarding surgery for ulcerative colitis all are true EXCEPT:-
a). 30% patients with total colitis will require surgery within 5 years.
b). Panproctocolectomy and pouch formation is appropriate as an elective
operation.
c). Pouches can be fashioned as 'S' 'J' or 'W' loops.
d). Over 90% patients with a pouch have perfect continence.
e). With a pouch the mean stool frequency is about 6 times per day.

21- Six weeks after undergoing a laparoscopic Roux-en-Y gastric bypass,


a 42-year-old man develops intolerance of solids and most liquids. Pre-
operatively, he weighed 135 kg and he had a BMI of 45 kg/m2. His
current weight is 114 kg and his BMI is 38 kg/m2. Consumption of a
three-ounce meal leads to dysphagia and vomiting. He is referred for an
upper endoscopy. The endoscopic can’t go through the gastro-
jejunostomy.
Which of the following is the most appropriate therapy?
a) Nasogastric decompression of the gastric pouch and proximal small
intestine.
b) Take down of the gastric bypass with small bowel resection and redo-
gastrojejunostomy.
c) Aggressive fluid resuscitation with normal saline.
d) Dilation of the gastrojejunostomy anastomosis followed by observation
and supportive care.
e) Administrations of a proton pump inhibitor and antibiotic therapy for
possible Helicobacter pylori with repeat endoscopy in six weeks

22- After insertion of the veress needle, ways to assume correct position
of the needle include:
a) Aspirate with a syringe to see what returns.
b) Inject 5cc saline and try to immediately get the fluid back.
c) Leave a drop of saline on the hub of the needle and see if it flows
freely into the abdomen.
d) Hook up the insufflator and see if low pressure flow results.
e) All of the above methods can, and usually should, be used.

23-Concerning base skull fracture of the anterior fossa all of the


following is true except:
a) Epistaxis.
b) CSF rhinorrhea.
c) Blindness.
d) Raccon eyes.
e) Battle sign.

24- One of the followings is true regarding Undescended testes:


a) Incidence is 1 in 10 000 boys
b) Most common location is the femoral region
c) Is associated with high rate of infertility
d) Orchidopexy is indicated at five years age
e) Increased risk of malignancy

25- Persistent, conjugated hyprebilirubinaemia may be caused by all of


the followings, EXCEPT:-
a) Alpha-1- antitrypsin deficiency
b) Hypothyroidism
c) Hemolytic disease
d) Cytomegalovirus infection
e) Cystic fibrosis

26- Neoadjuvant combined chemotherapy and radiation therapy in rectal


carcinoma:
a) Is not effective in down staging tumors and should not be considered in
patients with fixed lesion.
b) Has been proven to be effective in patients with tumors less than 3 cm
in diameter.
c) Should be followed by definitive operation in eight to ten weeks.
d) Is well tolerated by most patients but often result in higher operative
morbidity.
e) Decrease local recurrence rate when compared with postoperative
radiation therapy.
27- About gastric lymphoma one is TRUE:
a) Gastrointestinal bleeding is the most common symptom.
b) Endoscopic biopsy establishes the diagnosis in all cases.
c) Primary therapy is surgery.
d) Primary therapy is radiation.
e) The long term survival is similar to the adenocarcinoma.

28- In patients with carotid artery disease:-


a) A bruit is a reliable sign of the degree of stenosis.
b) Atheroma is most commonly seen in the external carotid artery.
c) An embolic event often results in an ipsilateral hemiplegia.
d) Prophylactic aspirin reduces the risk of a stroke.
e) Surgery is of proven benefit in those with asymptomatic stenoses.

29- Which of the following statements regarding malignant hyperthermia


is TRUE?
a) It is inherited as an autosomal recessive trait.
b) It is more common in the elderly than in children.
c) Patients with a family history of malignant hyperthermia cannot safely
undergo elective surgery if general anesthetics are to be used.
d) Termination of the procedure and intravenous use of danazol are the
recommended treatment for patients experiencing an acute intraoperative
episode of malignant hyperthermia.
e) The pathogenesis is related to intramuscular calcium transport

30- Oxygen delivery is dependent on all of the following EXEPT:-


a) Cardiac output
b) Hemoglobin
c) Oxygen saturation
d) Arterial oxygen pressure
e) Metabolic acidosis

31. The major indications for performing orchidopexy.


a) To enhance fertility.
b) To prevent the likelihood of development of cancer.
c) To repair a concomitant hernia.
d) To reduce the likelihood of torsion, trauma, and pain.
e) For psychological effect and cosmesis.
32. Clinical features of blunt trauma of children, All of the following are
correct EXCEPT:
a) The most common cause of death and disability following severe
trauma in children, is related to CNS injury.
b) Thoracic injuries in children are second to head injury as a cause of
mortality.
c) Massive hemothorax is the most common indication for urgent
thoracotomy in the injured child.
d) CT scan of the abdomen is superior to both U/S and Isotope scan, as it
provides more information and clearer images.
e) Flail chest is primarily a problem of children under 12 years of age.

33. All the following are manifestations of hepatocellular failure


EXCEPT:
a) Gynecomastia
b) Polycythemia
c) Hypoalbuminemia
d) Secondary hyperaldosteronism
e) Increased sensitivity to morphine

34.Radiotherapy plays an important role in the treatment of the following


tumors except
a) Rectal carcinoma
b) Wilms tumor
c) Gastric carcinoma
d) Medulloblastoma (intracranial)
e) Hodgkin’s diseas

35. In patient receiving assisted ventilation with positive end expiratory


pressure (PEEP).
The sudden occurrence of hypotension most likely caused by
a) Hypovolemia
b) Acute congestive cardiac failure
c) Haemothorax
d) Massive atelactasis
e) Tension pneumothorax

36. The most reliable indicator of the adequacy of burn resuscitation is.
a) Central venous pressure.
b) Pulmonary capillary wedge pressure.
c) Urine output.
d) Blood pressure and heart rate.
e) Mental status

37.Infection affects wound healing by all the following mechanisms


except:
a) Prolonging oedema.
b) Decreasing tissue PO2.
c) Increasing collagenolysis.
d) Decreasing the inflammatory phase.
e) Increasing the inflammatory phase.

38. In females with UTI the best way to collect urine for culture is
a) Mid stream urine
b) Suprapubic aspiration
c) Initial stream
d) By catheterization
e) By cleaning the area and any stream

39. When a renal mass identified by I.V.P., features suggestive of


malignancy include all except
a) Calcifications within the mass
b) Increase tissue density
c) Irregularity of the margin
d) Decrease tissue density
e) Distortion of the collecting system

40. Contraction of bladder as a whole generally requires stimulation by


a) Sympathetic nerve
b) Parasympathetic nerve
c) Somatic nerve
d) It contracts spontaneously
e) Sympathetic and parasympathetic

41. Microscopic BPH describes


a) Enlarged” prostate
b) a proliferative process of the stromal and epithelial elements of the
prostate
c) a proliferative process of the stromal elements
d) a proliferative process of the epithelial elements
e) prostate size should be more than 40 gm

42. Chronic scrotal pain is most often due to


a) Testicular torsion
b) Trauma
c) cryptochidism
d) hydrocele
e) orchitis

43. The most common complication after TURP is


a) Failure to void
b) Hemorrhage requiring transfusion
c) Clot retention
d) UTI
e) TUR syndrome

44. What is the most sensitive test for identifying residual freagments
after PCNL
a) Nephrestomytubogram
b) MRI
c) Ultrasonography
d) Noncontrast CT
e) CT with contrast

45. The standered method of urinary tract reconstruction during renal


transpantationis
a) ureteropyeloplasty
b) ureteroureterostomy
c) ureteroneocystostomy
d) vesicopyeloplasty
e) cutaneous ureterostomy

46. In electrolyte disorders that occur when jejunum is used for urinary
intestinal diversion, one is not happened:
a) hyponatremia
b) hyperchloremia
c) hyperkalemia
d) azotemia
e) acidosis
47. The landmark in retroperitoneoscopy:
a) Psoas muscle
b) Renal artery
c) Renal vein
d) Ureter
e) Transversalis muscle

48. In Renal Trauma, one is true:


a) The degree of hematuria and the severity of the renal injury correlate
consistently
b) All blunt trauma patients with gross hematuria should undergo renal
imaging.
c) Renal image is not mandatory in penetrating injuries with microscopic
hematuria
d) The preferred imaging study for renal trauma is non-contrast-enhanced
CT
e) patients with microscopic hematuria and hemodynamically stable
should undergo renal imaging

49. Definitive diagnosis of a ruptured bladder is made by performing a


(n):
a) IVP.
b) Ultrasound of the abdomen.
c) CT of the abdomen.
d) CT cystogram.
e) VCU.

50. Adequate urine output for adult postoperative surgical patients is


greater than
a) 35 ml|hr regardless of body size
b) 50 ml|hr regardless of body size
c) 0,5 ml|kg|hr
d) 1,0 ml|kg|hr
e) 1,5 ml|kg|hr

51. For pediatric patients with appendicitis, which of the following


statements is NOT true?
a) The rate for misdiagnosis is highest in children under 3 years old.
b) CT scan is less accurate in children than adults.
c) CT scan may be used to rule out peri-appendiceal abscess.
d) Urinary sepsis is the commonest misdiagnosis.
e) WBC count does not effectively differentiate perforated from
nonperforated appendicitis.

52. A 38-year-old man has a painless, nontender mass in his left neck that
moves with swallowing. Fine-needle aspiration shows medullary
carcinoma. The right side appears normal. The best course of action
would be
a) Left thyroid lobectomy
b) Subtotal right lobectomy and left lobectomy
c) Total thyriodectomy
d) Total thyroidectomy with central neck dissection
e) Total thyroidectomu with radical neck dissection

53. Bacteremia in patients with biliary tract sepsis is most likely due to
a) Bacteroides Fragilis
b) Enterobacter
c Escherichia coli
d) Enterococcus
e) Coagulase-negative Staphylococcus

54. A 24-year old man undergoing laparotomy for symptomatic Crohn’s


disease has a 2-cm stricture of the mid-ileum without any evidence of
disease elsewhere in the gastrointestinal tract. The best surgical option for
this lesion would be

a) Resection with primary anastomosis


b) Heineke-Mikulicz strictureplasty
c) Heineke-Mikulicz strictureplasty with biopsy
d) Mechanical dilatation
e) Isoperistaltic side-to-side strictureplasty

55. Incompatible blood transfusion during surgery under general


anesthesia, which of the following is NOT true.
a) Unexplained bleeding.
b) Hematuria may be present.
c) Skin rash may be seen
d) Hypertension is well know problem in spite transfusion
e) Volume support is mandatory.
56. In regard to ventilation in pediatric age group, which of the following
is the most deleterious?
a) CPAP.
b) High volume ( Volutrauma).
c) High pressure ( Barotrauma)
d) PEEP.
e) High ventilatory rate.

57. Regarding urethral injuries, all are true except:


a) Prostatic hematoma may be present.
b) Perineal hematoma may be seen.
c) Fracture pelvis may be the cause.
d) Descending urethrogram is the investigation of choice.
e) May be treated with suprapubic urinary bladder catheterization.

58. Regarding overwhelming post splenectomy sepsis, which of the


following is true.
a) Pneumococcal vaccination is not protective.
b) It is common after traumatic splenectomy.
c) Mortality rate is low.
d) It is common after splenectomy for blood diseases.
e) It is common after 10 years post surgery.

59. Regarding abdominal trauma in paediatric age group, which of the


following is true:

a) Clinical examination, stabilization and resuscitation are not of priority.


b) Ultrasound is the diagnostic tool of choice.
c) Blood transfusion should be started immediately.
d) Peritoneal lavage is contra-indicated.
e) Splenectomy is not a routine management for all types of splenic
injuries

60. With regard to Hirschsprung’s disease ( aganglionosis), all the


following are true except:
a) It is more common in males.
b) It may be complicated by enterocolitis.
c) Barium enema study may be normal.
d) It is best diagnosed by full thickness rectal biopsy.
e) Surgery can’t be accomplished without colostomy.

61. Which of the followings is not a complication of esophageal atresia


and tracheoesophageal fistula repair?:
a) Esophageal stricture.
b) Anastomotic leakage.
c) Gastroesophageal reflux.
d) Recurrent fistula.
e) Massive variceal bleeding.

62. The most common condition, which needs liver transplantation in


pediatric age group, is:
a) Metabolic liver disease.
b) Malignant liver tumors.
c) Biliary atresia.
d) Choledochal cyst.
e) Viral hepatitis.

63. The least blood loss during burn wound excision occurs when
escharectomy is performed:
a) During the first 48 hours.
b) On days 3-5 post-burn.
c) On days 6-10 post-burn.
d) After day 10 post-burn.
e) When eschar is infected.

64. Contraindications to breast reconstruction after mastectomy include:


a) Diabetes Mellitus.
b) Age over 60 years.
c) Obesity.
d) Bilateral breast cancer.
e) None of the above.

65. All the following muscles are supplied by the trigeminal nerve
EXCEPT:
a) Tensor veli palatini.
b) Tensor tympani.
c) Masseter.
d) Posterior belly of digastric.
e) Mylohyoid.

66. The advantages of early excision and graft for full-thickness burns in
the first 48 hours post-burn include all of the following EXCEPT:
a) Less blood loss.
b) Less pain.
c) Improved cosmetic result.
d) Decreased hospital stay.
e) Ease of differentiation between partial and full-thickness burns.

67. All of the following statements are correct concerning Corticosteroids


EXCEPT:
a) Inhibit fibroblast migration into the wound.
b) Inhibit the activity of prolylhydroxylase.
c) Prevent collagen deposition.
d) Inhibit wound contracture.
e) Vitamin A restores the harmful effects of steroids.

68. The burn that is difficult to estimate in percentage is the:


a) Electrical burn
b) Sun burn
c) Scald burn
d) Direct flame burn
e) Chemical burn

69. All of the following are signs of burn inhalation injury EXCEPT:
a) Erythema of oral mucosa
b) Coughing
c) Black sputum
d) Red colored burn
e) Burn of nasal hair

70. A blow-out fracture of the orbit is by definition:


a) A fracture that results in rupture of the globe.
b) A fracture that results in disruption of the orbital rim.
c) A fracture of the thin walls of the orbit allowing herniation of
orbital contents.
d) A fracture that results from an explosion.
e) A fracture that includes bones adjacent to the orbit.
71. A patient who has a penetrating wound to the extremity has the
arteriogram shows less than 5mm intimal defects and psudoaneurysm.
Management of this injury should be
a) immediate operation
b) Anticoagulant for 6 wks.
c) Observation
d) Elective repair
e) Antiplatelet therapy for 6 wks.

72. Which of the following statements about carotid endarterectomy is


not true?
a) Patients with a symptomatic critical stenosis benefit from carotid
endarterectomy
b) The risk of contralateral stroke is reduced after carotid
endarterectomy
c) Operative mortality is less than 2%
d) It prolongs survival when compared with patients treated medically
e) It should be accompanied by antiplatelet medication

73. Which of the following is not associated with left-sided portal


hypertension
a) Gastric varices
b) Abdominal pain
c) Splenomegally
d) Hepatomegally
e) Chronic pancreatitis

74. A 15 years old boy complains of excessive sweating of his hands.


They are cold, clammy, and damp to the touch. Perspiration dripping
from his hands forms a small puddle during the examination. The
remainder of medical history is unremarkable. Which of the following
statements is TRUE
a) the result of sympathectomy for this disorder are generally good
b) the sweat glands involved here are apocrine
c) if not treated, this will likely be permanent
d) topical aluminum salts create ulceration
e) a short course of methotrexate is ofen helpful

75. Which of the following statements about blunt tracheobronchial


trauma is true
a) Repair should include a prophylactic tracheostomy
b) Delayed repair is preferred
c) Aneasthesia is safer if the endotracheal tube can be passed beyond
the site of injury
d) Massive hemoptysis is common at presentation
e) Bronchoscopy is hazardous and should be avoided

76. Regarding the functions of a tracheostomy all are true, except:


a) Bypasses an upper airway obstruction.
b) Increases the anatomical dead space.
c) Decreases airway resistance.
d) Protects against aspiration.
e) Allows frequent airway suction.

77. With regard to protein loss after injury, which of the following
statements is true?
a) It results from impaired protein synthesis.
b) It occurs primarily from catabolism of skeletal muscle.
c) It occurs primarily from acute renal failure.
d) It occurs primarily from the site of injury.
e) It can be prevented by total parenteral nutrition.

78. Which of the followings is the most important stimulus for triggering
the endocrine response to injury?
a) Tissue acidosis.
b) Local wound factors.
c) Afferent nerve stimuli from the injured area.
d) Hypovolemia.
e) Temperature changes.

79. All of the followings are true regarding perforated appendicitis,


except:
a) Higher rate in children.
b) Higher rate in old patients.
c) Usually due to delay in presentation and diagnosis.
d) Appendicular mass is felt in more than 50% of the cases.
e) The commonest complication is wound infection.
80. Factors which contribute to wound dehiscence include all the
followings, except:
a) Old age.
b) Coughing.
c) Hypoproteinemea.
d) Anaemia.
e) Malignancy.

81.The lesions in familial adenomatous polyposis with the smallest


malignant potential are
a) Gastric polyps
b) Duodenal polyps
c) Abdominal desmoids
d) Brain tumors
e) Ileal polyps

82. Regarding hernias All true except


a) One portion of the bowel wall is herniated , but not
entire lumen is called Richter hernia
b) Spigelian hernia is Associated with the semilunar
line
c) Femoral hernias are more common in females than
males
d) Paradoudenal hernia is an internal hernia where
the superior mesenteric vein forms part of its
boundaries
e) Obturator hernia Causes anterior thigh pain with
walking

83. All of the following are consequences of obstructive


jaundice except
a) Wound healing is impaired
b) The function of kupffer cells is increased
c) There is a decrease in the absorption of fat soluble
vitamins
d) There is an increased risk of renal failure
e) There is an increased chance of the bile being
infected in malignant obstruction

84. In patients with upper gastrointestinal haemorrhage due to


a peptic ulcer All true except
a) A visible vessel in the ulcer bed at endoscopy is a
risk factor for rebleeding
b) An older patient would be expected to have a
higher chance of rebleeding
c) An adherent clot seen at endoscopy reduces the
risk of rebleeding
d) H2 antagonists do not reduce the bleeding rate
e) A gastric ulcer is more likely to bleed than a
duodenal ulcer

85.In the management of trauma patient all true except


a) Mouth to mouth breathing provides a maximum
inspired oxygen concentration of 10%
b) Criciod pressure should be not be used to aid
endotrachial intubation in vomiting patients.
c) Nasogastric intubation is contraindicated in patients
who are suspected to have fracture of the base of
skull
d) Needle cricothyroidotomy aids effective ventilation
for up to 30 minutes
e) Insertion of (oropharyngial airway) should be
attempted in only on unconscious patients

86. Local anesthetic agents


a) Should not be given intravenously
b) Lignocaine (xylocaine) is four times more potent
than bupivacaine
c) The maximum safe dose for Lignocaine is 3mg/kg
and up to 7mg/kg if mixed with adrenaline
d) Can be used safely in the distal limbs even if mixed
with adrenaline
e) Bupivacaine is less cardiotoxic than lignocaine
87. Regarding anal fissures, all are true except:
a) 10% occur in the posterior midline
b) Multiple fissures suggests the diagnosis of TB or
crohn's disease
c) 50% of the fissures heal with the use of bulking
agent
d) Sphinterotomy has a success rate of over 90%
e) Sphinterotomy is associated with minor fecal
incontinence in over 15% of patients

88. All of the following are indications for surgery in patients


with spontaneous pnuemothorax except;
a) Scuba divers
b) Individuals living in remote area
c) Airline pilots
d) Age more than 60 years
e) Previous contralateral pnuemothorax

89. Low molecular weight Heparin as its primary inhibitory


effect on one of the following:
a. Factor II.
b. Factor IXa.
c. Antithrombin III.
d. Factor Xa.
e. Factor XIIa.

90. The O2 dissociation curve is shifted to the right by :


a) Decreased CO2 tension
b) Increased CO2 tension
c) Increased PH
d) Increased N2 tension
e) Decreased N2 tension

91. The Glasgow coma scale is the dependant upon all of the
following except
a) response to speech
b) respnse to pain
c) response of the pupils
d) motor response
e) response of the patient

92. In traumatic subarachnoid haemorrhage all of the following


are correct except .
a) is commoner than aneurysmal haemorrage
b) is commoner than subarachnoid haemorrhage associated
with AVM.
c) may cause vasospasm and cerebral infarction
d) is often associated with frontal skull fracture
e) is more common in elderly people

93. The adverse prognostic factors for the development of acute


subdural hematoma involve all of the following except
a) old age
b) young age
c) chronic alcoholism
d) skull fracture
e) temporal agenesis

94. Extradural hematoma has the following characters except


a) is more common than acute subdural hematoma
b) is more often associated with vault skull fracture
c) is less often associated with primary brain injury
d) is more like to expand
e) is more likely to be arterial in origin

95. The following are indication for admission to the hospital after a
minor head injury except
a) reduce level of consiousness
b) concussion without skull fracture
c) clinical and radiological evidence of skull frature
d) focal neurological dysfunction
e) difficulty in assessing the patient

96. Which anastamosis location has the highest leakage rate?


a) Jejunojejunostomy after a Roux-en-Y reconstruction
b) Pancreaticojejunostomy
c) Esophagojejunostomy
d) Rectosigmoid anastamosis
e) Gastrojejunostomy

97. Bilibrubin
a) Is conjugated to glucuronic acid in the gallbladder
b) Is transported in hepatic sinusoidal blood bound to
albumin
c) When conjugated, is secreted into bile by passive
diffusion
d) Is converted to urobilinogen by jejunal enterocytes
e) Is produced predominantly by early phase (< 3
days) erythrocyte heme breakdown
98.Exclusion criteria for laparoscopic colectomy for diverticulitis
include all of the following EXCEPT
a) Multiple areas of colonic involvement
b) Purulent peritonitis
c) BMI > 30 kg/m2
d) Free peritoneal perforation
e) Multilocular abscess

99.Forty-eight hours after performing a laparoscopic


cholecystectomy, a surgeon realizes that one of the clips was
placed across the common bile duct. The correct approach
would be
a) Drainage via percutaneous transhepatic cholangiogram
b) Operative removal of the clips
c) Choledochoduodenostomy
d) Loop choledochojejunostomy
e) Roux-en-Y choledochojejunostomy

100. The ilioinguinal nerve


a) Is a branch of the femoral nerve
b) Has a motor component
c) Can be injured easily at the internal spermatic ring
d) Provides sensation for the penis and upper scrotum
e) Runs along the hypogastric vein

1-A
2-C
3-B
4-C
5-B
6-D
7-B
8-B
9-B
10-B
11-B
12-E
13-D
14-E
15-B
16-A
17-A
18-A
19-A
20-D
21-D
22-E
23-E
24-E
25-C
26-E
27-C
28-D
29-E
30-E
31-B
32-E
33-B
34-C
35-E
36-C
37-D
38-D
39-D
40-B
41-B
42-D
43-A
44-D
45-c
46-b
47-a
48-b
49-d
50-c
51-d
52-d
53-c
54-a
55-d
56-b
57-d
58-d
59-e
60-e
61-e
62-c
63-a
64-e
65-d
66-e
67-c
68-a
69-d
70-c
71-c
72-d
73-d
74-a
75-c
76-b
77-b
78-c
79-d
80-d
81-a
82-d
83-b
84-c
85-a
86-c
87-a
88-d
89-d
90-b
91-c
92-d
93-b
94-a
95-b
96-b
97-b
98-c
99-e
100-d
1.Regarding Peutz Jeghers syndrome all are true, except:
a. Is an autosomal dominant condition.
b. Often presents with anaemia in childhood.
c. Is characterised by circumoral mucocutaneous pigmented lesions.
d. Is associated with adenomatous polyps of the small intestine.
e. Malignant change occurs in 2-3% of polyps.

2. Regarding peptic ulceration all are true, except:


a. H. pylori is a gram-negative bacillus.
b. Duodenal is more common than gastric ulceration.
c. Zollinger-Ellison syndrome is associated with gastrin hyposecretion.
d. H2-blockers will heal 85-95% of duodenal ulcers in 8 weeks.
e. Triple therapy can eradicate H. pylori in 80% of patients in one week.

3.Regarding head Injuries all are true, except:


a. More than 1 million people are seen in UK hospitals each year with head injuries.
b. Skull X-rays can not exclude an intracerebral haematoma.
c. Raised intracranial pressure is associated with an increase in cerebral perfusion.
d. Cushing's response consists of a rise in blood pressure and fall in heart rate.
e. Pupillary dilatation usually occurs on the same side of the intracerebral
haematoma.

4. Regarding stones in the gallbladder, one is correct:


a. Pigment stones are the most common.
b. Pigment stones are due to increased excretion of polymerised conjugated
bilirubin.
c. Are not a risk factor for the development of gallbladder carcinoma.
d. 90% of gallstones are radio-opaque.
e. A mucocele of the gallbladder is mostly caused by a stone impacted in
Hartmann's pouch.

5.Regarding stones in the common bile duct, all are true, except:
a. Are found in 10 to 15% of patients undergoing cholecystectomy (Without pre-op
ERCP).
b. Can present with Charcot's Triad.
c. Are suggested by a bile duct diameter >8mm on ultrasound.
d. ERCP, sphincterotomy and balloon clearance is now the treatment of choice.
e. If removed by exploration of the common bile duct the T-tube can be removed
after 3days.

6. Regarding crystalloid solutions, all are true, except:


a. Normal saline contains 154 mmol sodium and 154 mmol chloride.
b. 3 litres of dextrose 1/5 th saline in a day will provide 90 mmol of sodium.
c. 2 grams of potassium chloride is equal to 57 mmol of the salt.
d. Hartmann's solution contains calcium and bicarbonate.
e. The daily maintenance potassium requirement of a 40 Kg woman is about 40
1
mmol.

7. Regarding colloid solutions, all are true, except:


a. Human albumin has a molecular weight of 69 kDa.
b. Albumin has a half life in the circulation of about 15 days.
c. Gelatins (e.g. Haemaccel®) are polysaccharides with a MW of about 35 kDa.
d. Dextrans reduce platelet aggregation and can induce anaphylaxis.
e. 6% Hydroxylethyl Starch (HES) is a synthetic polysaccharide derived from
amylopectin.

8. Regarding central parenteral nutrition, all are true, except:


a. Is a hypo-osmolar solution.
b. Typically contains 14-16g nitrogen as L-amino acids.
c. Typically contains about 250g glucose.
d. Is associated with metabolic disturbances in about 5% of patients.
e. Can induce derangement of liver function tests.

9.Regarding solitary thyroid nodules, all are true, except:


a. Are more prevalent in women.
b. In the adult population, less than 10% are malignant.
c. More than 50% of scintigraphically cold nodules are malignant.
d. The risk of a hot nodule being malignant is negligible.
e. Should not be surgically removed in all patients.

10. Regarding the femoral canal, one is correct:


a. Lies lateral to the femoral vein.
b. Has no relation to the inguinal ligament.
c. Has the lacunar ligament as its lateral border.
d. Has the pectineal ligament as its posterior border.
e. Contains the lymph node of Lund.

11.Regarding the pathology of ulcerative colitis, one is true:


a. Shows full thickness bowel inflammation.
b. The rectum is almost always not involved.
c. 10% of patients have terminal ileal disease.
d. Enterocutaneous or intestinal fistulae are common.
e. The serosa is usually affected.

12. Regarding benign breast disease, all are true, except:


a. Cyclical mastalgia is the commonest reason for referral to the breast clinic.
b. Fibroadenomas are derived from the breast lobule.
c. Lactational breast abscesses are usually due to Staph aureus.
d. Duct ectasia is less common in smokers.
e. Atypical lobular hyperplasia is associated with an increased risk of breast cancer.

2
13.Regarding infantile hypertrophic pyloric stenosis, one is true:
a. Occurs with a male : female ratio of 4:1.
b. Sons of affected mothers have a 80% risk of developing the lesion.
c. Invariably presents between six and eights months of age.
d. Typically presents with bile stained projectile vomiting.
e. Surgical treatment is by Heller's cardiomyotomy.

14. Regarding oesophageal atresia , all are true, except:


a. Is often associated with a distal tracheo-oesophageal fistula.
b. Polyhydramnios is often present late in pregnancy.
c. 50% have other associated congenital abnormalities.
d. Contrast x-ray studies are necessary to confirm the diagnosis.
e. Post-operatively over 15% develop oesophageal strictures.

15.All the following are features of Fallot's Tetralogy, except:


a. Atrial septal defect.
b. Pulmonary stenosis.
c. Right ventricular hypertrophy.
d. A right to left cardiac shunt.
e. Cyanotic attacks during feeding and crying.

16.Regarding phaeochromocytomas, one is correct:


a. Are neuroendocrine tumours of the adrenal cortex.
b. Are benign in 10% of cases.
c. Occur as part of the MEN type I syndrome.
d. Can be imaged with a meta-iodobenzylguanidine scan.
e. Require only beta-blockage drug prior to surgery.

17.Regarding the functions of a tracheostomy all are true, except:


a. Bypasses an upper airway obstruction.
b. Increases the anatomical dead space.
c. Decreases airway resistance.
d. Protects against aspiration.
e. Allows frequent airway suction.

18. Regarding hepatitis B, all are true, except:


a. It is due to a DNA virus.
b. It has an incubation period of 2 to 6 days.
c. It can be transmitted via blood products.
d. About 1% of the UK population are hepatitis B surface antigen positive.
e. About 10% of infected patients become chronic carriers.

3
19.The best surgical modality to treat recurrent primary spontaneous
pneumothorax in a 20 year old man is:
a.bullectomy.
b.pleurectomy.
c.bullectomy and pleurectomy.
d.pleurodesis.
e.decortication.

20-Regarding the anatomy of diaphragm, all are true except:


a.The hiatal opening is situated at the level of the tenth thoracic vertebra.
b.The inferior vena cava passes through the right side of diaphragm at the level of
12th thoracic vertebra.
c.The principle blood supply of the diaphragm is derived directly from the aorta.
d.The left diaphragm is lower than the right diaphragm in a normal adult.
e.Nerve supply to the diaphragm is via phrenic nerve which arises from 3 rd,4th&5th
cervical nerve roots.

21.Which one of the following diagnostic methods is the best to diagnose


pneumothorax?
a.Standing CXR, expiratory film.
b.Standing CXR, inspiratory film.
c.Supine CXR, expiratory film.
d.Supine CXR, inspiratory film.
e.Lateral CXR.

22.Regarding chest trauma, all are true, except:


a.Rib fracture in a child is usually a marker of severe injury.
b.Thoracostomy tube should be inserted prior to CXR in any patient with suspected
pneumothorax associated with hypotension.
c.A patient with flail chest, should be considered for ventilatory support if his
respiratory rate is above 35/min.
d.Exploratory thoracotomy should be done if the amount of blood production
through the chest tube is more than 200 cc hourly for 4 hours.
e.Chest tube should be clamped during the transport of the patient.

23. The best site to insert a chest tube is:


a.2nd intercostal space, anterior axillary line.
b.3rd intercostal space,posterior axillary line.
c.4th intercostal space ,posterior axillary line.
d.5th intercostal space, anterior axillary line.
e.7th intercostal space, mid axillary line.

4
24. The best non invasive diagnostic procedure to diagnose diaphragmatic
injury after trauma is:
a.Chest xray.
b.Chest CT scan.
c.Chest MRI.
d.Chest ultrasound.
e.Barium meal.

25.Regarding the anatomy of the tracheobronchial tree and the lung, all are
true, except:
a.The right lung is larger than the left lung.
b.The major (oblique fissure) separates the lower lobe from the upper and middle
lobe.
c.There are 8 segments in the right lung.
d.The right main bronchus is shorter and wider than the left main bronchus.
e.The posterior wall of the trachea is membranous.

26. All the followings are true data about the anatomy and the physiology of
lower oesophageal sphincter, except:
a.In adults, it measures 3 to 5 cm in length.
b.In normal individuals, manometry demonstrates pressure of 5 mm Hg within the
lower oesophageal sphincter.
c.The lower oesophageal sphincter remains contracted at rest.
d.It is not a true sphincter.
e.It normally relaxes with the approach of the primary peristaltic wave.

27.Which of the following is probably a posterior mediastinal mass rather than


an anterior one?
a.Thyroid tumour.
b.Neurogenic tumour.
c.Thymic carcinoma.
d.Teratoma.
e.Lymphoma.

28. Regarding thoracic duct,one is correct:


a.Is the main collecting vessel of the lymphatic system and is far smaller than the
right terminal lymphatic duct.
b.Originates from the cysterna chyle at the level of 4th lumbar vertebra.
c.Originates in the left side.
d.Ascends to enter the chest through the inferior vena cava hiatus.
e.Lies on the anterior surface of the vertebral column behind the esophagus and
between the aorta and the azygous vein.

5
29. The maximum safe dose of local anesthetic administered subcutaneously in
a 70 kg man is:
a.10 to 20 mL of 1% lidocaine.
b.40 to 50 mL of 2% lidocaine with epinephrine.
c.40 to 50 mL of 1% lidocaine with epinephrine.
d.40 to 50 mL of 1% bupivacaine.
e.40 to 50 mL of 1% lidocaine without epinephrine.

30.Which of the following vessels of the lower limb is least likely to be occluded
by atherosclerosis?
a.Anterior tibial.
b.Peroneal.
c.Proximal posterior tibial.
d.Distal posterior tibial.
e.Dorsalis pedis.

31. After thoracotomy, a woman has a loss of sensation of the nipple areolar
complex,injury to which intercostal nerve is responsible for this?
a.Second.
b.Third.
c.Fourth.
d.Fifth.
e.Sixth.

32. Oxygen delivery can be increased by increasing all of the followings, except:
a.Hemoglobin.
b.Atmospheric pressure.
c.Cardiac output.
d.Inspired oxygen concentration.
e.Oxygen extraction.

33. Which of the followings is the best abdominal site to assess the bowel sounds
using the stethoscope?
a.Right upper quadrant.
b.Right lower quadrant.
c.Left upper quadrant.
d.Left lower quadrant.
e.Periumbilical.

34. Brain stem death is diagnosed in the absence of these brain stem reflexes,
except:
a- Corneal reflexes.
b- Pupillary reflexes.
c- Doll's eye movements.
d- Caloric response.
e- Apnoeic test.
6
35. The followings are indications for admission to hospital after a minor head
injury, except:
a- reduced level of consciousness.
b- Concussion without skull fracture.
c- Clinical or radiological evidence of skull fracture.
d- Focal neurological dysfunction.
e- Difficulty in assessing the patient.

36. In Extradural haematomas all the followings occur, except:


a- One in ten occur in the frontal region.
b- Less than half exhibit a lucid interval.
c- 20% occur in the temporal area.
d- The first symptom is often progressive paroxysmal headache.
e- Nearly always associated with vault skull fracture.

37. The source of bleeding in the extradural space are the followings, except:
a- Diploic vessels of the skull vault.
b- The cerebral veins.
c- The middle meningeal vessels.
d- The dural sinuses.
e- The dural arterioles.

38.The following regimens are used to treat raised intracranial pressure


(ICP), except:
a- Hypoventilation .
b- Mannitol 20% solution, 1g mannitol/kg body weight.
c- Drainage of CSF.
d- Barbiturates.
e- Surgical decompression.

39. Cerebrospinal fluid (CSF) characterizes by the followings ,except:


a. Circulates at about 0.5 ml/min.
b. Secreted at about 500 ml/24 hours.
c. Secreted in the lateral,third,and fourth ventricles.
d. Secreted predominantly by active secretion.
e. Absorbed by the choroid villi.

7
40. Definitive diagnosis of Pulmonary embolism is made by one of the
followings:
a- ECG (EKG).
b- Chest X-ray.
c- VQ scan.
d- ABG.
e- Pulmonary angiography.

41.In the body metabolism, 10g of protein, would produce approximately:


a. 20 kcal.
b.41 kcal.
c.410 kcal.
d.4100 kcal.
c. 900 kcal.

42. The Glasgow coma scale is dependant upon all of the followings, except:
a.Response to speech.
b.Response to pain.
c.Response of the pupils.
d.Motor response.
e.Response of the patient.

43. Which one of the following spinal segments causes signs of wasting of the
intrinsic muscles of the hand?
a- C5.
b- C7/8.
c- T1.
d- L5.
e- S1.

44. The methods by which bacteria gain access into the brain include all the
followings, except:
a- Direct spread from the surrounding tissues.
b- Haematogenous.
c- Via implantation during trauma.
d- Lymphatics.
e- During surgery.

45. All the followings are premalignant lesions of the skin, except:
a. Actinic keratosis.
b. Bowen’s disease.
c. Seborrheic keratosis.
d. Dysplastic nevi.
8
e. Nevus sebaceous.

46. The commonest cause of death after major burns in modern burn units is:
a) Inadequate resuscitation.
b) Pneumonia.
c) Burn wound sepsis.
d) Renal failure.
e) Bacterial endocarditis.

47. Complications of mandibular fractures include all the followings, except:


a. Malocclusion.
b. Lower lip paraesthesia.
c. Trismus.
d. Ankylosis of the temperomandibular joint.
e. Paralysis of the lower lip.

48. An ulnar nerve injury at the elbow would result in all the followings,
except:
a. Clawing of the ring and little fingers.
b. Inability to abduct the little finger.
c. Inability to flex the ring and little fingers.
d. Inabilility to adduct the thumb.
e. Radial deviation of the wrist in flexion.

49. The most important factor which decide the efficacy of laser treatment for a
particular condition is:
a. Age of patient.
b. Depth of lesion.
c. Energy Fluence (Power density x Duration of treatment).
d. Spot size.
e. Wavelength as decided by the lasing medium.

50. A blow-out fracture of the orbit is by definition:


a. A fracture that results in rupture of the globe.
b. A fracture that results in disruption of the orbital rim.
c. A fracture of the thin walls of the orbit allowing herniation of the orbital
contents.
d. A fracture that results from an explosion.
e. A fracture that includes bones adjacent to the orbit.

51. Which of the followings is considered the primary blood supply to the skin?
a. Fascial plexus.
b. Subcutaneous plexus.
c. Subdermal plexus.
d. Dermal plexus.
e. Subepidermal plexus.
9
52. Principles of skin wound repair include all of the followings, except:
a. Obliteration of the dead space.
b. Eversion of the skin edges.
c. Tension free closure.
d. Layered tissue closure.
e. Use of nylon simple sutures at the skin edge.

53. The most reliable indicator of the adequacy of burn resuscitation is:
a. Central venous pressure.
b. Pulmonary capillary wedge pressure.
c. Urine output.
d. Blood pressure and heart rate.
e. Mental status.

54. The most common site for squamous cell carcinoma of the lip is the:
a. Upper lip midline.
b. Upper lip laterally.
c. Lower lip midline.
d. Lower lip laterally.
e. Oral commissure.

55. Which of the following cell types is essential for normal wound healing?
a. Leukocytes.
b. Monocytes.
c. Platelets.
d. Erythrocytes.
e. All of the above.

56. The most important factor in the development of ulcers in a spinal cord
injury patient is:
a. Malnutrition.
b. Septic episodes.
c. Anemia.
d. Local pressure.
e. Diminished sensation.

57. Which vitamin D metabolite stimulate intestinal calcium absorption?

a. 1,25–dihydroxyvitamin D.
b. 1,25–dihydroxyvitamin D1.
c. 1,25–dihydroxyvitamin D2.
d. 1,25–dihydroxyvitamin D3.
e. 1,25–dihydroxyvitamin D4.

10
58. Urinary stones generally do not pass spontaneously if they are larger than?

a. 2 mm.
b. 3 mm.
c. 4 mm.
d. 7 mm.
e. 8 mm.

59.Which of the following studies replace intravenous urography in the initial


evaluation of patients with renal colic?

a. Kidney, ureter and bladder study.


b. Renal tomography.
c. Ultrasonography.
d. CTScan.
e. MRI.

60. Initial hormonal evaluation of an infertile man with a sperm count of 5


million sperm/ml should include assays of testosterone level and:

a. Prolactin.
b. Follicle–stimulating hormone (FSH).
c. FSH, luteinizing hormone (LH) and prolactin.
d. FSH and thyroid function.
e. Sex steroid hormone binding globulin (SHBG).

61. Where does benign prostatic hyperplasia (BPH) originate?

a. In the transition zone.


b. In the peripheral zone.
c. In the periurethral glands.
d. In the transition zone and periurethral zone.
e. In the whole organ.

62. It is advisable in a man with BPH and a slightly elevated creatinine level to
perform a(an):

a. Transurethral resection of the prostate (TURP).


b. Intravenous pyelography.
c. Renal sonography.
d. Urodynamic study.
e. kidney, ureter and bladder X-ray.

11
63.A 20 year old man has undergone a retroperitoneal dissection for a
testicular germ cell tumor. The inferior mesenteric artery has been divided
during reflection of the intestines to expose the retroperitoneum. This can be
expected to result in:

a. Ischemia of the descending colon.


b. Ischemia of the sigmoid colon.
c. Ischemia of the rectum.
d. Ischemia of the transverse colon.
e. None of the above.

64. The predominant buffering system in humans is:

a. Bicarbonate.
b. Titratable acids.
c. Ammonium (NH4+).
d. Urea.
e. Phosphate.

65. Terminal hematuria (at the end of the urinary stream) is usually due to:

a. Bladder neck or prostatic inflammation.


b. Bladder cancer.
c. kidney stones.
d. Ureteric calculi.
e. Ureteral stricture disease.

66. Pneumaturia may be due to all of the followings, except:

a. Diverticulitis.
b. Colon cancer.
c. Recent urinary tract instrumentation.
d. Inflammatory bowel disease.
e. Ectopic ureter.

67. As one proceeds outward from the adrenal medulla, the three separate
functional layers of the adrenal cortex are, in correct order:

a. The zona reticularis, zona fasciculata, then zona glomerulosa.


b. The zona fasciculata, zona reticularis, then zona glomerulosa.
c. The zona glomerulosa, zona fasciculata, then zona reticularis.
d. The zona glomerulosa, zona reticularis, then zona fasciculata.
e. The zona reticularis, zona glomerulosa, then zona fasciculata.

12
68. What proportion of the cardiac output is delivered to the kidney?

a. 5%.
b. 20%.
c. 50%.
d. 85%.
e. 100%.

69. During surgical dissection, the ureter can be identified as it enters the
pelvis:

a. At the aortic bifurcation.


b. Crossing the superior border of the sacrum.
c. Crossing the common iliac artery at the branching of the internal iliac artery.
d. Crossing the uterine artery.
e. At the internal inguinal ring.

70. All the followings are consequences of major haemorrhage, except:


a- Rise in serum osmolality.
b- Rise in intracellular osmotic pressure.
c- Decrease in plasma oncotic pressure .
d- Peak albumin synthesis in 6 hrs.
e- Rise in blood sugar.

71. Signs of cardiac tamponade may include all of the followings, except:
a- Agitation and confusion.
b- Hypotension.
c- Congested neck veins.
d- Muffled heart sounds.
e- Deviated trachea .

72. About achalasia, one is not true :

a- It is the most common primary esophageal disorder.


b- Manometry reveals non relaxation of the lower esophageal sphincter at rest.
c- Acceptable treatment includes Endoscopic myotomy.
d- It is most often associated with hyperperistalsis of the esophageal body.
e- The intraluminal pressure is elevated.

73. All are causes of metabolic acidosis, except:


a- Untreated diabetes mellitus.
b- Duodenal Ulcer.
13
c- Uremia.
d- Shock.
e- Liver failure .

74. All are recognized complications of acute pancreatitis, except:


a- Adult respiratory distress syndrome .
b- Tetany.
c- Acute renal failure .
d- Disseminated intravascular coagulation .
e- Hypercalcaemia .

75. The following body fluids have high risk of HIV transmission, except:
a- Semen .
b- Blood.
c- Urine.
d- Breast milk.
e- Fresh frozen plasma.

76. Factors which contribute to wound dehiscence include all the followings,
except:
a- Old age.
b- Coughing.
c- Hypoproteinemea.
d- Anaemia.
e- Malignancy.

77. The most common extracranial solid tumour in children is:


a. Wilm's tumour.
b. Neuroblastoma.
c. Hepatoblastoma.
d. Hepatocellular carcinpma.
e. Seminoma.

78. In intussusception, all are true, except:


a. Idiopathic causes are the most common in the infantile age group.
b. Ileo-colic intussusception are seen most commonly.
c. Abdominal colic and vomiting present in most of the cases.
d. Bloody stool is an indication for open surgical reduction.
e. In older children, a pathological leading point must be excluded.

79. Regarding inguinal hernia in children, all are true, except:


a. Is more common in pre-term babies.
b. Is more common in males.
c. Testicular atrophy might be a long term complication.
d. Complications are more common in pre-terms than in term babies.
14
e. Contralateral groin exploration is routinely done to exclude bilateral hernias.

80. One of the followings is true regarding undescended testes:


a. The incidence is 1 in 10 000 boys.
b. The most common location is in the femoral region.
c. Is associated with high rate of infertility.
d. Orchidopexy is indicated at the age of five years.
e. Bilateral cases are more frequent than unilateral cases.

81. The most common indication for splenectomy in children is:


a. Splenic injury in blunt abdominal trauma.
b. Sarcoidosis.
c. Idiopathic thrombocytopenic purpura ( ITP ).
d. Hereditary spherocytosis.
e. Thalassemia.

82. Recurrent UTI in children mostly raise the suspicion of:


a. Urethral diverticulum.
b. Vesico ureteric reflux.
c. Ureteric duplication.
d. Renal ectopia.
e. Neurogenic rectum.

83. The absolute contraindication for circumcision in children is:


a. Hypospadias.
b. Blood dyscrasia.
c. Recurrent UTI.
d. Para-phimosis.
e. Phimosis.

84. A child weighs 15 Kg, his fluid requirement is:


a. 50 ml per hour.
b. 50 ml per kg per hour.
c. 1000 ml per 24 hours.
d. 10 ml per kg per hour.
e. 150 ml per hour.

85. Biliary Atresia is definitely diagnosed by:


a. HIDA scan.
b. Abdominal ultrasound.
c. Liver function test.
d. Abdominal C.T scan with i.v contrast.
15
e. Intra-operative cholangiogram.

86. Two years old infant previously healthy, presented with sudden onset of
respiratory distress, Choking and transient cyanosis. The most likely diagnoses
is:
a. F.B aspiration.
b. Pneumothorax.
c. Pneumonia.
d. Hemothorax.
e. Hemo-pneumothorax.

87. All are causes of bile-stained vomiting in newborns, except:


a. Sepsis.
b. Duodenal atresia.
c. Meconium ileus.
d. Complicated intestinal malrotation.
e. Esophageal atresia.

88. All are normal blood tests in the term newborn at day one of age, except:
a. Blood glucose 45mg-60mg / dl.
b. WBC 20. 000 to 30.000.
c. Creatinine up to 1.3mg%.
d. PCV 55%- 65%.
e. ESR 60mm/first hour.

89. The most common cause of death in old children is:


a. Malignancy.
b. Congenital heart disease.
c. Trauma.
d. Renal Anomalies
e. CNS abnormalities.

90. Causes of abdominal calcification on x-ray include all of the followings,


except:
a. Faecolith.
b. Wilm's tumour.
c. Neuroblastoma.
d. Nephrocalcinosis.
e. Calcified lymph nodes.

91. Which of the followings is the most important stimulus for triggering the
endocrine response to injury?
16
a. Tissue acidosis.
b. Local wound factors.
c. Afferent nerve stimuli from the injured area.
d. Hypovolemia.
e. Temperature changes.

92. Which of the following substances is elevated during the acute response to
injury?

a. Glucagon.
b. Insulin.
c. Thyroid stimulatring hormone (TSH).
d. Prolactin.
e. Calcitonin.

93. With regard to protein loss after injury, which of the following statements is
true?
a. It results from impaired protein synthesis.
b. It occurs primarily from catabolism of skeletal muscle.
c. It occurs primarily from acute renal failure.
d. It occurs primarily from the site of injury.
e. It can be prevented by total parenteral nutrition.

94. Normal wound healing is accelerated by one of the followings?


a. Ascorbic acid.
b. Vitamin A.
c. Zinc.
d. Increased local oxygen tension.
e. Frequent dressing.

95. All of the followings are true regarding perforated appendicitis, except:
a. Higher rate in children.
b. Higher rate in old patients.
c. Usually due to delay in presentation and diagnosis.
d. Appendicular mass is felt in more than 50% of the cases.
e. The commonest complication is wound infection.

96. With regard to blunt abdominal trauma, the commonest intra abdominal
organ which can be affected is?
a. Urinary bladder.
b. Large intestine.
c. Pancreas.
d. Stomach.
e. Spleen.

17
97. One of the followings is not a cause of hypokalemia?
a. Inadequate oral intake.
b. Potassium-free intravenous fluids.
c. High output nasogastric tube.
d. Blood transfusion.
e. Massive vomiting and diarrhea.

98. With regard to wound classification, penetrating abdominal trauma is?


a. Clean wound.
b. Clean/contaminated wound.
c. Contaminated wound.
d. Dirty wound.
e. Not classified.

99. The most common cause of hypercalcemia in hospitalized patients is?


a. Malignancy.
b. Over oral intake.
c. Bone fracture.
d. Blood transfusion.
e. Parathyroid adenoma.

100. One of the following is not a cause of dilutional hyponatremia?


a. Excessive oral water intake.
b. Iatrogenic (intravenous) excess of free water administration.
c. Postoperative increase of antidiuretic hormone secretion.
d. Decreased oral intake of sodium.
e. Some drugs which cause water retention.

18
Key Answers

19
1. D
2. C
3. C
4. E
5. E
6. C
7. C
8. A
9. C
10. D
11. C
12. D
13. A
14. D
15. A
16. D
17. B
18. B
19. C
20. B
21. A
22. E
23. D
24. C
25. E
26. B
27. B
28. E
29. C
30. C
31. C
32. E
33. B
34. C
35. B
36. C
37. B
38. A
39. E
40. E
41. B
42. C
43. C
44. D
45. C
20
46. B
47. E
48. C
49. E
50. C
51. C
52. E
53. C
54. D
55. B
56. D
57. D
58. C
59. D
60. B
61. D
62. C
63. E
64. A
65. A
66. E
67. A
68. B
69. C
70. B
71. A
72. D
73. B
74. E
75. D
76. D
77. B
78. D
79. E
80. C
81. D
82. B
83. A
84. A
85. E
86. A
87. E
88. E
89. C
90. A
21
91. C
92. A
93. B
94. C
95. D
96. E
97. D
98. D
99. A
100. D

22
1. which of the following amino acids is a critical nutrient supporting
the immune function:
a) Taurine.
b) Branched chain amino acids (BCAA).
c) Alanine. *
d) Arginine.
e) Tyrosine.

2. The optimal initial treatment of aspiration pneumonia is:


a) Bronchoalveolar lavage with antacids.
b) Bronchoalveolar lavage with saline.
c) Parenteral corticosteroids.
d) Maintenance of oxygenation. *
e) Early prophylactic antibiotics.

3. All are true about ultrasound, EXCEPT:


a) Can detect a lesion less than 1 cm in the liver. *
b) Is useful about differentiating a thyroid cyst.
c) It helps in the diagnosis of acute appendicitis.
d) Its not the best modality to visualize the pancreas.
e) Investigation of choice in GB stones.
4. Which statement is correct regarding neurapraxia:
a) The injury involves the axon without loss of continuity of nerve
sheath.
b) Momentary loss of conduction through the nerve without any
organic lesion. *
c) Partial or complete division of the nerve.
d) Spontaneous recovery is impossible.
e) Wallerian degeneration occurs.

5. The most important factor in the development of pressure ulcers in


a spinal cord injury patient is:
a) Malnutrition.
b) Septic episodes.
c) Anemia.
d) Local pressure. *
e) Diminished sensation.
6. The earliest sign of Volkmann's ischemic contracture is:
a) Absence of the radial pulse.
b) Pain on passive finger extension. *
c) Loss of voluntary finger motion.
d) Absent sensation in the median nerve distribution.
e) Pallor of the nail beds.

7. Regarding the oxygen dissociation curve, one is TRUE:


a) Is shifted to the left by increase 2,3-DPG.
b) Is sigmoid shape for both hemoglobin and myoglobin.
c) Is shifted to the left when one ascends to high altitude.
d) Is shifted to the right by an increase in the PH.
e) Is ideally 50% saturated at a PO2 of 28mmHg. *

8. Regarding insulin, all are true EXCEPT:


a) It is a polypeptide composed of two amino acid chains.
b) Its secretion is affected by glucagon.
c) It stimulates gluconeogenesis. *
d) It has little or no effect on the uptake of glucose in the brain.
e) If lacking, it may be a cause for fatty liver.

9. Which of the following statements regarding hemorrhoids is NOT


TRUE:
a) Hemorrhoids are specialized "cushions" present in every one to aid
continence.
b) External hemorrhoids are covered by skin, while internal
hemorrhoids are lined by mucosa.
c) Pain is often associated with uncomplicated hemorrhoids. *
d) Haemorrhoidectomy is reserved for third and fourth degree
hemorrhoids.
e) They are classified and treated according to the degree of their
symptoms.
10. Infection affects wound healing by all the following mechanisms
EXCEPT:
a) Prolonging oedema.
b) Decreasing tissue PO2.
c) Increasing collagenolysis.
d) Decreasing the inflammatory phase. *
e) Increasing the inflammatory phase.
11. All the followings may be linked to the etiology of malignant
melanoma EXCEPT:
a) Fair complexion.
b) Intermittent sun exposure.
c) Positive family history.
d) Junctional nevus. *
e) Dysplastic nevus.

12. The most reliable indicator of the adequacy of burn resuscitation is:
a) Central venous pressure.
b) Pulmonary capillary wedge pressure.
c) Urine output. *
d) Blood pressure and heart rate.
e) Mental status.
13. The most common site for squamous cell carcinoma of the lip is the:
a) Upper lip midline.
b) Upper lip laterally.
c) Lower lip midline.
d) Lower lip laterally. *
e) Oral commissure.
14. As the amount of total body fat increase, the total body water:
a) Increases.
b) Decrease. *
c) Unchanged.
d) Depends on patient's age.
e) Depends on patient's sex.
15. One week of starvation results in a loss of approximately 100 gm of
nitrogen (13.8 gm N/ day). This is associated with protein loss of:
a) 625 gm. *
b) 100 gm.
c) 1000 gm.
d) 50 gm.
e) 16 gm.
16. The HIGHEST potassium content is in:
a) Saliva. *
b) Gastric juice.
c) Bile.
d) Ileal.
e- Duodenal
17. During vessel wall injury, the initial step in clot formation is:
a) Local mast cells release of adenosine diphosphate
(ADP).
b) Fibrin polymerization.
c) Plasminogen activation.
d) Platelet adherence to sub endothelial collagen. *
e) None of the above.
18. Postoperative staphylococcal wound infection usually detected in:
a) 1st to 2nd day.
b) 3rd to 4th day.
c) 5th to 8th day. *
d) 9th to 14th day.
e) After the 14th day.
19. A palpable radial pulse indicates a systolic blood pressure greater
than:
a) 40 mmHg.
b) 60 mmHg.
c) 80 mmHg. *
d) 100 mmHg.
e) 120 mmHg.
20. What is the first parameter to change in early hypovolemic shock?
a) Systolic blood pressure.
b) Pulse rate. *
c) Diastolic blood pressure.
d) Respiratory rate.
e) Level of consciousness.
21. A previously healthy 18 years old man involved in automobile
accident. He lost consciousness, but regains it and appeared normal.
An hour later he became somnolent and has weakness on his right
side, with dilated pupil on the left side. Most likely he has:
a) Acute epidural hematoma. *
b) Acute subdural hematoma.
c) Subarachnoid hemorrhage.
d) Intraventricular hemorrhage.
e) Brain concussion.
22. Concerning factors that increase the risk of pulmonary embolism,
all are correct EXCEPT:
a) Superficial phlebitis one year ago. *
b) Estrogen therapy.
c) Obesity.
d) Pregnancy.
e) Deep venous thrombosis one year ago.

23. The TRUE statement regarding small intestine is:


a) The entire small intestine is intraperitoneal.
b) The jejunum is longer in length, larger in diameter, and thinner-
walled than ileum.
c) The muscularis, the muscle layer, provide strength for placement of
sutures for creation of a bowel anastomosis.
d) Peyer’s patches are most prominent in the distal ileum *
e) The marginal artery of Drummond provides blood supply to the
duodenum.

24. A 40 years old female had a subtotal thyroidectomy, several hours


later she complained of difficulty in breathing and she had stridor
and a markedly swollen, tense neck wound the first step in the
management of this patient should be to:
a) Intubate with an endotracheal tube.
b) Perform a tracheostomy.
c) Control the bleeding site in the operating room.
d) Open the wound and evacuate the hematoma at bed side *
e) Aspirate the hematoma.

25. Patients with acute pancreatitis who are at increased risk to develop
complications include those with each of the following EXCEPT:
a) Ransons score> 3.
b) An APACHI II score >12.
c) Poorly enhancing areas of 50% of the pancreas on a dynamic CT
scan.
d) Amylase values more than eight times upper limits of normal. *
e) Partial obliteration of the lesser sac and acute fluid collections near
the splenic hilum and inferior to pancreatic head.
26. Regarding acute suppurative ( toxic ) cholangitis, one is TRUE :
a) Third generation cephalosporin are the antibiotic of choice for
patients with renal impairment.
b) Urgent decompression of the biliary tree is indicated. *
c) Emergency surgical decompression is indicated for patients with
known cholilithiasis.
d) The catheter for percutaneous drainage should be placed through the
common duct into the duodenum.
e) Nasobiliary tube decompression is an effective form of drainage for
obstructing lesion in the upper third of the common duct.
27. The most common presenting symptom in colorectal carcinoma in
patients under age of 40 is:
a) Abdominal pain.
b) Weight loss.
c) Back pain.
d) Rectal bleeding. *
e) Abdominal distension.

28-A 24 years old man has appendectomy, histopathological report


notes acute inflammation with a 1.2 cm carcinoid tumor on the mid of
appendix. This patient should have:

f) No further treatment. *
g) Chemotherapy.
h) Right hemicolectomy.
i) Serial urinary 5- hydroxyindole acetic levels.
j) Regional radiation therapy.
29- Gastric ulcers are:
A- Malignant when located in the greater curvature.
B- More common in women.
C- Most commonly complicated by perforation.*
D- Associated with hypersecretion of acid in over 90% of cases.
E- Not associated with Helicobacter pylori.
30- Which is the most commonly cultured hospital acquired organism in
critical care patients with aspiration pneumonia?
a- Streptococcus pneumoniae
b- Staphylococcus aureus
c- Anaerobic species
d- Pseudomonas aeruginosa*
e- Haemophilus influenzae
31- Which of the following will immediately delay or cancel an elective
surgical case if not obtained appropriately preoperatively?
a- CBC
b-Urinanalysis
c- CXR
d- Informed consent*
e- ECG
32- The most common cause of nipple discharge is
a- carcinoma
b- fibrocystic disease
c- intraductal papilloma*
d- duct-ectasia
e- trauma
33- ALL OF THE FOLLOWING ARE SINGS OF ATROPINE
POISONING EXCEPT
a) dry mouth
b) red face
c) small pupils *
d) tachycardia
e) drowsiness
34- PAIN IS CARRIED TO THE BRAIN BY THE FOLLOWING TRACT
a) spino cerebellar tract
b) spino thalamic tract *
c) cortico- spino tract
d) reticular tract
e) olvospinal tract
35- A CETYLE CHOLINE IS THE CHEMICAL TRANSMITTER IN
a) all; pre and post ganglionic sympathetic system
b) all; pre and post ganglionic parasy pathetic system *
c) all post ganglionic sympathetic system
d) in adrenal medulla
e) all pre ganglionic sympathetic system
36- ALL OF THE FOLLOWING AFFECTING NERVE
CONDUCTION EXCEPT:
a) myelination of the nerve
b) nerve fiber diameter
c) hypoxia
d) autonomic nervous system *
e) inter nodal distance
37- THE MOST HELPFULL CLINICAL SIGN FOR SPINAL CORD TUMOUR
AT T10 LEVEL IS:
a) exaggerated of knee jerk
b) anesthesia up to umbilicus level *
c) urine retention
d) anesthesia up to hypogastrium
e) dissociated sensory loss
38- THE EARLY TREATMENT OF CERVICAL SPINES INJURY INCLUDES
ALL OF THE FOLLWING EXEPT:
a) immobilization by neck collar
b) the patient should be covered
c) lateral spine x-ray while patient on stretcher
d) change the patient position to take off his clothes in E.R *
e) treatment of spinal shock
39- THE MOST IMPORTANT FUNCTION OF INTERVERTEBRAL DISC IS:
a) Preserve spinal curvature
b) Protects spinal cord
c) Shock absorber *
d) Transmit weight of the body to the lower limbs
e) Has no significant function on adult age
40 – BLOOD SUPPLY OF SPINA LCORD IS USUALLY COME FROM :
a) vertebral arteries only
b) ant , post spinal arteries and radicular arteries *
c) vertebral arteries and abdominal aorta
d) vertebral arteries and ascending cervical arteries only
e) radicular arteries from the aorta only
41- FRACTURE OF THE L 4 VERTEBRAE WITH LOWER LIMBS
WEAKNESS IS DUE TO :
a) spinal cord contusion at that level
b) only dura injury
c) dura and nerves injury *
d) spinal cord , dura and nerves injury
e) spinal concussion without any nerve injury
42 – BARO RECEPTORS IS LOCATED IN :
a) skin
b) wall of the great vessels in the neck *
c) heart
d) in the brain at the medulla level
e) in the vessels of the brain
43- THE MOST SERIOUS COMPLICATION WHICH AFFECT THE
SEQUALE OF HEAD INJURY IS DUE TO:
a) brain infection and brain abscess
b) brain hypoxia *
c) secondary bleeding
d) decrease intracranial pressure
e) CSF leakage
44-A LADY IN CHILDBEARING AGE WITH REPEATED PREGNANCES
AND HAS POOR INTAKE OF ANIMAL DIET IS PRONE TO:
Osteoporosis
a) Osteomalacia *
b) Rickety bone
c) Vitamin B deficiency
d) Compressed spinal fractures
45- ALL OF THE FOLLOWING STRUCTURES PASSING UNDER THE
FLEXOR RETINACULUM EXCEPT:
Flexor Digitorum longus
a) Flexor Digitorum brevis
b) Median nerve
c) Flexor indices
d) Palmaris longus *
46- THE ORIGION OF SYMPATHETIC SYSTEM FROM THE SPINAL CORD
IS:
Cranio cervical outflow
a) Thoraco lumbar outflow *
b) Cranio sacral outflow
c) Only lumbar outflow
d) Whole spinal cord
47-METABOLIC ACIDOSIS MAY BE BROUGHT BY:
a) loss of CO2 by increase ventilation
b) retention of CO2 by respiratory obstruction
c) persistent vomiting
d) absorption of excessive amount of Na HCO3
e) excessive exercise *
48-THE MOST POTENT STIMULANT OF RESPIRATION WOULD BE:
a) a two fold increase in PCO2 of inspired air *
b) a two fold increase in PO2 of inspired air
c) a 50% decrease in PCO2 of inspired air
d) a 50% decrease in PO2 of inspired air
e) a & d are equally potent stimuli
49-The psoas major muscle
a. flexes the thigh at the hip. *
b. extends the thigh at the hip.
c. adducts the thigh at the hip.
d. abducts the thigh at the hip.
e. assists in full contraction of the diaphragm

50-The renal arteries typically branch from the abdominal aorta a


the level of the:

a. 12th thoracic vertebral body.


b. first lumbar vertebral body.
c. second lumbar vertebral body. *
d. third lumbar vertebral body.
e. fourth lumbar vertebral body.

51-A 20–year–old man is undergoing retroperitoneal dissection


for a testicular germ cell tumor. The inferior mesenteric artery is
divided during reflection of the intestines to expose the
retroperitoneum. This can be expected to result in:

a. ischemia of the descending colon.


b. ischemia of the sigmoid colon.
c. ischemia of the rectum.
d. ischemia of the splenic flexture
e. none of the above. *

52-The cremaster muscle is innervated by:

a. the ilioinguinal nerve.


b. the iliohypogastric nerve.
c. the obturator nerve.
d. the genital branch of the genitofemoral nerve. *
e. the femoral branch of the genitofemoral nerve.
53-As one proceeds outward from the adrenal medulla, the three
separate functional layers of the adrenal cortex are, in correct
order:

a. the zona reticularis, zona fasciculata, then zona glomerulosa. *


b. the zona fasciculata, zona reticularis, then zona glomerulosa.
c. the zona glomerulosa, zona fasciculata, then zona reticularis.
d. the zona glomerulosa, zona reticularis, then zona fasciculata.
e. the zona reticularis, zona glomerulosa, then zona fasciculata.

54-Proceeding from posterior to anterior, the structures


encountered in the renal hilum are, in correct order:

a. the renal artery, renal vein, and renal pelvis.


b. the renal pelvis, renal artery, and renal vein. *
c. the renal pelvis, renal vein, and renal artery.
d. the renal vein, renal artery, and renal pelvis.
e. the renal artery, renal pelvis, and renal vein.

55-During inguinal hernia repair in a male patient, the ilioinguinal


nerve is injured in the canal, which will most likely produce:

a. anesthesia over the dorsum of the penis.


b. anesthesia over the pubis and scrotum, and loss of cremasteric
contraction.
c. anesthesia over the pubis and anterior scrotum only. *
d. anesthesia over the anterior and medial thigh.
e. anesthesia over the pubis only.

56-Hematuria is distinguished from hemoglobinuria or


myoglobinuria by:

a. dipstick testing.
b. the simultaneous presence of significant leukocytes.
c. microscopic presence of erythrocytes. *
d. examination of serum.
e. evaluation of hematocrit.
57-Glucose will be detected in the urine when the serum level is
above:

a. 75 mg/dl.
b. 100 mg/dl.
c. 150 mg/dl.
d. 180 mg/dl. *
e. 225 mg/dl.

58-What proportion of the cardiac output is delivered to the


kidney?

a. 5%
b. 20% *
c. 50%
d. 85%
e. 100%

59-The predominant buffering system in humans is:

a. bicarbonate. *
b. titratable acids.
c. ammonium (NH4+).
d. sodium lactate
e. phosphate

60-After a 7–hour–long, complex urethral reconstruction


performed in the extended lithotomy position, the patient has
severe thigh and buttock pain. The creatine phosphokinase
levels are dramatically elevated. To prevent ARF, the next step
should be:

a. dopamine infusion.
b. plasmapheresis.
c. dobutamine infusion.
d. forced alkaline diuresis. *
e. percutaneous nephrostomy.
61-The renal structure at greatest risk for ischemic injury is the:

a. vasa recta.
b. cortical collecting duct.
c. juxtaglomerular apparatus.
d. medullary thick ascending loop of Henle. *
e. distal convoluted tubule.

62-Acute pyelonephritis is best diagnosed by:

a. chills, fever, and flank pain. *


b. bacteriuria and pyuria.
c. focal scar in renal cortex.
d. delayed renal function.
e. vesicourethral reflux.

63-The most reliable early clinical indicator of septicaemia is:

a. chills.
b. fever.
c. hyperventilation.*
d. lethargy.
e. change in mental status.

64-Which of the following is true regarding testosterone?

a. Testosterone is synthesized by the Sertoli cells of the testes.


b. Testosterone is synthesized by the Leydig cells of the testes.*
c. Testosterone is a direct precursor of pregnenolone.
d. 5α–Reductase is an enzyme that converts DHT into testosterone.
e. Aromatase converts estrogens into testosterone

65-Calcium is maximally absorbed in which portion of the


gastrointestinal tract?

a. Stomach
b. Jejunum
c. Jejunum and proximal ileum *
d. Ileum
e. Ascending colon
66- the most common type of congenital diaphragmatic hernia is caused
by:
a. a defect in the central tendon
b. eventration of the diaphragm in the fetus
c. a defect through the space of Larry
d. a defect through the pleuroperitoneal fold. *
e. all of the above.
67- The calorie-nitrogen ratio for infant should be maintained at:
a. 75:1
b. 100:1
c. 50:1
d. 150:1 *
e. 25:1

68- A patient has undergone an ileal resection . Which of the following


conditions would he be least likely to develop?

a- alopecia *

b- megaloblastic anemia

c- nephrolithiasis

d- cholelithiasis

e- steatorrhea

69.With regard to Hirschsprung’s disease ( aganglionosis), all the


following are true except:
a- It is more common in males.
B. It may be complicated by enterocolitis.
C. Barium enema study may be normal.
D. It is best diagnosed by full thickness rectal biopsy.
E .Surgery can’t be accomplished without colostomy. *

70. In regard to malignant hyperthermia, one is true.


A. It triggered by Depolarizing muscle relaxing agents *
B. Non polarizing muscle agents are contra indicated .
C. Dantroline must be given after induction as prophylaxis.
D. Manitole and fluid support are not useful to prevent renal failure.
E. Alkalosis is the main metabolic disturbance.
71. Incompatible blood transfusion during surgery under general
anesthesia, which of the following is NOT true.
A. Unexplained bleeding.
B. Hematuria may be present.
C. Skin rash may be seen
D. Hypertension is well know problem in spite transfusion *
E. Volume support is mandatory.
72. The most common cause of death in Paediatric age group is :
A. Infection
B. Trauma. *
C. Malignancy.
D. Congenital anomalies.
E. Malnutrition.
73-In the management of pyloric stenosis, which of the following is true.
A. Acidosis should be corrected first.
B. Chloride level is normal in spite alkalosis.
C. Hypokalaemia should be corrected. *
D. Usually there is compensatory hypocarbia.
E. Paradoxical aciduria is not a feature.
74 - The ligamentum teres represents an obliterated
a- Ductus venosus
b- Ductus arteriosus
c- Internal iliac artery
d- Umbilical vein *
e- porta hepatis
75- Tributaries of the portal vein include all of the the following except:
a- Superior mesenteric
b- Para umbilical
c- uterine *
d- pyloric
e- lienal ( splenic )

76- The following statements are true regarding the liver anatomy
except :
a- Is attached to the diaphragm and anterior abdominal wall by the Falciform
ligament
b- Is totally covered by peritoneum *
c- Drains by hepatic veins into the inferior vena cava
d- Has a lymph drainage to both the mediastinal and porta hepatic nodes
e- Is directly related to the right suprarenal gland
77- An animal is in negative nitrogen balance when :
a- The intake exceeds output
b- New tissue is being synthesized
c-Output exceeds intake *
d- Intake is equal to output
e- The urine is nitrogen – free
78- Bilirubin secreted into the intestine is subjected to enzymatic
degradation , the final product being :
a- Biliverdin
d- Bilirubinogen
c- Urobilinogen
d- Stercobilin *
e- Mesobilirubinogen
79- In mammals , Norepinephrine is synthesized from :
a- Pyruvate
b- Arginine
c- Catechol
d- Tyrosine *
e- Tryptamine
80- Tissue graft within the same individual is known as :
a- Allograft
b- Isograft
c- Autograft *
d- Xenograft
e- Non of the above
81- The most effective means of sterilization by heat is :
a- steam
b- Boiling
c- Hot air
d- Fractional sterilization
e- Steam under pressure *
82- One of the following statements is true regarding Exotoxins:
a- Heat stable
b- Lipopolysaccharide in nature
c- part of cell wall of becteria
d- protein in nature *
e- less potent than endotoxins
83- Gall stones associated with pernicious anemia are most likely to be :
a- pure cholesterol
b- Calcium bilirubinate *
c- Calcium carbornte
d- Mixed gall stones
e- combined gall stones

84- In familial polyposis coli, polyps tend to appear most frequently :

a- At birth
b- During the first decade
c-during the second and third decades of life *
d- During the fourth decades of life
e- After age of 40

85- The substance involved with protein synthesis

a- DNA
b- RNA *
c- Adenine
d- Guanine
e- Cytosine

86- In surgical skin wound 80% of tensile strength of unwounded skin is


restored by :
a- 1 week
b-2 weeks
c- 4 weeks
d- 12 weeks *
e- 24 weeks

87- Calcitonin sometimes is elaborated by which one of the following


tumors :
a-Medullary carcinoma of the breast
b- Medullary carcinoma of the thyroid *
c- Giant cell carcinoma a of pancrease
d- Carcinoid tumor of appendix
e- Ganglioneuromas
88- Normal " prothrombin time " as measured by usual clinical tests
depends on normal levels of all of the following factors excepts :
a- Factor II
b- Factor V
c- Factor VII
d- Factor VIII *
e- Factor X
89- Side effects of large doses of cortisone include all of the following
except :
a- Hirsutism and osteoporosis
b- Mental changes
c- Recurrence or appearance of new peptic ulcers
d- Increased susceptibility to infection
e- Depletion of serum sodium *
90- The average daily loss of water through the lungs and skin
( insensible water loss ) is approximately :
a- 10 ml
b- 100ml
c- 1000ml *
d- 2000ml
e- Any of these depending on circumstances

91- The most common cause of gastric outlet obstruction in adults is;
A- Adenocarcinoma of the stomach.*
B- Hypertrophic pyloric stenosis.
C- Duodenal stenosis secondary to peptic ulceration.
D- Bezoar.
E- Gastric lymphoma.
92- About colorectal carcinoma associated with Crohn’s disease, one is
TRUE:

A- It is usually common in women.


B- The frequency of carcinoma is similar in patients with
extensive, long standing ulcerative colitis.*
C- The right colon is involved in over 70% of patients.
D- The mean age of patients with colorectal carcinoma is 35.
E- The prognosis is the same for Ca associated with Crohn's dis
and Ca in colon without Crohn's dis,
93- Regarding pancreatic carcinoma all are true Except:-

A- 90% are ductal adenocarcinomas

B- Less than 20% occur in the head of the gland *

C- The usual presentation is with pain, weight loss and obstructive jaundice

D- Ultrasound has a sensitivity of 80-90% in the detection of the tumour

E- Less than 20% of patients are suitable for curative surgery

94- Heparin all are true Except:-

A-Is a heterogeneous mixture of sulphated polypeptides *

B- Potentiates the actions of antithrombin III

C- Has a half life of 90 minutes.

D-Can be reversed by protamine sulphate

E-Can induce an idiosyncratic thrombocytopenia

95- All of the following affect Gastrin release EXCEPT:


a. antral acidification.
b. antral alkalization.
c. carbohydrates in antrum. *
d. gastric distension.
e. somatostatin release.

96- What portion of the colon absorb the majority of fluid?


a. ascending colon *
b. transverse colon
c. descending colon
d. sigmoid colon
e. rectum
97- The concentration of which electrolyte in pancreatic secretion
increases as the rate of secretion increases?
a. sodium
b. potassium
c. chloride
d. bicarbonate *
e. calcium

98- All of the following promote LES relaxation EXCEPT:


a. atropine *
b. nitric oxide
c. cholecystokinin (CCK)
d. gastric distention
e. pharyngeal stimulation

99- Intraabdominal adhesions following abdominal surgery have been


associated with all of the following EXCEPT:
a. small bowel obstruction
b. infertility
c. chronic pelvic pain
d. intestinal malabsorption *
e. increased risk for enterotomy on subsequent laparatomy

100- Which of the following is true regarding small bowel carcinoid


tumors?
a. most are biochemically atypical tumors lacking the enzyme dopa
decarboxylase
b. they are most common GI carcinoid tumors
c. regional lymph node involvement is common in tumors less than 1 cm in
size *
d. diagnosis is frequently made in patients prior to surgery
e. among carcinoid tumors, they are associated with the lowest rate of
second primary malignancy
KEY 1 C
2 D
3 A
4 B
5 D
6 B
7 E
8 C
9 C
10 D
11 D
12 C
13 D
14 B
15 A
16 A
17 D
18 C
19 C
20 B
21 A
22 A
23 D
24 D
25 D
26 B
28 A
29 C
30 D
31 D
32 C
33 C
34 B
35 B
36 D
37 B
38 D
39 C
40 B
41 C
42 B
43 B
44 B
45 E
46 B
47 E
48 A
49 A
50 C
51 E
52 D
53 A
54 B
55 C
56 C
57 D
58 B
59 A
60 D
61 D
62 A
63 C
64 B
65 C
66 D
67 D
68 A
69 E
70 A
71 D
72 B
73 C
74 D
75 C
76 B
77 C
78 D
79 D
80 C
81 E
82 D
83 B
84 C
85 B
86 D
87 B
88 D
89 E
90 C
91 A
92 B
93 B
94 A
95 C
96 A
97 D
98 A
99 D
100 C

27---------------D
1- Arterial injuries that can be safely ligated in the unstable trauma
patient include all except :

*a- SMA
b- radial art
c- inferior mesentric art
d- internal iliac artery
e- celiac art

2-in which of the following venous injuries is ligation most


tolerated :

*a- infra renal vena cava


b- supra renal vena cava
c- common femoral vein
d- popliteal vein
e- innominate vein

3-following blunt abdominal trauma , mandatory exploration is


indicated for any non expanding hematoma identified on CT –
scan in which of the following areas :

a- Rt . perinephric
*b- mid line infra mesocolic
c- lateral pelvic area
d- retro hepatic
e- lt. perinephric

4-which structure is the most sensitive to hypoxia?

a-skin
b- bone
*c-non myelinated nerve
d- myelinated nerve
e- skeletal muscle

Pulmonary artery catheter is used to measure which variable


-directly :

a-cardiac index
*b- systemic vascular resistance
c- mixed- venous oxygen saturation
d- left ventricular end diastolic index
e- pulmonary vascular resistance index
6-An example of anti-inflammatory cytokine is:
a-IL-2
b-IFN-y
c-Lymphotoxin-alpha
*d-IL-4
e-TNF-alpha

7-Trendelenburg position considered a viable treatment


option for which category of shock?

a-Cardiogenic
*b-Neurogenic
c-Hypovolemic
d-Septic
e-Cardiac compressive

8-All of the following symptoms are seen with


hypermagnesemia except:

*a-Tachypnea
b-Depression of reflexes
c-Arrythmias
d-hypotension
e-Central nervous system depression

9-The most likely cell of origin for gastrointestinal


stromal tumor(GIST):

*a-Interstitial cells of Cajal


b-Kulchitsky cells
c-Myofibroblasts
d-G-cells
e-Paneth cells
10-Which of the following regarding inflammatory bowel
disease is true:

a-Bloody diarrhea is more common in crohns disease than in


ulcerative colitis.
*b-Associated extra intestinal disease is less common in
ulcerative colitis than in crohns disease
c-Crohns disease frequently involves the entire colon
d-Crohns is more commonly associated with primary
sclerosing cholangitis than is ulcerative colitis.
e-Surgical intervention for ulcerative colitis is limited to
management of complications of the primary disease process.

11-Each of the following is associated with familial


polyposis syndrome except:

a-Gardners syndrome
b-Turcots syndrome
*c-MEN type 2b
d-Peutz-Jeghers syndrome
e-Juvenile polyposis syndrome.

12-Surgical exploration for patient with primary


hyperparathyroidism reveals all four glands to be:
enlarged.What is the most appropriate way to manage?

a-Closure with localization study


b-Incision biopsy of all glands
*c-Subtotal parathyroidectomy
d-Excision of the largest enlarged gland
e-Selective venous PTH sampling.

13-The most common clinical manifestation of MEN 1 is:

a-Hypoglycemia.
b-Renal stones
c-Galactorrhea
d-Osteoporosis
*e-Peptic ulcer disease.
14-In MEN 1, the most common pancreaticodoudenal
tumors are
:
a-Gastrinomas
*b-Non functioning tumors
c-Insulinomas.
d-Somatostatinomas.
e-VIPomas.

15-Cholecystokinin:
*a-Relaxes the sphincter of oddi.
b-Inhibits gastric acid secretion.
c-Inhibits gall bladder contraction.
d-Causes mesenteric vasodilatation.
e-Inhibits pancreatic exocrine secretions.

16-Rectus sheath hematomas


*a-Can be caused by coughing.
b-Are rarely associated with anticoagulative therapy.
c-Usually occur at the semicircular line of Douglas at the entry
site of superior epigastric artery into the rectus sheath.
d-Are infrequently palpable on physical examination.
e-Usually require operative drainage.

17-Most primary and metastatic tumors to liver derive


nearly all their vascular inflow from branches of:
A-Portal vein.
b-Collateral circulation.
*c-Hepatic artery.
d-Celiac axis.
e-Unnamed branches of aorta.
Which of the following is not a complication of Caroli
-disease:
a-Biliary stone formation.
b-Recurrent cholangitis.
c-Septicemia.
d-Cholangiocarcinoma.
*e-Renal disorders.

19-Which of the following is not considered a risk factor


for gall bladder cancer?
a-Gall stones>3cm in size.
*b-Multiple small gall stones.
c-porcelain gall bladder
d-Biliary salmonella typhi infection.
e-Gall bladder adenoma.

20-Which is true about surgical anatomy of pancreas:


a-The head lies over the first lumber vertebra.
b-The accessory duct enters the second part of duodenum.
c-The superior mesenteric vessels pass behind the uncinate
process.
*d-The SMA lies to the right of the superior mesenteric vein.
e-The superior mesenteric vein join the portal vein behind the
head of pancreas.

21-Most common indication for splenectomy for a red cell


enzymatic defect is:
a-Hereditary spherocytosis.
b-G6PD.
*c-Private kinase deficiency.
d-Hereditary high red blood cell phosphatidyl choline anemia.
e-Cold-agglutinin syndrome.
22-Which of the following is true of retroperitoneal soft
tissue sarcomas:

a-The most common histological cellular type is malignant


fibrous histiosarcoma.
b-Pre-operative biopsy is contraindicated because of tumor
seeding risk.
*c-Resection of contaguous organs that are not involve with
tumor is indicated to obtain adequate margins.
d-Death usually results from distant metastases.
e-Lung is the most common site of distant metastases.

23-Which of the following structures form the medial


border of the (triangle of doom) to be avoided in
laparascopic inguinal herniorrhaphy?

a-Inferior epigastric vessels.


*b-Vas deferens.
c-Spermatic vessels.
d-Iliac vessels.
e-Lateral femoral cutaneous nerve.

24-A 45 year old man who has been HIV positive for 15 years has a
painful 3-cm neck mass. A 2 week course of antibiotics does not change
the mass. Biopsy will most reveal:

a-Hodgkin’s lymphoma.
*b-B cell lymphoma.
c- T cell lymphoma.
d- Metastatic lung carcinoma.
e- Kaposi’s sarcoma.

25-Risk factors for male breast cancer include:


a-Gynecomastia
b-BRCA 1 gene mutation
*c- BRCA 2 gene mutation
d- Anabolic abuse.
e- Cannabis abuse.
26-A 2-cm gastric ulcer in the antrum of the stomach is associated with all
of the following except:

a-Helicobacter pylori.
*b-Increased acid secretion.
c- Malignancy.
d-nonsteroidal anti-inflammatory drugs.
e-Atrophic gastritis.

27-The most common presenting clinical sign in Budd-Chiari syndrome


is:
*a-Ascitis.
b-Esophageal varices.
c-Jaundice.
d-Encephalopathy.
e-Hemorroids.

28-Percutaneous image guided drainage of intra abdominal abscesses is


LEAST successful in which of the following sites?

a-Subphrenic.
b-Hepatic.
*c-Peripancreatic.
d-Diverticular.
e-Appendiceal.

29-In the workup on a patient for possible appendicitis, CT scanning should be


performed:
a) Before consulting the surgeon, by the emergency physician
b) In patients with equivocal physical findings
c) Routinely, in all patients with right lower quadrant pain
d) With equal frequency in men and women
e) Never

b—
30-The gastric mucosal cell that secretes intrinsic factor is the
a) G cell
b) Parietal cell
c) D cell
d) Enterochromaffin-like cell
d) Chief cell

b—

31-Hypertrophic pyloric stenosis is likeliest to occur in a(n)


a) Firstborn child
b) African-American infant
c) Child 6 to 9 months of age
d) Female infant
e) Infant born prematurely

a—

32-Overwhelming postsplenectomy infection (OPSI) a) Occurs more frequently after


resection for trauma than hematologic disease
b) Occurs with equal frequency in children and adults
c) Is most frequently caused by Streptococcus pneumoniae
d) Usually occurs within 2 years after splenectomy
e) Generally has an identifiable site of infection

c—

33-The characteristic feature of Crohn's colitis that best distinguishes the clinical
entity from ulcerative colitis is
a) Perianal disease
b) Rectal bleeding
c) Risk of malignancy
d) Obstructive symptoms
e) Pseudopolyps

a—

34-Type 1 gastric ulcers are


a) At the incisura
b) Along the greater curvature
c) Prepyloric
d) Associated with simultaneous duodenal ulcers
e) Close to the esophagogastric junction

a—
35-Radiation enteritis
a) Usually presents with perforation
b) Is caused by thrombosis of mucosal vessels
c) Occurs after 3,000 cGy of abdominal radiation
d) Routinely requires operative therapy
e) Is likely in patients who have undergone laparotomy

e–

36-The small bowel tumor with the greatest propensity for bleeding is
a) Carcinoid
b) Lymphoma
c) Adenocarcinoma
d) Hamartoma
e) Leiomyoma

e—

37-Gastric stump cancers (gastric adenocarcinomas in patients who


underwent gastric resection)

a. Are the consequence of Helicobacter pylori infection


b. Are generally of the polypoid type
c. Have a 5-year survival rate of 40%
d. Occur within 5 years after gastrectomy
e. Are caused by bile reflux

e—

38-A grade III laceration of the spleen is characterized by

a. Intraparenchymal hematoma > 5 cm


b. Capsular tear 1-3 cm deep
c. Hilar vessel disruption
d. Subcapsular hematoma involving 10%-50% of surface area
e. Active hemorrhage

a—
39-The spleen filters all of the following particles/cells EXCEPT

a. Malformed erythrocytes

b. T lymphocytes

c. Malarial parasites

d. Streptococcus pneumoniae

e. Platelets

b—

40-Acute appendicitis

a. Occurs most commonly in the second and third decades of life


b. Can be cured readily by antibiotics
c. Is most commonly caused by a fecalith
d. Carries an overall mortality rate of 7%
e. Induces leukocytosis in 90% of patients

41-The stomach bed does not include :

A. Splenic artery.
B. Celiac trunk.
C. Transverse mesocolon.
D. Left adrenal gland.
E. Neck of the pancreas.

E
42-which of the following is the strongest of all other risk factors in the
development of Gastric carcinoma :

A. Helicobacter pylori.
B. Atrophic gastritis.
C. Blood group A.
D. Pernicious anemia.
E. Low socioeconomic class.

43-whilst performing a small bowel resection for strictures following crohn”s


disease, you realize that on inspection, there are marked differences between
jejunal and ileal anatomy. Such differences include all the following Except:

A. Wider lumen in the jejunum.


B. Less lymphatics in the jejunal mesentry compared to ileal.
C. More prominent and multiple arcades of vessels in the ileum.
D. Thicker wall of the jejunum.
E. Thicker and more fat-laden mesentry increasing towards the ileum.

44-A 54 years old woman is referred to your surgical team with a diagnosis of
small bowel obstruction. Which one of the following clinical signs would you
look for in trying to identify the commonest cause of this condition :

A. surgery scar.
B. Lump in the groin above & medial to the pubic tubercle.
C. Lump in the groin below & lateral to the pubic tubercle.
D. Cachexia & nodule at the umbilicus.
E. Circumoral pigmentation & a family history of previous obstruction.

45-True statements regarding appendiceal neoplasms include which of the


following?

A. Carcinoid tumors of the tip of the appendix less than 1.5 cm are adequately treated
by simple appendectomy
B. Appendiceal carcinoma is associated with secondary tumors of the GI tract in up to
60% of patients
C. Survival following right colectomy for a Dukes’ stage C appendiceal carcinoma is
markedly better than that for a similarly staged colon cancer at 5 years
D. Mucinous cystadenocarcinoma of the appendix is adequately treated by simple
appendectomy, even in patients with rupture and mucinous ascites
E. Up to 50% of patients with appendiceal carcinoma have metastatic disease, with
the liver as the most common site of spread
:A

46-All of the following statements about carcinoma of the gallbladder are correct
Except :

A. The neoplasm usually starts in the cystic duct and neck of the gallbladder.
B. It is found more commonly in women than men.
C. It is associated with the presence of gallstones in > 85% of cases.
D. Prognosis is generally poor with < 1 year survival with local invasion.
E. Chemotherapy and radiotherapy do not alter disease progression.

47-All of he following statements are true regarding diverticular disease Except :

A. It is found more commonly in the developed world.


B. Surgical treatment is usually unnecessary in acute uncomplicated cases.
C. Diverticulae are more commonly found in the descending colon.
D. Perforation and fistula formation can result from an attack of acute diverticulitis.
E. Resolution of the diverticulae can occur with high fiber diet and adequate hydration.

48-All of the following arteries are branches of the superior mesenteric artery,
except:

a. Ileocolic
b. Replaced left hepatic
c. Inferior pancreaticoduodenal
d. Jejunal
e. Replaced right hepatic

b—

49-Mesenteric cysts

a. Occur primarily in the mesocolon


b. Are discovered mainly in children
c. Can contain chyle or serous fluid
d. Require total enucleation for a cure
e. Are neoplastic

c—

50-Decompression for abdominal compartment syndrome should be


performed

a. If the urine output falls to 30 mL/hour


b. When the patient's respiratory rate increases to 24 breaths per minute
c. Based purely on physical findings
d. When bladder pressure exceeds 35 mm Hg
e. If the patient becomes hypoxemic

d—

51-In laparoscopic surgery, the vessel most likely to be punctured during


trochar placement is the

a. Aorta
b. Right common iliac artery
c. Vena cava
d. Left common iliac artery
e. Right common iliac vein

b—

52-A walled-off pelvic abscess secondary to sigmoid diverticulitis is Hinchey


stage

a. I
b. II
c. III
d. IV
e. V

b—

53-Following resolution of acute appendicitis, an interval appendectomy

a. In 90% of interval appendectomies and no pathologic abnormality.


b. Always reveals luminal occlusion of the appendix
c. Should be performed because of the high incidence of recurrent appendicitis
d. Is not indicated in patients older than 40 years
e. Needs to be performed open because of the fibrosis induced by appendicitis

54-True statements concerning the diagnosis and management of retroperitoneal


fibrosis include all of the following except:

A. Most patients present with dull, non-colicky back, flank, or abdominal pain
B. Evidence of impaired renal function with an elevated blood urea nitrogen is common
C. The diagnosis is most commonly suggested by intravenous pyelography although
contrast studies with CT scan or MRI are useful in further defining the disease
D. Most patients will need operative intervention.
E. The prognosis for nonmalignant retroperitoneal fibrosis is grim with progression of
disease until death occurring in most patients

:E

55-Lynch Syndrome all are true except?

a) It is due to mutation in MMR gene

b) Associated with APC gene

c) It has Autosomal Dominant Inheritence

d) Adenomas in patients with Lynch syndrome display high grade dysplasia than
adenoma in patients with sporadic colorectal cancer?

e) It occurs predominantly on right side, has increased incidence of synchronous


and metachronous disease.

56-About chronic pancreatitis all are true except?

a) Parlington Rochelle operation is longitudanal Pancreaticojejunostomy

b) Whipple Pancreaticoduodenectomy removes 60% of pancreatic parenchyma


and has 25% incidence of Diabetes

c) Frey's operation is Duodenal Presereving Pancreatectomy

d) Beger's operation is Duodenal Preserving Pancreatectomy

e) is a different disease entity from acute pancreatitis


57-Which of the folowing about Pancreatic Ascites is not true

a) Conservative treatment effective in only 1/4th of patients

b) ERCP should be done before surgery

c) It is exudative

d) Metaplastic cells are present

e) Pancreatic fluid has high amylase and high albumin

58-Tetanus all are true except?


a. Is due to an infection with a gram-negative spore forming rod
b. The organism produces a powerful exotoxin
c. The toxin prevents the release of inhibitory neurotransmitter
d. Clostridium tetani is sensitive to penicillin
e. Risus sardonicus is the typical facial spasm

59-Regarding gas gangrene


a. It is due to Clostridium botulinum infection
b. Clostridial species are gram-negative spore forming anaerobes
c. The clinical features are due to the release of protein endotoxin
d. Gas is invariably present in the muscle compartments
e. antibiotics alone are the main stay of treatment

D
60-A 34-year-old morbidly obese diabetic woman underwent a gastric bypass
about 12 hours ago. The operation was technically difficult but finally went
well. You are called because she now has a temperature of 99.2‫ ؛‬F, pulse of
134, and some pain in her incision and her back. She looks well; the incision is
clean; and her examination is otherwise negative. A bolus of 500 ml. of
dextrose/lactated Ringer's did not change her vital signs, except that her
pulse rose to 140 without an increase in urine output. Your next step should
be:

A. Another bolus of crystalloids.


B. Posterioanterior and lateral chest films.
C. Obtain white cell count, differential count, and electrolyte values.
D. Call the operating room and warn them that you need to re-explore for a leak.
E. Increase her pain medication.

Answer: D

61-All of the following statements about the embryology of Meckel's diverticulum


are true except:

A. Meckel's diverticulum usually arises from the ileum within 60 cm. of the ileocecal
valve.
B. Meckel's diverticulum results from the failure of the vitelointestinal duct to obliterate.
C. The incidence of Meckel's diverticulum in the general population is 5%.
D. Meckel's diverticulum is a true diverticulum possessing all layers of the intestinal wall.
E. Gastric mucosa is the most common ectopic tissue found within a Meckel's
diverticulum.

Answer: C

62-A 43-year-old woman presents with complaints of anal pain and spotting of
blood with defecation. Physical examination reveals a 2 3 cm area of ulceration
within the anal canal. The remainder of the physical examination is normal.
Incisional biopsy is positive for squamous cell carcinoma. Appropriate
management includes which of the following?

A. Abdominoperineal resection
B. Wide local excision, skin grafting, proximal diverting colostomy
C. Primary radiation therapy
D. Local excision and primary closure
E. chemotherapy

Answer: C
63-The initial goal of therapy for acute toxic cholangitis is to:

A. Prevent septicemia by decompressing the duct system.


B. Remove the obstructing stone, if one is present.
C. Alleviate jaundice and prevent permanent liver damage.
D. Prevent the development of gallstone pancreatitis.
E. Prepare the patient for urgent cholangiography.

Answer: A

64-The clinical picture of gallstone ileus includes all of the following Except?

A. Air in the biliary tree.


B. Small bowel obstruction.
C. A stone at the site of obstruction.
D. Acholic stools.
E. Associated bouts of cholangitis.

Answer: D

65- The following Nyhus classification of hernias is correct except for:


A. Recurrent direct inguinal hernia—Type IVa.
B. Indirect inguinal hernia with a normal internal inguinal ring—Type I.
C. Femoral hernia—Type IIIc.
D. Direct inguinal hernia—Type IIIa.
E. Indirect inguinal hernia with destruction of the transversalis fascia of Hesselbach's
triangle—Type II.
Answer: E

66- The following statements about the repair of inguinal hernias are true
except:
A. The conjoined tendon is sutured to Cooper's ligament in the Bassini hernia repair.
B. The McVay repair is a suitable option for the repair of femoral hernias.
C. The Shouldice repair involves a multilayer, imbricated repair of the floor of the inguinal
canal.
D. The Lichtenstein repair is accomplished by prosthetic mesh repair of the inguinal canal
floor in a tension-free manner.
E. The laparoscopic transabdominal preperitoneal (TAPP) and totally extraperitoneal
approach (TEPA) repairs are based on the preperitoneal repairs of Cheattle, Henry, Nyhus,
and Stoppa.
Answer: A

67-Which of the following statements is true concerning the diagnosis and


management of epigastric hernias?

A. A large peritoneal sac containing abdominal viscera is common


B. At the time of surgical repair, a careful search for other defects should be performed
C. Recurrent epigastric hernias after simple closure is uncommon
D. Patients with symptoms of a painful midline abdominal mass frequently will contain
incarcerated small bowel
Answer: b

68- Chylous ascites is the accumulation of chyle within the peritoneal cavity.
Which of the following statement(s) is/are true concerning chylous ascites?

a. The cisterna chyli lies at the anterior surface of the first and second lumbar vertebrae and
receives lymphatic fluid from the mesenteric lymphatics
b. Chylous ascites is most commonly associated with abdominal lymphoma
c. Paracentesis and analysis of chylous fluid typically reveals elevated triglycerides, protein,
and leukocyte levels. The cytology is seldom positive despite the presence of malignancy.
d. Treatment of chylous ascites with dietary manipulation will be successful in most cases
e. The mortality rate in adults with chylous ascites is in excess of 50%

Answer:d

69- A 48-year-old woman maintained on Warfarin for a history of cardiac valvular


replacement and a history of recent upper respiratory infection presents with
severe abdominal pain exacerbated by movement. Her physical examination
shows tenderness in the right paramedian area with voluntary guarding but no
peritoneal signs. Which statement is true concerning the diagnosis and
management of this patient.

a. Urgent laparotomy should be performed because of concern for arterial mesenteric


embolus
b. The correct diagnosis could likely be made by CT scan and operation avoided
c. The status of her anticoagulation should be checked and if her prothrombin time is
excessively prolonged,surgery is necessary
d. If untreated, hemodynamic instability is common
e. The usual cause is severe trauma.
Answer: b

70-True statements concerning the diagnosis and management of retroperitoneal


fibrosis include:

a. Most patients present with dull, non-colicky back, flank, or abdominal pain
b. Evidence of impaired renal function with an elevated blood urea nitrogen is common
c. The diagnosis is most commonly suggested by intravenous pyelography although
contrast studies with CT scan or MRI are useful in further defining the disease
d. Most patients can be managed nonoperatively
e. The prognosis for nonmalignant retroperitoneal fibrosis is grim with progression of
disease until death occurring in most patients
Answer: a, b, c

- In biliary atresia:
a. Medical treatment can be curative.
b. Operative success for bile drainage improved if surgery delayed after 3 months.
c. its rarely progress to cirrhosis.
d. indirect bilirubin level above 2 mg\dl is suggestive of the diagnosis.
e. liver biopsy shows inflammation and proliferation of the bile ducts.
Answer is: E

2- The best of the following to diagnose Hirschsprung’s disease is:


a. Aganglionosis of dilated bowel segment.
b. Presence of hypertrophied nerve trunks in rectal biopsy specimen.
c. Barium enema examination.
d. Failure to pass meconium in the first 24 hours.
e. Relaxation of internal sphincter during anorectal manometry study.
Answer is: B

3- A infant with intussusception:


a- A pathologic lead points is usually identified.
b- The site of origin is usually in the small bowel.
c- Triad of colicks, red currant Jelly stool and palpable abdominal mass presents in less than
50% of cases.
d- Surgical resection is mostly required.
e- Peaks around 4 years of age.
Answer is: C
4- 10 years boy presented to ER with abdominal pain after minor trivial blunt Lt lower chest wall
trauma, on exam: the HR 130, BP 70/40, the chest exam was unremarkable, the abdomen distended
and tender all over and absent bowel sounds. His mother reported history of cervical
lymphadenopathy with Flu like illness 2 weeks ago. Most likely this child’s scenario resulted from:
a- Mesenteric lymphadenitis.
b- Acute non obstructive appendicitis.
c- Meckle’s diverticulitis.
d- Splenic injury.
e- Fractured rib.
Answer is: D

5- 3 years old girl noticed by her mother to have a lump around her labia that’s firm and tender which
can’t be pushed back in and the physician appreciated tenderness in Rt lower quadrant, she most
likely have:
a- Acute appendicitis
b- Inguinal lymphadenopathy.
c- Incarcerated ovary in indirect inguinal hernial sac.
d- She experienced sexual abuse.
e- Bartholin gland abscess.
Answer is: C
6- A 5 year old child who suffered from vomiting for 2 days, when presented he was lethargic,
tachypnic and febrile and his abdominal exam revealed generalized tenderness and rigidity without
localization, the next step:
a- Order an abdominal CT scan with oral contrast and IV contrast.
b- Rule out intussusception by Barium enema.
c- Start broad spectrum IV antibiotics and observe.
d- Diagnostic lap. and accordingly.
e- Consider rigid esophagoscopy.
Answer is: D

7- A 6 years old girl with anterior neck mass with overlying redness which rises when she sticks out
her tongue, the mom reports same presentation 2 months earlier which improved with oral antibiotic
but persisting lump, which of the following is true:
A- Prepare for surgery to incise and drain the presumed abscess.
B- Excise the body of hyoid bone during surgery to avoid recurrence.
C- Start on IV steroids to decrease the edema.
D- Tracheostomy is mandatory.
E- Secure the airway with endotracheal tube.
Answer is: B

8- On the 30th day of a healthy looking newborn, he started to have persistent strongly projectile non-
bilious vomiting after feeding for 2 days, you suspect hypertrophic pyloric stenosis, which of the
following is true:
a- An impalpable abdominal mass rules out the above diagnosis.
b- A double bubble sign on X-ray film.
c- Immediate Pyloromyotomy must be done to resume feeding soon.
d- Rule out other important congenital anomalies (VACTERL).
e- Ultrasonography is superior to upper GI series.
Answer is: E

9- All the following are associated with Tracheoesophageal fistula except:


a- Congenital heart disease.
b- Hemivertebrae.
c- Omphalocele.
d- Horseshoe kidney.
e- Imperforate anus.
Answer is: C
10- The best management of irreducible inguinal hernia in a 2 months old infant is:
a- Reduction if successful and follow up till the age of one year.
b- Reduction if successful and surgery after 2-5 days.
c- Immediate surgical management.
d- Consider mesh repair to prevent recurrence.
e- Reduction if successful and surgery after 1 month.
Answer is: B
<Q>Hypercalcemia (acute): The first line of management Is:
<C+>Fluid volume correction
<C>Furosemide
<C>Steroids
<C>Hemodialysis
<C>Mithramycin

<Q>All the followings are hazards of bicarbonate therapy, Except:


<C>Shifts the hemoglobin dissociation curve to the left
<C>Paradoxical acidosis
<C>Hypernatremia and hyperosmolality
<C>Decreases the plasma level of ionized calcium
<C+>Activates the simultaneously administered catecholamines

<Q> Blood transfusion fatal reactions, the most common cause Is:
<C>An allergic reaction
<C>An anaphylactoid reaction
<C+>A clerical error
<C>An acute bacterial infection transmitted in blood
<C>Hypokalemia

<Q> Humoral immunity, which type of cell is NOT involved?


<C>Plasma cell
<C>B cell
<C>Memory cell
<C+>Killer T cell
<C>Th2 cell

<Q>Total parenteral nutrition (TPN): Deficiency of which of the following mineral


occurs in patient on TPN:
<C>Chromium
<C>Manganese
<C>Zinc
<C+>All of the above
<C>None of the above

<Q>Diathermy, all are true, Except:


<C+>Works by using a direct current
<C>The frequencies used range from 400kHz to 10MHz
<C>A cutting effect is produced by using a continuous output
<C>A coagulation effect is produced by using a pulsed output
<C>Maximal current density and heating effect is seen at the point of least cross-
sectional area

<Q>Immunoglobulins, all are true, Except:


<C>Rhesus antibodies are of IgG class
<C>IgG is the 1st response to invading organisms
<C+>IgA has the highest concentration in human secretion
<C>IgM is the best complement activator
<C>IgE mediates allergy by binding antigen on the membrane of the mast cell

<Q> 'Keratoderma blenorrhagicum' is seen in:


<C>Crohn's disease
<C>Ulcerative colitis
<C+>Reiters' disease
<C>Gonorrohea
<C>Diverticulosis

<Q>Tetanus, all are true, Except:


<C>It is a wound infection
<C>Low grade fever is present
<C>The shorter the incubation period the worse is the prognosis
<C>It may be mistaken for acute abdomen
<C+>It is mainly a carpopedal spasm

<Q>After hospitalization for a parasternal knife wound, all of the following may be
anticipated, Except:
<C>Distended neck veins
<C>Hypotension
<C>Pulsus paradoxus
<C+>Increased heart sounds
<C>Pleural effusion

<Q>The most appropriate method to diagnose small bowel injury in a conscious trauma
patient with seatbelt injury is:
<C>Diagnostic peritoneal lavage
<C>Ultrasound
<C>Computed tomography scan
<C+>Serial abdominal examination
<C>Plain abdominal film

<Q>Parotid Pleomorphic adenoma: All are true, Except:


<C>It is the most common parotid gland tumor and it accounts for 70-80% of all benign
parotid tumors
<C+>Malignant transformation is seen in 30 % of cases and is usually associated with
tumors that have been present for 1-2 years
<C>Pain does not indicate that a neoplasm is malignant; however, in patients with known
malignancy, pain is a poor prognostic sign
<C>Facial weakness is sign of malignancy, and indicates a poor prognosis
<C>It is more common in women

<Q>Tongue ulcers, all are true, Except:


<C>Dental ulcers commonly occur at the side of the tongue
<C>Syphilitic ulcers occur typically in the midline
<C+>Malignant ulcers are usually adenocarcinoma type
<C>Tuberculous ulcers occur on the dorsum
<C>If the ulcer does not heal promptly when the apparent cause is removed, it must be
biopsied

<Q>Cystic swelling appearing in front of the upper part of sternomastoid muscle in a


young adult, suggests:
<C+>Carotid body tumor
<C>Cystic hygroma
<C>Tuberculous lymph node
<C>Ranula
<C>Branchial cyst

<Q>Lateral aberrant thyroid tissue, the treatment is:


<C>Reassurance
<C>Thyroxin
<C+>Radical surgery
<C>Radioactive iodine
<C>External beam radiotherapy

<Q>Medullary thyroid carcinoma, all are true, Except:


<C>Is a tumor of the parafollicular C cells
<C>Produce calcitonin as the principle hormone
<C+>10% of cases are sporadic
<C>Can occur as part of the MEN type II syndrome
<C>Total thyroidectomy is the surgical treatment of choice

<Q>Thyroglossal duct cyst (TGDC): Other than the history and physical exam, which of
the following tests is considered an essential feature of the preoperative evaluation of a
patient with a suspected this condition?
<C+>Cervical ultrasound
<C>Thyroid scan
<C>Serum T3 and T4 levels
<C>Needle aspiration
<C>None of the above

<Q>Breast conservation surgery, all are associated with a lower rate of recurrence,
Except:
<C>Low proportion of in-situ carcinoma
<C>Tumor size less than 4 cm
<C>Use of radiotherapy
<C+>Younger age at presentation
<C>Use of adjuvant systemic treatment
<Q>Tuberculous Mastitis, all are true, Except:
<C>It usually affects women in the reproductive age group
<C>AIDS represents a predisposing factor
<C>Demonstration of a caseating granuloma is diagnostic
<C>The treatment of choice is wide surgical resection with anti-tubercular drugs
<C+>Polymerase chain reaction (PCR) on excised tissue has no diagnostic advantage

<Q>All the following malignancies are associated with chromosomal deletions, Except:
<C>Retinoblastoma
<C+>Squamous cell carcinoma of the lung
<C>Colorectal carcinoma
<C>Multiple endocrine neoplasia type 2-A
<C>Medullary carcinoma of thyroid

<Q>Bone metastases, all are true, Except:


<C+>Less than 5% of patients with malignant disease develop bone metastases
<C>10% of patients with bone metastases develop a pathological fracture
<C>Breast cancer is the commonest cause of bone metastases in women
<C>Radiological changes are seen late in the disease process
<C>Prostate cancer can cause osteosclerotic lesions

<Q>Chemotherapy is least effective in:


<C>Breast cancer of no special type (NOS)
<C>Duke’s C colonic carcinoma
<C>Seminomas
<C+>Thigh fibrosarcoma
<C>Pediatric rhabdomyosarcoma

<Q>Malignant melanoma, all are true, Except:


<C+>Lentigo maligna melanoma is a superficial lesion with a good prognosis
<C>Acral lentiginous melanomas most commonly occur on the dorsum of then hand
<C>The Breslow thickness is a good prognostic factor
<C>Sentinel lymph node biopsy is a useful means of assessing lymph node status
<C>Isolated limb perfusion with cytotoxic agents palliates recurrent limb disease

<Q>Sliding hernia, all are true, Except:


<C+>The anterior wall of the hernial sac is formed by a viscus
<C>It is due to slipping of parietal peritoneum on the underlying cellular tissue
<C>Usually large in size
<C>More on the left side
<C>Occasionally the large intestine is strangulated

<Q>Peritoneum, all are true, Except:


<C+>It has a surface area of 10m2
<C>Peritoneal fluid has a protein concentration of less than 3g\dl
<C>Normal peritoneal fluid WBC count is less than 3x 109/liter
<C>Peritoneal fluid has no fibrinogen clotting system
<C>Peritoneal fluid circulates via the diaphragmatic lymphatics

<Q>A 34-year-old morbidly obese diabetic woman underwent a gastric bypass about 12
hours ago. The operation was technically difficult but finally went well. You are called
because she now has a temperature of 99.2º F, pulse of 134, and some pain in her incision
and her back. She looks well; the incision is clean; and her examination is otherwise
negative. A bolus of 500 ml. of dextrose/lactated Ringer's did not change her vital signs,
except that her pulse rose to 140 without an increase in urine output. Your next step
should be:
<C>Another bolus of crystalloids
<C>Posteroanterior and lateral chest films
<C>Obtain white cell count, differential count, and electrolyte values
<C+>Call the operating room and warn them that you need to re-explore for a leak
<C>Increase her pain medication

<Q>Bleeding from upper gastrointestinal tract, all are true, Except:


<C>Acute gastrointestinal (GI) bleeding is a potentially life-threatening
abdominal emergency that remains a common cause of hospitalization
<C>The incidence of upper gastrointestinal bleeding (UGIB) is approximately
100 cases per 100,000 population per year
<C>Bleeding from the upper GI tract is approximately 4 times as common as
Bleeding from the lower GI tract
<C+>Mortality rate from UGIB is 1% overall
<C>An aging patient population with an increasing prevalence of associated medical
comorbidities has kept the mortality figures largely unchanged for the past 30 years,
despite technologic advances in endoscopy and other minimally invasive procedures

<Q>Chronic duodenal ulcer is associated with:


<C>Hyperthyroidism
<C+>Hyperparathyroidism
<C>Insulinoma
<C>Phaeochromocytoma
<C>Cholelithiasis

<Q>Gastric leiomyomas, all are true, Except:


<C>They are the most common type of gastric tumor of the stomach at autopsy
<C+>The leiomyoblastoma cell type reflects malignant transformation of gastric
leiomyomas.
<C>A conservative surgical approach is indicated for their resection since regional
lymphadenectomy has not been proved reliable even when they turn out to be malignant.
<C>Severe hemorrhage may occur from deep ulcerations overlying the intramural tumor.
<C>Gastric leiomyomas are not encapsulated, even though on section they appear to be
well-circumscribed.

<Q>Gastric polyps, One of the following is True:


<C>Like their colonic counterparts, gastric epithelial polyps are common tumors.
<C>They are analogous to colorectal polyps in natural history.
<C>Endoscopy can uniformly predict the histology of a polyp based on location and
appearance.
<C>In a given patient, multiple polyps are generally of multiple histologic types.
<C+>Gastric adenomatous polyps greater than 2 cm. in diameter should be excised
because of the risk of malignant transformation.

<Q>Gastrin-releasing peptide (GRP), all are true, Except:


<C>In species other than man and dog, GRP is commonly referred to as bombesin.
<C>GRP serves as a neurotransmitter.
<C+>GRP inhibits pancreatic secretion when given intravenously.
<C>GRP stimulates gastric acid secretion when given intravenously.
<C>GRP is released in response to cholinergic stimulation of the parietal cells to
stimulate release of gastrin.

<Q>The most effective therapy for morbid obesity, in terms of weight control, is:
<C>Intensive dieting with behavior modification.
<C>A multidrug protocol with fenfluramine, phenylpropanolamine, and mazindol.
<C>A gastric bypass with a 40-ml. pouch, a 10- to 20-cm. Roux-en-Y gastroenterostomy.
<C+>A gastric bypass with a 15-ml. pouch, a 40- to 60-cm. Roux-en-Y
gastroenterostomy.
<C>Daily exercise with strong emphasis on utilizing all four limbs.

<Q>Vascular compression of the duodenum in adults, One of the following has been
used successfully in its treatment:
<C>Subtotal gastrectomy and Roux-en-Y gastrojejunostomy
<C>Total parenteral nutrition
<C>Division of the ligament of Treitz and duodenal mobilization
<C>Percutaneous endoscopic gastrostomy
<C+>Duodenojejunostomy

<Q>Zollinger-Ellison syndrome, treatment of choice is


<C>Total gastrectomy
<C>Ranitidine
<C>Partial gastrectomy
<C>Vagotomy with gastric drainage
<C+>Proton-pump inhibitors (PPIs)

<Q>The optimal management of traumatic duodenal hematoma is:


<C>Angiography and embolization
<C>Laparotomy and evacuation
<C>Laparotomy and gastrojejunostomy
<C+>Observation
<C>Aspiration under U/S
<Q>Crohn’s disease, all are common complications, Except:
<C>Recurrent intestinal obstruction
<C>Intestinal stricture
<C>Perianal disease
<C>Entero-enteric fistula
<C+>Free perforation in to the peritoneal cavity

<Q>Intestinal bypass, all are true, Except:


<C>The operation produced weight loss similar to that of the gastric bypass.
<C>The operation produced severe metabolic disturbances, including hypocalcemia,
increased bile salts and glycine synthesis.
<C>Bacterial overgrowth in the bypassed segment led to liver failure.
<C>The operation demonstrated that an adult human could survive with 40 to 50 cm. of
small intestine.
<C+>Is the treatment of choice for morbid obesity

<Q>Malabsorption, simple tests include all the following, Except:


<C>Microscopic examination.
<C>D-xylose absorption.
<C>A 72-hour stool collection for fats.
<C>Small bowel x-ray series.
<C+>Enteroscopy

<Q>Mesenteric ischemia, all are true, Except:


<C>Characterized by severe pain out of proportion of the physical findings
<C>The presence of peritoneal irritation signs is a grave situation
<C+>Non occlusive visceral ischemia has an excellent prognosis
<C>The most common site for acute arterial obstruction is the origin of the superior
mesenteric artery
<C>Angiography is the standard investigation for acute arterial occlusion

<Q>Perforation in enteric fever, the commonest site is:


<C>Jejunum
<C>Cecum
<C+>Terminal ileum
<C>Ascending colon
<C>Stomach

<Q>Short bowel syndrome, all are true, Except:


<C>Might be seen in Crohn’s disease
<C>Up to 70% of small bowel can be safely resected if the terminal ileum and ileocecal
valve are left intact
<C+>Massive small bowel resection leads to gastric hypo-secretion and increased bowel
PH
<C>Unabsorbed fats are irritating to colon, increasing diarrhea and steatorrhea
<C>Fluid and electrolyte problems are due to shortened transit time and diarrhea

<Q>The lamina propria between the intestinal epithelium and the muscularis mucosae
contains all the followings, Except:
<C>Blood and lymph vessels.
<C>Nerve fibers.
<C+>Enterochromaffin cells.
<C>Macrophages.
<C>Connective tissue.

<Q>Carcinoid of the appendix, One of the followings is True:


<C>Carcinoids of appendix most often occur at the base
<C>Nearly 50% are multiple
<C>Appendiceal carcinoids are often malignant
<C>30% of appendiceal carcinoids present with the classic carcinoid syndrome
<C+>If regional lymph nodes are enlarged right hemicolectomy is indicated

<Q>Anorectal functional testing, all are true, Except:


<C>Anorectal manometry is often performed through open-tipped multilumen catheters
perfused with fluid.
<C+>Anorectal manometry can not differentiate between segmental and global defects of
the anal sphincter in patients with incontinence.
<C>Electromyography can demonstrate persistent contraction of the pubis rectalis
muscle during defecation, which is diagnostic of paradoxical pelvic floor contraction.
<C>Measurement of sensory thresholds may reveal insensitivity in patients with chronic
constipation.
<C>Catheters are connected to a transducer and register internal and external sphincter
pressures and presence of the anorectal inhibitory reflex.

<Q>Colon physiology, all are true, Except:


<C>Colonic recycling of urea is accomplished by the splitting of urea by bacterial
ureases.
<C>Fermentation by colonic bacteria may rescue malabsorbed carbohydrates.
<C+>The preferred fuel of the colonic epithelium is glucose.
<C>The colonic epithelium utilizes n-butyrate for the absorption of sodium and water
<C>Insoluble fibers create bulk in the stool.

<Q>Colorectal cancer, adjuvant chemotherapy: One of the following recommendations is


True:
<C>Patients with Stage I or Dukes A and B1 disease should receive adjuvant treatment
for 1 year with levamisole combined with 5-FU.
<C+>Patients with Stage III or Dukes C disease should receive adjuvant treatment for 1
year with levamisole combined with 5-FU.
<C>There is no role for adjuvant therapy for colon cancer at any stage.
<C>Adjuvant chemotherapy is active in colon cancer only when combined with
radiotherapy.
<C>5-Fluorouracil (5-FU) achieves an 80% to 90% response in patients with advanced
disease.

<Q>Diverticular disease, all are true, Except:


<C>It is more common in Western Europe than in Africa
<C>A low-fiber diet may predispose to development of diverticulosis
<C>It involves sigmoid colon in more than 90% of patients
<C+>Sixty percent develop diverticulitis sometime during their lifetime
<C>It is the most common cause of massive lower gastrointestinal hemorrhage

<Q>Familial adenomatous polyposis (FAP), One of the followings is True:


<C>Inherited in an autosomal dominant manner, this genetic defect is of variable
penetrance, some patients having only a few polyps whereas others develop thousands.
<C>The phenotypic expression of the disease depends mostly on the genotype
<C+>Appropriate surgical therapy includes total abdominal colectomy with ileorectal
anastomosis and ileoanal pull-through with rectal mucosectomy
<C>Panproctocolectomy with ileostomy is not appropriate therapy for this disease
<C>Pharmacologic management of this disease may be appropriate in some instances

<Q>Fistula in Ano, all are true, Except:


<C>Intersphincteric is the commonest type
<C>Usually preceded by a perianal abscess
<C+>Fistulas secondary to other disease are more common than primary ones
<C>Best treated by unroofing of the fistula tract
<C>Surgery may end up with anal incontinence

<Q>Perianal abscess, all are true, Except:


<C>Usually caused by mixed microbes
<C>Commonly starts as anal gland infection
<C>Recurrent and multiple could be caused by inflammatory bowel disease
<C+>Antibiotic administration may treat abscess in its early stage
<C>Improperly drained abscess can lead to fistula formation

<Q>Pouchitis can frequently complicate the ileal pouch/and anastomosis procedure, with
regard to this condition, One statement is True:
<C>It occurs with equal frequency in patients with familial polyposis and ulcerative
colitis
<C>It is found more frequently in patients with capacious S-shaped pouches than in those
with J-pouches
<C+>Most patients are treated successfully with oral metronidazole
<C>The responsible pathogen is usually Bacteroides
<C>Recurrent persistent pouchitis invariably necessitates pouch excision

<Q>Rectal injuries, all are true, Except:


<C>The most common cause is iatrogenic
<C>A diverting colostomy is usually needed
<C+>Repair of the defect should be done
<C>The role of the distal rectal washout is controversial
<C>Drainage of the retroperitoneal space is not mandatory

<Q>Technique of anterior resection, all are true, Except:


<C>Mobilization of the splenic flexure is done on the middle colic artery
<C>Division of the inferior mesenteric artery at its origin is not an oncological need
<C+>Division of the inferior mesenteric vein at the lower end of the pancreas is needed
from an oncological point of view
<C>The pelvis must be drained
<C>Circular staplers have the same anastomotic leak rate as hand sewn ones

<Q>The presence of generalized gastrointestinal polyposis with alopecia, atrophy of


finger nails, cutaneous pigmentations, characterizes which syndrome:
<C+>Cronkhite-Canada syndrome
<C>Gardner's syndrome
<C>Peutz-Jeghers syndrome
<C>Meigs syndrome
<C>Mallory-Weiss syndrome

<Q>Ulcerative colitis (Toxic): Initial management should include all the followings,
Except:
<C>Broad-spectrum antibiotics.
<C>Large intravenous doses of corticosteroids
<C>Intravenous fluid and electrolyte resuscitation.
<C+>Opioid antidiarrheals
<C>Total parenteral nutrition to improve nutritional status

<Q>Cecal diverticular: One of the following is True:


<C>Cecal diverticuli are usually multiple
<C>Preoperative diagnosis of cecal diverticulitis is possible is 80% of cases
<C+>Simple diverticullectomy with closure of defect is the procedure of choice in
localized cecal diverticulitis
<C>Removal of the appendix is contraindicated if the intraoperative diagnosis is cecal
diverticulitis
<C>Cecal diverticula are usually acquired mucosal herniation through muscularis propria
of cecum

<Q>A 45-year-old man is scheduled to undergo laparoscopic cholecystectomy for


symptomatic cholelithiasis. He has a long midline scar extending from the xiphoid
process to the umbilicus due to open splenectomy for traumatic splenic rupture 30 years
ago. What are the alternatives for initial trocar placement?
<C>Extend the vertical incision with careful peritoneal dissection
<C>Insert the initial trocar high in the epigastric region
<C>Insert the initial trocar in the right anterior axillary line!
<C+>All of the above
<C>The position of the initial trocar need not be changed

<Q>Cholangiocarcinoma (CCA): all are true, Except:


<C>A strong relationship exists between CCA and primary sclerosing cholangitis (PSC)
<C>Certain chemical exposures have been implicated in the development of CCA,
primarily among workers in the aircraft, rubber, and wood finishing industries
<C>CCA occasionally has developed years after administration of the radiopaque
medium thorium dioxide (ie, Thorotrast)
<C>Congenital diseases of the biliary tree, including choledochal cysts and Caroli
disease, have been associated with CCA
<C+>Alpha1-antitrypsin deficiency is associated with gallbladder cancer but not CCA

<Q>Choledochal cysts, all are true, Except:


<C>The morbidities associated with choledochal cysts are age dependent
<C>The most worrisome complication of choledochal cysts is cholangiocarcinoma
<C+>Choledochal cysts are more prevalent in males
<C>Most patients with choledochal cysts have some clinical manifestation of the disease
in childhood
<C>Todani Type I choledochal cysts are the most common and represent 80-90% of the
lesions.

<Q>Dye used for intravenous cholangiography is:


<C>Iapanoic acid
<C+>Meglumine
<C>Na diatrizoate
<C>Urograffin
<C>Giemsa

<Q>Hepatic amebiasis: One of the following is True:


<C>The infection is most commonly transmitted by person-to-person contact rather than
by contaminated food and water
<C+>An increasing incidence is seen among homosexual men
<C>Active disease is present when trophozoites or cyst are found in the stool
<C>The abscesses tend to rupture intraperitoneally when found in the left lobe
<C>Diagnosis is usually made by aspirating protozoans from abscess

<Q>Hydatid disease of liver: Casoni's test is positive in:


<C>35%
<C>50%
<C+>65%
<C>95%
<C>100%

<Q>Treatment of Carbon dioxide embolus should include all the followings, Except:
<C>Cardiopulmonary resuscitation if necessary
<C>Evacuate the pneumoperitoneum
<C>Place patient in head-down, left lateral decubitus position
<C>Place an emergent central venous line
<C+>Stick a needle into the right chest

<Q>Liver neoplasms (Benign): One of the following is True:


<C>Hepatic adenomas (HAs) are usually multiple and can reach a diameter of 30 cm
<C+>Unlike patients with adenomas, patients with focal nodular hyperplasia (FNH) are
usually asymptomatic
<C>Hemorrhage, necrosis, and malignant change are more common in FNH than
adenomas
<C>FNH derives its blood supply peripherally, where as adenomas derive blood supply
centrally
<C>Adenomas blend with the surround normal hepatic tissue, whereas FNH appears
distinct

<Q>Yesterday, a 38-year-old woman underwent a laparoscopic cholecystectomy for


cholelithiasis and was discharged home 8 hours after surgery. She returns this morning
complaining of worsening abdominal pain. The oral narcotics that the patient was
prescribed are ineffective in controlling the pain. The patient's temperature is 38.3 C.
Laboratory studies reveal an elevated white blood cell count. Abdominal ultrasonography
shows a large subhepatic fluid collection. The fluid is percutaneously aspirated and
reveals enteric contents.
What step should be taken next in the management of this patient? Please choose the
single most appropriate answer to the question:
<C+>Immediate laparptomy
<C>Observation in the hospital until pain improves
<C>Intravenous antibiotics and close observation
<C>ERCP
<C> PTC

<Q>Acute pancreatitis, all are true, Except:


<C>The initiating event may be anything that injures the acinar cell and impairs the
secretion of zymogen granules, such as alcohol use, gallstones, and certain drugs.
<C>Can develop when ductal cell injury leads to delayed or absent enzymatic secretion,
such as with the CFTR gene mutation
<C>Lysosomal and zymogen granule compartments fuse, enabling activation of
trypsinogen to trypsin
<C+>Intracellular somatostatin triggers the entire zymogen activation cascade
<C> Secretory vesicles are extruded across the basolateral membrane into the interstitium,
where molecular fragments act as chemoattractants for inflammatory cells

<Q>Cancer of the exocrine pancreas, all are true, Except:


<C>It is usually an adenocarcinoma of duct cell origin
<C>Most common site is the head
<C+>The majority are multicentric
<C>It includes mucinous and adenosquamous varieties
<C>CEA is usually elevated but not always

<Q>Following a motor vehicle accident, a truck driver complains of severe abdominal


pain. The amylase level is markedly increased to 800 u. pancreatic trauma is suspected.
Pancreatic trauma:
<C>Is usually an isolated single-organ injury
<C>Often requires a portocaval shunt operation
<C+>May easily be overlooked at operation
<C>Is proven by the elevated amylase level
<C>Blunt trauma is about three times more common than penetrating trauma and is
associated with a higher mortality rate.

<Q>Mucin secreting tumors of the pancreas (Intraductal papillary mucinous tumors


(IPMT) of the pancreas): all are true, Except:
<C>It is more common in males
<C>Prolonged symptoms of pancreatitis should prompt a work up of this tumor
<C>The diagnosis should be suspected when there is mucin extravasation or plug on
ERCP
<C+>Conservative management (no surgery) is enough since the tumor is benign and
indolent
<C>The neoplasms were typically more than 5 cm in diameter

<Q>Normal functions of the adult spleen include al of the following, Except:


<C+>Hematopoiesis
<C>Erythrocyte processing and repair
<C>Production of opsonins
<C>Destruction of aged and damaged erythrocytes
<C>Clearance of circulating bacteria

<Q>Overwhelming post-splenectomy sepsis:


<C>Commonly occurs after splenectomy for trauma
<C>Does not occur if accessory spleens are present
<C+>Can be fatal within hours of onset
<C>Is most common in elderly patients
<C>Most fatal cases occur 10 to 15 years after splenectomy

<Q>Hyperaldosteronism (Conn's syndrome), all are true, Except:


<C+>The commonest cause is hyperplasia of the zona glomerulosa of the adrenal cortex
<C>Its incidence is more in female
<C>It constitutes 1% of patients investigated for hypertension
<C>There is increase in urinary potassium
<C>There is increase in blood sodium

<Q>Pheochromocytoma, all are true, Except:


<C>Urinary catecholamine estimation is 90% sensitive for its diagnosis
<C+>Iodocholesterol isotope scan is used for its localization
<C>It can be associated with multiple endocrine neoplasia type 2
<C>10% of cases are caused by extra-adrenal source
<C>Adrenalectomy is the treatment of choice

<Q>Adult respiratory distress syndrome (ARDS): all are true, Except:


<C>Its name is misleading, because children, as well as adults, may be affected
<C>The basic fault is a breakdown of the endothelial-alveolar barrier (alveolo-
capillary junction)
<C>The lungs as a whole become very "stiff," and it becomes much harder for
the patient to breathe
<C>Can be caused by pancreatitis
<C+>The pulmonary artery occlusion pressure is more than 20 mm Hg

<Q>The most common tumor in the superior mediastinum is:


<C>Pericardial cyst
<C>Ectopic parathyroid gland
<C>Thymoma
<C>Aortic arch aneurysm
<C+>Retrosternal extension of goiter

<Q>Thoracic outlet syndrome, all are true, Except:


<C+>Is more common in men than women
<C>Neurological symptoms are more common than arterial or venous symptoms
<C>Symptoms are often worsened by carrying weights or lifting the arms above head
height
<C>Diagnostic tests include the Roo's and Adson's maneuver
<C>Onset of symptoms is precipitated by trauma in about one-third of patients

<Q>Ischemic heart disease: All the followings are known risk factors, Except:
<C+>Hyperthyroidism
<C>Family history of coronary artery disease
<C>Smoking tobacco
<C>Hyperlipidemia
<C>Diabetes mellitus

<Q>Patent ductus arteriosus (PDA): In a premature infant with hyaline membrane


disease and inability to be weaned from mechanical ventilation, which of the following
would suggest hemodynamically significant (PDA)?
<C>Systolic murmur
<C+>Hyperactive precordium with bounding peripheral pulses
<C>Jaundice
<C>Diminished femoral pulses
<C>Diastolic murmur

<Q>Penetrating cardiac injury: The most useful incision in the operating room is:
<C>Left anterior thoracotomy
<C>Right anterior thoracotomy
<C>Bilateral anterior thoracotomy
<C+>Median sternotomy
<C>Subxyphoid

<Q>Biliary atresia, the most accurate test for diagnose is:


<C>HIDA scan
<C>Abdominal ultrasound
<C+>Liver biopsy
<C>CT scan
<C>PTC (percutaneous transhepatic cholangiogram)

<Q>Esophageal atresia (Isolated) without a tracheoesophageal fistula: the most


appropriate initial management is:
<C>Emergency exploration and anastomosis
<C+>Cervical esophagostomy and gastrostomy
<C>Gastrostomy only
<C>Immediate esophageal replacement
<C>Feeding jejunostomy

<Q>Intussusception, all are true, Except:


<C+>Is most common in children from 6 to 12 years
<C>Presents with colicky abdominal pain, rectal bleeding and an abdominal mass
<C>10% present with diarrhea and vomiting suggestive of gastroenteritis
<C>If no shock or peritonitis, hydrostatic reduction can be attempted
<C>A Meckel's diverticulum can induce an intussusception

<Q>Massive upper gastrointestinal bleeding in children, the most common cause is:
<C>Peptic ulcer disease
<C+>Esophago-gastric varices
<C>Gastro-esophageal reflux
<C>Congenital arterio-venous malformation
<C>Esophageal atresia

<Q>Burn: Full Thickness Burn (FTB): One of the following is True:


<C>Blister formation is characteristic sign
<C>Sensation usually is intact
<C>Capillary refill time is less than 3 seconds
<C>Bleeding on pin prick
<C+>Destruction of both layers of epidermis and dermis

<Q>Cleft lip abnormality is caused by:


<C>Failure of fusion of palatal shelves
<C>Failure of fusion of medial nasal process with frontonasal process
<C+>Failure of fusion of medial nasal process with the maxillary process
<C>Failure of fusion between both medial and lateral nasal processes
<C>Failure of fusion maxillary process with the mandibular process

<Q>Hemangiomas: Which of the following would be helpful in distinguishing a deep


hemangioma from AV-malformation:
<C+>Computerized tomography (CT scan) with contrast injection
<C>Clinical examination X-ray investigation
<C>Arteriography
<C>Ultra sound scan (USS)
<C>X-ray investigation

<Q>Number of muscles inserted on the index finger:


<C>Three
<C>Four
<C>Five
<C>Six
<C+>Seven

<Q>A disease identified by the presence of Donovan bodies and caused by an organism
similar to Klebsiella pneumoniae is:
<C>Chancroid
<C+>Granuloma inguinale
<C>Lymphogranuloma venereum
<C>Malakoplakia
<C>Bacterial vaginosis

<Q>A 50 year old male patient presented to the emergency room with acute, ischemic leg
pain found to have a history of painful calf on walking for the last 3 years. This patient
could have one of the following, Except:
<C+>Thromboangiitis obliterans (Buerger's disease)
<C>Acute arterial thrombosis
<C>Acute arterio-arterial emboli
<C>Acute embolus originated from the heart
<C>Acute femoral artery thrombosis following diagnostic arteriogram

<Q>A previously healthy 16-year-old high school student begins to notice pain in his
right calf during football practice after running two laps around the track. The pain is
relieved immediately when he stops running. The most likely diagnosis is:
<C>Arterial embolus
<C>Muscle cramps
<C+>Popliteal entrapment syndrome
<C>Popliteal aneurysm
<C>Deep venous thrombosis

<Q>Axillary-subclavian vein thrombosis is associated with all the following, Except:


<C>Pancoast syndrome
<C>Parenteral nutrition
<C>Cervical rib
<C+>Cervical discopathy
<C>Congestive cardiac failure

<Q>Arteriovenous fistula (congenital of the lower limb): You would suspect it if all the
following are present, Except:
<C>Unusual varicosities were present in an enlarged limb
<C>Painful ulcer present with normal foot pulses
<C+>Occlusion of the femoral artery leads to tachycardia
<C>The pulse pressure is great
<C>Machinery murmur is heard

<Q>Chronic venous disease: One of the following is True:


<C>Bicuspid valves in perforating veins normally direct flow from the deep to the
superficial venous system
<C+>The normal decrease in distal venous pressure with exercise exceeds 60%
<C>Following exercise, the distal venous pressure usually returns to the resting level
within 10 seconds
<C>Brownish skin discoloration of chronic venous disease is due to excess melanocyte
activity
<C>If the feet and toes are involved, venous insufficiency is more likely than
lymphedema

<Q>Leriche's syndrome: Symptoms or sings of atherosclerotic occlusive disease of the


bifurcation of the abdominal aorta (Leriche's syndrome) include One of the followings:
<C+>Claudication of the buttock and thigh
<C>Causalgia of the lower leg
<C>Retrograde ejaculation
<C>Gangrene of the feet
<C>Dependent rubor of the feet

<Q>Renal fibromuscular dysplasia: The single most characteristic finding is:


<C>Complete occlusion of the renal artery
<C>Renal artery aneurysms
<C+>Area of narrowing alternating with area of dilatation in the main renal arteries
<C>Involvement of the renal artery origin
<C>Arterio-venous malformation in the kidney

<Q>Vascular grafts: all are true, Except:


<C+>PTFE grafts are more porous than Dacron
<C>Vein grafts have better long-term patency
<C>Graft failure less than a month after surgery is usually due to a surgical error
<C>Graft failure within the first year is often due to neo-intimal hyperplasia
<C>Graft failure beyond one year is due to progression of the underlying disease
<Q>All are indications for surgical reconstruction of the esophagus, Except:
<C>Continuing requirement for frequent dilation of an extensive esophageal stricture for
a minimum of 2 years
<C>Failure or refusal of the patient to comply with a treatment regimen of regular
dilation
<C>Development of a fistula between the esophagus and tracheobronchial tree
<C+>Iatrogenic perforation of the esophagus during attempted dilation
<C>Very tight stricture not amenable for dilatation

<Q>Esophageal cancer: One of the followings about its surgical treatment is True:
<C>The finding of severe dysplasia in association with Barrett's mucosa is an indication
for an antireflux operation to prevent subsequent development of carcinoma
<C>The number of lymph nodes resected with the tumor determines survival
<C+>The morbidity and mortality rates for cervical esophagogastric anastomotic leak are
substantially less than those associated with intrathoracic esophagogastric anastomotic
leak
<C>The leading complications of intrathoracic esophagogastric anastomosis are bleeding
and wound infection
<C>Transhiatal esophagectomy without thoracotomy achieves better long-term survival
than transthoracic esophagectomy

<Q>oesophageal reflux disease (GERD): which of the following tests is the most
sensitive for its detection?
<C>Barium swallow
<C>Manometry
<C+>24 hours PH-monitoring
<C>Acid perfusion (Bernstein) test
<C>Standard antireflux test

<Q>The best management for a 48-hour-old distal esophageal perforation Is:


<C>Antibiotics and drainage
<C>Division of the esophagus and exclusion of the perforation
<C+>Primary repair with buttressing
<C>Resection with cervical esophagostomy, gastrostomy, and jejunostomy
<C>T-tube fistula and drainage

You might also like